Med Surg III Final

Ace your homework & exams now with Quizwiz!

glaucoma - angle closure s/x

- medical emergency - p!, bradycardia, reduced central VA, pupil is vertically oval in fixed semi-dilated position and unreactive to light and accommodation

Macular Degeneration

- most common cause of visual loss in ages >60 - s/x --> drusen spots, gradual p!-less loss of central vision over months/years - many retain peripheral vision -difficulty reading small print

Cataracts - definition

- opacity of lens that distorts image projected onto retina - can be senile, congenital, traumatic, or 2ndary - most common risk factors: Hx myopia, smoking, Hx diabetes, obesity, Hx alcohol abuse (Chart 58-8 pg 1772)

Pre-Op patients should be screened for possible critical allergies?

Pre-Op pts should be screened for critical allergies: Latex, Iodine, and allergies to anesthesia that may result in Malignant Hyperthermia.

Which action by the nurse is most important following hypophysectomy?

Performing a routine neurologic assessment. They are important to detect complications after surgery involving the central nervous system.

The nurse's first action for a patient who presents to the Emergency Department with manifestations of corneal perforation is to: Contact the patient's ophthalmologist. Place the patient in a supine position. Cover the eye with gauze. Administer tears for dry eyes.

Place the patient in a supine position. Rationale: If corneal perforation is suspected, the nurse should first place the patient in the supine position, then close the eye, and cover it with a dry, sterile dressing. Thereafter, the nurse should notify either the patient's ophthalmologist or an on-call ophthalmologist to see the patient. The complications of corneal perforation include infection of deep eye structures and loss of ocular contents. Nothing should be placed in the eye.

Excessive thirst is called...

Polydipsia

Hyperglycemia symptoms

Polydipsia Polyuria Lethargy Fruity Breath Abdominal Pain

Symptoms of uncontrolled DM

Polydipsia, polyuria, polyphagia, glycosuria, nocturia, fatigue, blurred vision, abdominal pain, headache

Excessive hunger is called...

Polyphagia

Which symptom is commonly associated with hyperglycemia?

Polyuria

Which assessment finding should the nurse expect to see in the patient with uncontrolled diabetes insipidus?

Polyuria, Polydipsia and Dehydration. (Diabetes insipidus is caused by lack of antidiuretic hormone, which results in excess urination, thirst and dehydration.)

PET Scan

Positron emission tomography. Radiation emitted from radioactive substances is detected after being injected.

1. Low temperature 2. pulse rate 3. Weight gain, 4. Thickened skin 5. Personality changes 6. Constipation 7. Low T3 (below 70) 8. Low T4 (below 5)

What are the signs and symptoms of hypothyroidism? (8)

1. Hypotension 2. Hypothermia 3. Bradycardia 4. Mental depression 5. Mood swings 6. Hyponatremia 7. Hypoglycemia 8. Coma

What are the signs and symptoms of mxedema coma?

Prevents T4 from becoming T3

What does PTU or methiamazole prevent?

Iodine to make T3 and T4

What does the thyroid need in order to make T3 and T4?

glucagon

What injection can be given IM in emergency situations with hypoglycemia?

Evaluation: adrenal hormones

evaluate effectiveness of glucocorticoid therapy. If inflammation hasn't improved, a drug change may be necessary Continue monitoring for side effects, especially if high dose

Evaluation: thyroid replacement

evaluate effectiveness of thyroid drug and drug complaince continue monitoring for side effects from drug accumulation/overdose

polyphagia

excessive hunger

polydipsia

excessive thirst (as in cases of diabetes or kidney dysfunction)

Nocturia

excessive urination at night

nocturia

excessive urination at night

Byetta

exenatide class: antidiabetic: amylin analog Dose: subQ: 5 mcg bid w/in 60 min before morning/evening meal; max: 20 mcg/d Action: supresses glucagon secretion

RD risk factors

extreme myopia, aphakia after cataract surgery, stress, family Hx, HTN, diabetes

hyperopia - definition

farsightedness; blurry close vision

Diabetes can be diagnosed when...

fasting glucose levels are 126 mg/dL or higher; Or when Glucose Tolerance Test is200 mg/dL or higher; Or when HbA1c levels are 6.5% or higher

How would someone classify as pre-diabetic?

fasting glucose: 100-125 oral glucose tolerance: 140-200

What are the signs and symptoms of type 2 diabetes?

fatigue, polyuria, polydipsia, vaginal infections & UTIs, blurry vision since high glucose levels will cause this, skin wounds that heal poorly; HOWEVER a lot of times there are NO symptoms at all and the person does not even know they have diabetes until they go in to the doctor for an unrelated problem such as a vascular or neural complication.

S&S of hypothyroidism

fatigued, lethargic, personality and mental changes (impaired memory, slowed speech, decreased initiative, somnolence), low exercise tolerance, SOB on exertion, anemia, decrease GI motility, constipation, cold intolerance, hair loss, dry and coarse skin, brittle nails, weight gain

postprandial

following a meal (especially dinner)

excess of GH in childhood

gigantism

excessive GH in children

gigantism

glaucoma - angle closure Tx

give hyperosmotics, azetazolamide, topical occular hypoTN, and betaxlol (more meds pg 1771, chart 58-4)

how to treat central DI

give hypoteonic IVF (0.45% NS or D5W) hormone replacement-DDAVP (desmopressin=ADH) oral, IV or intranasal

What insulins can not be mixed with other insulins?

glargine (Lantus) detemir (Levemir)

what are examples of 2nd generation sulfas?

glucatrol xl micronase

4% - 6% is the normal level of...

glycohemoglobin

enlarged thyroid gland

goiter

patient teaching regarding RAI

has a delayed response, max effect may not be seen for 2-3 months; usually treated with antithyroid meds until it takes effect

What is the main feature that defines all types of diabetes?

having high blood sugar

SIADH: high or low extreacelular fluid volume

high

SIADH: high or low- glomerular filtration rate

high

hyperparathyroidism: high or low serum calcium

high

hyperthyroidism: high or low T4

high

hyperthyroidism: high or low basal metabolic rate

high

hypothyroidism: high or low TSH

high

diet for hyperthyroidism

high-calorie diet (4000-5000 cal/day)

DI: high or low: serum osmolality

high: concentrated

Sulfonylureas: how do they work to help type 2 diabetes?

they improve insulin action and decrease glucose production by the liver

What type of drug is Glucovance?

this is a mixed agent which combines classes of oral anti-diabetic drugs

What is insulin lipoatrophy?

this is the loss of fat tissue in areas of repeated injections causing changes in subQ fat - should rotate injection site but within the same area

Describe "Dawn Phenomenon"

this is when the patient becomes hyperglycemic between 5-6 am because of a nighttime release of growth hormone. The treatment: give more insulin for the overnight period. For instance, give NPH at 10 pm. The diagnosis is made by checking blood glucose during the night.

acute, rare condition in which all hyperthyroid manifestations are heightened

thyrotoxic crisis

physciological effects of clinical syndrome of hypermetabolisms that results form excess circulating levels of T4, T3, or both

thyrotoxicosis

Contra:synthroid

thyrotoxicosis, MI, severe renal disease caution: CVD, hypertension, angina pectoris

Avandia (rosiglitazone)

Thiazolidinedione antidiabetic

What statement by the patient with hypothyroidism indicates to the nurse that the plan of care is effective?

"I feel so much better now that my energy is returning." A patient with hypothyroidism typically is fatigued; returning energy is a sign of effective therapy.

A patient with hypothyroidism is started on levothyroxine (Synthroid), a synthetic thyroid hormone. You know she understands the side effects of this medication when she makes the following statement:

"I know I should call my doctor if my heart races."

Which of the following statements by a patient with newly prescribed contact lenses indicates a need for additional teaching? "I think it is so cool that my best friend, Taylor, and I have the same prescription. She has blue lenses and I have brown lenses, so we'll be able to trade to see what we would look like with different eyes." "If I run out of cleaning and wetting solution, I'll have to wait until I get a new supply before I'll be able to wear my lenses again." "I'll follow the manufacturer's guide to tell me when I need to take them out." "If I have eye redness or pain in my eyes, I'll take out my lenses and call the ophthalmologist first thing."

"I think it is so cool that my best friend, Taylor, and I have the same prescription. She has blue lenses and I have brown lenses, so we'll be able to trade to see what we would look like with different eyes." Rationale: Manufacturer's recommendations indicate to the wearer proper length of time for wear, cleaning, and storing. Patients are instructed to use cleaning and wetting solutions recommended by the optometrist, ophthalmologist, or lens manufacturer. Patients are instructed not to use water or homemade solutions for wetting or cleaning lenses. If eye redness, tearing, vision loss, or pain occurs, the patient should remove the lenses and contact the eye care professional as soon as possible. Lenses should not be shared or "tried on" by another person.

A patient who is preparing for surgery asks the nurse why his physician took him off his oral hypoglycemic and placed him on sliding-scale insulin. Which response by the nurse is best?

"It helps us to maintain better control of your blood glucose during surgery. you will most likely be back on pills before you go home."

The 77-year-old male smoker has developed cataracts on both eyes, and is awaiting surgery. The cataracts have prevented the patient from driving, reading, and seeing television. Which of the following statements would be appropriate for the nurse in the preoperative holding area to communicate to the patient? "Did your eye doctor explain to you that you could choose to use eye drops six times a day to dissolve the cataract?" "You are wise to have both eyes done at the same time." "If you were a non-smoker, you would never have developed cataracts." "Most patients undergoing cataract surgery do well and enjoy the improved vision."

"Most patients undergoing cataract surgery do well and enjoy the improved vision." Rationale: Surgical removal is the only treatment used at this time for cataracts; no medical treatment is available to prevent or treat them. Surgical removal of the cataract and lens is indicated when the cataract has developed to the point that vision and activities of daily living are affected. If the patient presents with bilateral cataracts, surgery is only performed on one eye at a time. Complications from cataract surgery are unusual, and occur in less than 1% of the surgeries.

Small bowel enteroscopy: double-balloon

"Push-and-pull"; balloons alternately inflate/deflate, telescoping the small intestine onto the overtube, providing much more visualization.

A 42-year old woman enters an outpatient clinic with symptoms of weight gain and fatigue. Lab studies are done, and she is diagnosed with primary hypothyroidism. She asks why her thyroid-stimulating hormone (TSH) level is elevated. How should the nurse respond?

"The pituitary makes more TSH to try to stimulate the underactive thyroid."

A patient with SIADH asks the nurse why he has gained 10 lbs. What is the best response for the nurse?

"You have too much of a hormone in your system that causes you to retain water. The extra 10 lbs. is likely water weight." Antidiuretic hormone causes water retention without sodium or potassium retention.

What instructions should the nurse provide to the patient who is being discharged after a thyroidectomy?

"You must take your thyroid replacement everyday just as the physician prescribed." Thyroid hormone is essential to survival. If the thyroid gland has been removed, exogenous hormone replacement is required for life.

hyperopia

(farsightedness) The condition in which the eyeball is short, causing the image to focus behind the retina.

Cataracts - surgery

- intra-ocular lens implant CM: morning discharge, scratchy feeling, blurred vision up to several weeks post-op - potential complications Table 58-5 pg 1774: most common is "after-cataract"

Hypothyroidism symptoms

*Be careful, the thyroid is in front of the parathyroid. *Can be caused by hyperthyroid treatments CARDIOVASCULAR- Bradycardia, decreased cardiac output, cool skin, cold intolerance NEUROLOGIC- Lethargy, slowed movements, memory loss, mental dullness, confusion PULMONARY- Dyspnea, hypoventilation INTUGUMENTARY- Cool, dry skin; brittle, dry hair GASTROINTESTINAL- Decreased appetite, weight gain, constipation, increased serum lipid levels REPRODUCTIVE- Decreased libido, erectile dysfunction. Hypercholesterolemia PRIMARY- Decreased TH, Increased TSH SECONDARY (pituitary cause)- Decreased TH, Decreased TSH *Can cause coma when not treated. *Monitor and record bowel movements.

Hyperthyroidism symptoms

*Results in excessive amounts of circulating thyroid hormone (thyrotoxicosis) *Autoimmune disease that can be caused by Graves disease *more common in young woman *Multimodular goiter is more common in older women. *Eyes wide open (Exophthalmos) caused by Graves Disease CARDIOVASCULAR- Tachycardia, increased cardiac output, warm skin, heat intolerance NEUROLOGIC- Fatigue, restlessness, hyperactive reflexes, tremor, insomnia, emotional stability PULMONARY- Dyspnea INTUGUMENTARY- Diaphoresis, Warm, moist skin; fine, soft hair GASTROINTESTINAL- Increased appetite, weight loss, frequent stools, decreased serum lipid levels REPRODUCTIVE- Decreased libido, erectile dysfunction, amenorrhea. PRIMARY- Increased TH, Decreased TSH SECONDARY (pituitary cause)- Increased TH, Increased TSH Therapeutic measures- Check thyroid hormone levels, antifibroid meds, radioactive iodine, use eyedrops to keep moist, may have to use tape to close eyes to sleep

More Diabetes Goals and recommendations:

*Yearly flu vaccine for all patients 6mos and older *1 lifetime pneumococcal vaccine for patients older than 2, re-vaccination for patients older than 64 if first vaccine was more than 5 years earlier *Aspirin therapy 75-162 mg/day if older than 30 years *Statin therapy if history of CVD or if older than 40 with CVD risk factors *Smoking cessation counseling *Yearly comprehensive foot exam *Dilated comprehensive eye exam within onset of Type 1 diabetes, or at diagnosis of Type 2 diabetes, and annually thereafter

post-op cataract surgery home care

- 1st week: blurry vision, tearing, sandy feeling common (wipe w/ wet sterile cotton swab) - avoid activities that increase IOP - advise soft, easily chewable diet until healing is complete - always wear sunglasses outside - wear eye shield HS for 1-4 weeks - notify MD if: increased redness/discharge, spots/floaters/worsening vision, increased p! (possible infection/RD)

macular degen - manage

- Amsler grid

Legally Blind - definition

- BCVA at 20/200 or worse - visual field diameter < or = to 20 degrees

Management of Thyroid Storm

- Hypothermia mattress or blanket - Humidified oxygen - IV fluids - PTU or methimazole - Hydrocortisone - treat shock/adrenal insufficiency - Iodine131 - decrease output of T4 from the thyroid gland (can cause GI distress)

KNOW procedure of mixing insulins! How would you mix a long acting and short acting insulin?

- Insert air into the long acting first (NPH) - insert air into the short acting (regular) - withdraw short acting (regular) - withdraw long acting (NPH) you always withdraw the shorter acting, or regular first.

Classic S/S of hyponatremia?

- Rapid HR with normal BP - then BP drops to orthostatic hypotension as sodium level decreases, pulse becomes weak and thready - rapid, shallow respirations - increased urine, decreased specific gravity - hyperactive bowel sounds, diarrhea - nausea - headache, confusion, cognitive changes - decreased deep tendon reflexes, skeletal muscle weakness

Surgical management of hyperthyroidism

- Reserved for special circumstances Examples - Pt with allergy to antithyroid meds, large goiters - Subtotal thyroidectomy - Removal of 5/6 of thyroid tissue

PRK Tx

- laser assissted keratectomy - reshapes eye to improve VA - longer recovery w/ more post-op discomfort than LASIK, but less eye dryness - s/e include: eye irritation, eyelid sticking, recurrent erosion during sleep (painful), p!, glare, reduced VA in low light

glaucoma - diagnostic tests

- VA test - tonometry - measures IOP - visual field test - peripheral vision - dilated eye exam - ophthalmic vision of inner eye - pachymetry - measures thickness of cornea

glaucoma - angle closure

- acute process w/ rapid onset, medical emergency, acute angle closure most common

What are the nurse's Pre-op responsibilites for a pt having eye surgery?

- administer antianxiety drugs - local anesthetic injected behind eyeball, in and around eye - sometimes a facial scrub is needed - clipping of eyelashes (sometimes) - pupils are dilated with medication

What do you need to teach to the patient about insulin... insulin education?

- avoid extreme temps: temps below 36 degrees or higher than 86 - refrigerate insulin not in use - inspect for frost, precipitation or change in color - prevent exposure to sunlight - may keep at room temp for up to 28 days label insulin vial when opened - avoid excessive shaking of vial - protect from direct heat

blepharitis

- bacterial inflamm of eyelid - s/x: burning, itching, sensation of foreign object, crusting of lids upon awakening, frothy tears, dry eyes - care: warm compress --> loosed crust and clean lids

Acute conjunctivitis - causes, s/x, Tx

- bacterial/viral infection, allergies, colds, URTI - s/x: eyelid swelling w/ purulent discharge, can affect both eyes - Tx: highly infectious, warm compress to eye 5-10 min TID for viral and bacterial, cool compress to eyes for allergies - Chart 58-12 for PT education

RD nursing implications

- best rest immediately - cover both eyes with patches

What are the s/s of detached retina?

- bright flashes of light -floating dark spots - sensation of a curtain being pulled over visual field - anxiety

astigmatism - definition

- cornea has oblong football shape instead of normal spherical shape - causes mild blurry or distorted vision

LASIK Tx

- laser eye therapy - reshapes eye to improve VA - has quick recovery - s/e include: over/under correction, subconjunct hemorrhage, halos, night-driving issues, keratoconus, eye dryness (most common)

Cataract surgery post-op care

- elevate head of bed 30-45 degrees - position pt on back or non-op side for 1-2 days post-op - monitor for post-op hemorrhage - educate client to avoid activities that can increase IOP - s/x complications: changes in vision, floaters, increased p! and/or redness --> risk retinal detach

Eye removal

- enucleation (most common; removal of complete eyeball w/ eyelid in tact), evisceration, exenteration

RD s/x and diagnostic

- flashes of light when looking to side, floaters, sense of curtain being drawn - comprehensive eye exam, US

Hypoglycemic medical management?

- give fast-acting carbohydrate 10-15 grams such as: 4-6 oz fruit juice, regular soda 6-10 life savers or hard candy 2-3 teaspoons sugar or honey if patient is unconscious, give injection of glucagon 1g IM or SQ = may take 20 min for pt. to regain consciousness. If in hospital, may give 50% dextrose IV to get results in minutes

glaucoma - diagnostic criteria

- high pressure inside eye, optic nerve damage, vision loss - not all three required for Dx, but all three present = slam dunk

glaucoma - definition

- increased IOP due to inadequate drainage of AH of overproduction of AH - causes damage to optic nerve; can lead to blindness - often p!-less and symptom free

glaucoma risk factors: most pertinent

- increasing age, black-Americans, family Hx glaucoma, thin cornea, myopia & HTN

glaucoma - open angle s/x

- p!-less, tired eyes, foggy vision, tunnel vision, halos, headache w/ ocular HTN

low vision - definition

- perceived myopia/hyperopia even w/ aid of glasses/contacts - BCVA 20/70-20/200

glaucoma - Assessment

- progressive loss peripheral vision - elevated IOP (10-21 mmHg WNL) - progressive tunnel vision - frontal headaches due to increased IOP

glaucoma - post-op manage

- protect eye with shield and patch - provide cold compress - elevate head of bed 30 degrees - monitor pt 3-4 hrs post-op for increased IOP - teach client to report eye p!, halos, and vision changes immediately - AVOID: atropine sulfate, nitroglycerine, anticholinergics

Cataracts - assessment findings

- pupils are opaque/cloudy-white or grayish pearly haze - gradual vision loss - p!-less, blurred vision w/ decreased color perception - better vision in dim light w/ pupil dilation - diplopia and photophobia

Keratoplasty

- purpose: corneal transplant - post-op: monitor for >IOP, avoid eye pressure, rest eye, prevent infection

Classic S/S of Dehydration???

- rapid, thready pulse -decreased BP -increased rate and depth of respirations - fever - dry, scaly skin - decreased urine output, constipation - dry mouth - poor skin turgor, tenting of skin - flat neck veins - weakness - weight loss

retinal detach

- rhegmatgenous is most common - tear/hole in the retina that allows fluid w/in the eye to slip though the opening and bet behind the retina, causing detachment - tractional - most often seen in diabetics - medical emergency

RD surgery

- simple RD - photocoagulation, cryopexy, pneumatic retinopexy (remember about gas buble --> must have client lying in prone position in order for bubble to act as tamponade for for retinal break) - severe RD - repair in hospital OR

Classic S/S of hyperkalemia?

- slow irregular heartrate - leading to ectopic beats, changes on EKG - EKG - shows tall t waves - hyperactive bowels, diarrhea - Flaccid paralysis, cramps, weakness - twitching muscles, parasthesias

sty

- tender, p! red bump located at base fo an eyelash or inside eyelid - acute infection of oil glands in eyelid or infected eyelash follicle - care: should not be pressed or squeezed to facilitate drainage

What are rules for when a diabetic is sick?

- test blood glucose every 3-4 hours - continue to take insulin or oral meds - report elevated glucose levels if greater than 300 or urine ketones to doc - to prevent dehydration: drink plenty of liquids if vomiting or have diarrhea - take 1/2 cup regular cola, orange juice, 1/2 cup broth, 1 cup gatorade every 30 minutes to 1 hour

cataracts - implementation: surgery

- uni/bilateral surgery - most common phacoemulsification: microsurgery that uses small US probe - extracapsular cataract extraction (ECCE) - large incision w/ stitches; removal of almost all natural lense - Pre-op: d/c anticoag therapy 5-7 day before surgery

Care of Legally Blind

- use normal voice, alert pt when entering room - orient pt to room - encourage independence - ensure safety (proper lighting, clear floors, call light w/in reach, etc) - assess pt/family readiness to learn before orientation/mobility training

Classic S/S of hypokalemia?

- weak thready pulse - Skeletal muscle weakness, flaccid paralysis - constipation - decreased ability to concentrate urine - EKG changes - tall Us - Nausea, vomiting - Anxiety, lethargy, confusion - Orthostatic hypotension - shallow, weak respirations, diminished breath sounds - #1 assessment priority is respiratory if suspected hypokalemia bc of associated skeletal muscle weakness - therefore will have shallow, weak respirations!

Rules for diabetic foot care:

- wear thick, soft socks with well fitting shoes - never go barefoot - break in new shoes by wearing for short amounts of time - do NOT use heating pads or hot water bottles - stop smoking, reduce alcohol intake - file toenails straight across and do not cut toenails - inspect feet daily - use moisturizer, but do not allow moisture between toes

Myxedema Coma

-A rare but serious disorder that results from persistent low thyroid production - It can be precipitated by acute illness, rapid withdrawal of thyroid medication, anesthesia and surgery, hypothermia, and the use of sedatives and narcotics

Subjective data collection for patient with DM

-Age and symptoms at onset -Understanding of Diabetes (1 & 2) and self care -Current treatment plan (medication, nutrition therapy, exercise) and adherence to plan -Frequency of blood glucose monitoring and pattern of BG levels (check diary) -History of diabetes related complications -Involvement of family or other support systems

Steps for mixing insulins; Remember "clear to cloudy":

-Assemble equipment -Check orders to conform type/dose of regular (clear) and intermediate acting (cloudy) insulins -Roll bottle of cloudy insulin -wipe tops of both bottles with alcohol -Draw up amount of air equal to dose of cloudy insulin 1. Inject air into cloudy vial, -Draw up amount of air equal to dose of clear insulin 2. Inject air into clear vial, 3. draw up clear insulin, have nurse double check 4. then draw up cloudy insulin, have nurse double check *If too much insulin drawn into syringe, discard and start over *Remember, never to mix insulin glargine (Lantus) *If the clear is drawn up last, the vial may be contaminated by the cloudy insulin, altering the action of the clear insulin *If the cloudy insulin is unknowingly contaminated by clear insulin, the clear will become cloudy and its effect will be diminished

Diabetes Goals and recommendations:

-Capillary plasma glucose should be measured at least three times a day for patients using multiple insulin injections -BP measured every office visit -Serum creatnine and Urine microalbumin should be assessed yearly Target levels are: -HbA1c s/b <7% -preprandial capillary glucose s/b 70-130 mg/dL -Peak postprandial capillary glucose s/b < 180 mg/dL **(Blood glucose can be different in hospitalized patients) -BP s/b <130/80 mm Hg -LDLs s/b <100 mg/dL or < 70 mg/dL inpatients with cardiovascular disease -Triglycerides s/b <150 mg/dL -HDLs s/b >40 mg/dL in men >50 mg/dL in women

Classic S/S of hypernatremia?

-Decreased cardiac output and contractility - pulmonary edema - twitching, muscle weakness - Skin is dry, flaky - Stupor, coma, lethargy, confusion, agitation - Seizures, manic episodes - Decreased urine output, increased specific gravity

Thyroid Storm

-Form of severe hyperthyroidism -Almost always fatal without treatment -Critically ill and requires astute observation and aggressive care

Hyperthyroidism

-Second most common endocrine disorder after DM -Graves' disease: most common type - Results from an excessive output of thyroid hormones caused by abnormal stimulation of the thyroid gland -Also known as thyrotoxicosis

Objective data collection for patient with DM

-Vital signs -Height, Weight, Body Mass Index -Skin, Integrity Turgor, condition of injection sites -Feet; pulses, color, temp, skin integrity, pressure points, sensation -Lab results: blood glucose, HbA1c, creatnine, lipid profile, albuminuria, urine and serum ketones

What are the s/s of glaucoma?

-cloudy, blurry vision -loss of peripheral vision - pain, headache -brow ache nausea/vomiting -"the silent thief" 5 Danger signs: 1)brow arching 2)halos 3) blurry vision 4)diminished peripheral vision 5) brow pain

What are the s/s of cataracts?

-gradual, painless blurred vision -double vision with decreased color perception - no pain or eye redness is usually associated with age related cataracts

What is the treatment for Meniere's?

-low sodium, low sugar diet; keep hydrated -avoid caffiene, nicotine, alcohol -take anit-emetics, diuretics (diamox), antihistamines, anti-anxiety drugs, anti-vertigo drugs -bed rest during acute phase -Surgical: last resort is the labryinthectomy, can cause bell's palsy

Classic S/S of Overyhydration?

-rapid, bounding pulse with full peripheral pulses - elevated BP - rapid, shallow respirations with dsypnea on exertion - Moist crackles - pitting edema, skin cool and pale - altered LOC, headache - visual disturbances, weakness - paresthesias - increased GI motility - distended neck and hand veins - weight gain

Glucophage

...

Glypizide

...

Insulin types

...

65. A nurse is reviewing a client's laboratory report and notes that the serum calcium level is 4.0 mg/dL. The nurse understands that which condition most likely caused this serum calcium level? A. Prolonged bed rest B. Renal insufficiency C. Hyperparathyroidism D. Excessive ingestion of vitamin D

... A. Prolonged bed rest

47. The client has received IV solutions for three (3) days through a 20-gauge IV catheter placed in the left cephalic vein. On morning rounds the nurse notes the IV site is tender to palpation and a red streak has formed. Which action should the nurse implement first? A. Start a new IV in the right hand. B. Discontinue the intravenous line. C. Complete an incident record. D. Place a warm washrag over the site.

... B. Discontinue the intravenous line Explanation A. (incorrect) A new IV will be started in the right hand after the IV is discontinued. B. (correct) The client has signs of phlebitis and the IV must be removed to prevent further complications. C. (incorrect) Depending on the health-care facility, this may or may not be done, but client care comes before documentation. D. (incorrect) A warm washrag placed on an IV site sometimes provides comfort to the client. If this is done, it should be done for 20 minutes four (4) times a day.

29. A client who is post-gallbladder surgery has a nasogastric tube, decreased reflexes, pulse of 110 weak and irregular, and blood pressure of 80/50 and is weak, mildly confused, and has a serum of potassium of 3.0 mEq/L. Based on the assessment data, which of the following is the priority intervention? A. Withhold furosemide (Lasix) B. Notify the physician C. Administer the prescribed potassium supplement D. Instruct the client on foods high in potassium

... B. Notify the physician Explanation The priority intervention for a client who had gallbladder surgery, has a nasogastric tube, decreased reflexes, pulse of 110 weak and irregular, and blood pressure of 80/50 and is weak, mildly confused, and has a serum potassium of 3.0 mEq/L would be to notify the physician that the potassium level is low. After notifying the physician, the furosemide (Lasix) may be withheld and potassium supplement should be administered as prescribed and may even be increased after talking with the physician. The client may also be instructed on foods high in potassium. These are all appropriate interventions but not the priority.

44. The client post-thyroidectomy complains of numbness and tingling around the mouth and the tips of the fingers. Which intervention should be implemented first? A. Notify the health care provider immediately. B. Tap the cheek about two (2) centimeters anterior to the ear lobe. C. Check the serum calcium and magnesium levels. D. Prepare to administer calcium gluconate IVP.

... B. Tap the cheek about two (2) centimeters anterior to the ear lobe A. (incorrect) The health care provider may need to be notified, but the nurse should perform assessment first. B. (correct) These are signs and symptoms of hypocalcemia, and the nurse can confirm this by tapping the cheek to elicit the Chvostek's sign. If the muscles of the cheek begin to twitch, then the health care provider should be notified immediately because hypocalcemia is a medical emergency. C. (incorrect) A positive Chvostek's sign can indicate a low calcium or magnesium level, but serum lab levels may have been drawn hours previously or may not be available. D. (incorrect) If the client does have hypocalcemia, this may be ordered, but it is not implemented prior to assessment. TEST-TAKING HINT: Assessment is the first step in the nursing process and is an appropriate option to select if the test taker has difficulty when trying to decide between two options.

49. The client has been vomiting and has had numerous episodes of diarrhea. Which laboratory test should the nurse monitor? A. Serum calcium. B. Serum phosphorus. C. Serum potassium. D. Serum sodium.

... C. Serum potassium

63. A nurse is caring for a client with a nasogastric tube. Nasogastric tube irrigations are prescribed to be performed once every shift. The client's serum electrolyte results indicate a potassium level of 4.5 mEq/L and a sodium level of 132 mEq/L. Based on these laboratory findings, the nurse selects which solution to use for the nasogastric tube irrigation? A. Tap water B. Sterile water C. Sodium chloride D. Distilled water

... C. Sodium chloride

45. Which statement best explains the scientific rationale for Kussmaul's respirations in the client diagnosed with diabetic ketoacidosis (DKA)? A. The kidneys produce excess urine and the lungs try to compensate. B. The respirations increase the amount of carbon dioxide in the bloodstream. C. The lungs speed up to release carbon dioxide and increase the pH. D. The shallow and slow respirations will increase the HCO3 in the serum.

... C. The lungs speed up to release carbon dioxide and increase the pH.

19. A client with COPD feels short of breath after walking to the bathroom on 2 liters of oxygen nasal cannula. The morning's ABGs were pH of 7.36, PaCO2 of 62, HCO3 of 35 mEq/L, O2 at 88% on 2 liters. Which of the following should be the nurse's first intervention? A. Call the physician and report the change in client's condition B. Turn the client's O2 up to 4 liters nasal cannula C. Encourage the client to sit down and to take deep breaths D. Encourage the client to rest and to use pursed-lip breathing technique

... D. Encourage the client to rest and to use pursed-lip breathing technique Explanation Clients with COPD, especially those who are in a chronic compensated respiratory acidosis, are very sensitive to changes in O2 flow, because hypoxemia rather than high CO2 levels stimulates respirations. Deep breaths are not helpful, because clients with COPD have difficulty with air trapping in alveoli. There is no need to call the physician, since this client is presently most likely at baseline.

64. A nurse is reviewing laboratory results and notes that a client's serum sodium level is 150 mEq/L. The nurse reports the serum sodium level to the physician and the physician prescribes dietary instructions based on the sodium level. Which food item does the nurse instruct the client to avoid? A. Peas B. Cauliflower C. Low-fat yogurt D. Processed oat cereals

... D. Processed oat cereals

51. A nurse is assigned to care for a group of clients. On review of the clients' medical records, the nurse determines that which client is at risk for deficient fluid volume? A. A client with a colostomy B. A client with congestive heart failure C. A client with decreased kidney function D. A client receiving frequent wound irrigations

...A. A client with a colostomy

24. Which of the following assessment findings would indicate to the nurse that a client's diabetic ketoacidosis is deteriorating? A. Deep tendon reflexes decreasing from +2 to +1 B. Bicarbonate rising from 20 mEq/L to 22 mEq/L C. Urine pH less than 6 D. Serum potassium decreasing from 6.0 mEq/L to 4.5 mEq/L

...A. Deep tendon reflexes decreasing from +2 to +1 Explanation A decrease in deep tendon reflexes is a sign that pH is dropping and that metabolic acidosis is worsening to diabetic ketoacidosis. An increase in bicarbonate would indicate that the acidosis is being corrected. A urine pH less than 6 indicates the kidneys are excreting acid. Serum potassium levels are expected to fall because acidosis is corrected and potassium moves back into the intracellular space.

66. A nurse is assessing a client with a suspected diagnosis of hypocalcemia. Which of the following clinical manifestations would the nurse expect to note in the client? A. Twitching B. Negative Trousseau's sign C. Hypoactive bowel sounds D. Hypoactive deep tendon reflexes

...A. Twitching

62. A nurse is caring for a client with acute congestive heart failure who is receiving high doses of a diuretic. On assessment, the nurse notes that the client has flat neck veins, generalized muscle weakness, and diminished deep tendon reflexes. The nurse suspects hyponatremia. What additional signs would the nurse expect to note in this client if hyponatremia were present? A. Dry skin B. Decreased urinary output C. Hyperactive bowel sounds D. Increased specific gravity of the urine

...C. Hyperactive bowel sounds

67. A nurse caring for a client with hypocalcemia would expect to note which of the following changes on the electrocardiogram? A. Widened T wave B. Prominent U wave C. Prolonged QT interval D. Shortened ST segment

...C. Prolonged QT interval

42. The client is admitted with a serum sodium level of 110 mEq/L. Which nursing intervention should be implemented? A. Encourage fluids orally. B. Administer 10% saline solution IVPB. C. Administer antidiuretic hormone intranasally. D. Place on seizure precautions.

...D. Place on seizure precautions

Common Preoperative Medications:

1) Benzodiazepines and barbiturates: for sedation and amnesia 2) Anticholinergics: to reduce secretions 3) Opioids: to decrease intraoperative anesthetic requirements and pain 4) Additional drugs include antiemetics, antibiotics, eye drops, and regular prescription drugs

How do you "test positive" for diabetes?

1) Fasting plasma glucose test: must have a blood glucose level of greater than 126 mg/dl or more on 2 occasions. Fasting means that you did not eat for 8 hours prior. 2) Random plasma glucose: tested at any time even after eating; had a level of greater than 200 on more than one occasion 3) 2-hr plasma glucose test: using a 75 gram glucose load, the patient is tested at hour intervals - a normal person's blood sugar would go up initially, and then go down. A diabetic's would go up and stay up. to test positive, you must have greater than 200 after 2 hours on more than one occasion

What is "metabolic syndrome"? What factors must be present?

1) abdominal obesity: male must have a waist of over 40 inches, a female with a waist over 35 inches 2) hyperglycemiaa present with an elevated glucose over 100 mg/dl 3) Hypertensive with a BP over 130/85 4) Dyslipidema with a triglyceride level over 150 mg/dl

Monitoring glucose:if on oral antidiabetic drugs, must check blood sugar how many times a day?

1-2 times a day

A patient with hyperthyroidism has been given an oral dose of radioactive iodine and is being released. The following should be included in discharge teaching.

1. Advise the patient to avoid close contact with family members for about 1 week. 2. Encourage the patient to drink plenty of fluids. 3. Inform the patient of the symptoms of hypothyroidism to report. 4. Tell the patient to be vigilent about careful handwashing after urinating.

What are some conditions outside of having diabetes that could raise a person't blood glucose that a nurse should be aware of?

1. Any disease of the pancreas will cause hyperglycemia because the pancreas will not be functioning normally with proper hormonal control of blood sugar levels 2. Other diseases of the endocrine system including: Cushing's disease, Acromegaly, Hyperthydroidism 3. Any body infection including Congenital rubella, cyto megalovirus 4. Certain prescribed drugs and therapies: being on corticosteroids, taking artificial thyroid hormones, dilantin,, using Nicotinic acid 5. having diseases like Down Syndrome or Huntington Disease 5. Being on IVs with dextrose: being on total parental nutrition.

Mr. Lloyd is admitted to your unit for severe end-stage respiratory disease. He has been on medication for many years to control his airway inflammation and now has Cushing's syndrome

1. Cushing's syndrome is caused by exposure to excess cortisol. This can occur because of an adrenal problem, a pituitary problem, or from treatment with exogenous corticosteroids. 2. Hyperglycemia, moon face, thin skin, buffalo hump, truncal obesity, sodium and water retention and osteoporosis are common signs of Cushing's disease. 3. Mr. LLoyd should reposition every two hours & use a pressure-reducing mattress can help prevent pressure ulcers. Using a lift sheet prevents skin injury from friction and shear. Moisturizing cream prevents dry skin. 4. Glucocorticoids suppress immune system so Mr. LLoyd should be taught to avoid people who are ill and to use good handwashing techniques. 5. Mr. Lloyd is placed on high-dose IV glucocorticoids to relieve dyspnea and his blood glucose skyrockets. Insulin would be given to treat the hyperglycemia; (oral hypoglycemics typically are ineffective for steroid induced diabetes.

What should be included in the teaching for a 40 yr old with type 2-diabetes mellitus to prevent long term complications?

1. Have regular eye examinations 2. Maintain good oral hygiene and make regular visits to the dentist 3. Wash, dry and inspect feet daily 4. Take daily aspirin as prescribed 5. See a podiatrist or other health care professional to monitor neuropathy

Correct sequential order for mixing insulin

1. Roll cloudy vial 2. Clean vial tops with alcohol 3. Inject air into cloudy insulin 4. Inject air into clear insulin 5. Draw up clear insulin 6. Draw up cloudy insulin

A young woman makes an appointment to see a physician at the clinic. She complains of tiredness, weight gain, muscle aches and pains, and constipation. The physician will likely order: 1. T3 and T4 serum level laboratory tests. 2. glucose tolerance test. 3. cerebral computed tomography (CT) scan. 4. adrenocortical stimulating test.

1. T3 and T4 serum level laboratory tests. These complaints are strongly suggestive of thyroid disorder; T3 and T4 laboratory tests are the most useful diagnostic tests.

The nurse needs to accomplish all the following interventions for a patient who is 24-hours post-thyroidectomy in this order:

1. Verify the airway is patent 2. Assess vital signs 3. Check the surgical site dressing for signs of bleeding 4. Administer and analgesic for post-operative pain. 5. Assist with range of motion exercises 6. Teach the patient about synthroid (levothyroxine) use after discharge

What are some common Nursing Interventions to reduce risk/avoid post-op complications?

1. Turn & reposition the pt to promote circulation and reduce the risk of skin breakdown, especially over boney prominences. Initially position pt in a Lateral recumbant position until arousal from anesthesia, then position pt in Semi or Fowler position to reduce breathing effort. 2. Encourage coughing & deep breathing. This helps clear anestetics from the body, lowers risk of pulmonary/fat emboli, and hypostatic pneumonia associated with secretion buildup in the airways. 3. Encourage use of Incentive Spirometer. 4. Monitor In's and Out's. Hydration and protein rich nutrition promotes healing and provides energy to meet the needs of the pts increased metabolism associated with surgery. 5. Promote early ambulation. Early post-op exercise and ambulation significantly reduce the risk of thromboembolism.

Mrs. Brown is a 52-year-old cook in a fast-food restaurant. She is 5'2" tall and weighs 190lbs. She says she is "shaped like a pumpkin" because her arms and legs are relatively thin compared to her middle. Because she tends to eat her meals at work, she has a high-fat, high-sodium diet that is low in fresh fruits and vegetables. She drinks diet soda all day long and spends long periods on her feet. Mrs Brown's sister has DM, but Mrs. Brown's blood glucose levels have always been normal whenever she has been tested.

1. What factors place Mrs. Brown at risk for diabetes? Heredity Obesity Body shape Age 2. What actions can Mrs. Brown take to reduce her risk of developing diabetes? Lose weight Exercise regularly 3. Mrs. Brown tries to lose weight butis unsuccessful and, at age 57, is diagnosed with type 2 diabetes. Diabetes is diagnosed when the fasting plasma glucose is __________ mg/dL or greater? 126 mg/dL or greater for fasting plasma 200 mg/dL or greater for casual plasma with symptoms of diabetes are diagnostic for diabetes. 4. Mrs. brown develops a small blister on he bottom of her right heel. Which actions are important for her to take to make sure her heel heals? Gently wash, dry and inspect the area daily Use crutches to avoid bearing weight on the blister See podiatrist if it is not healed in 2-3 days *Wear shoes and socks for support *Never open blisters/wounds 5. Mrs. Brown is very disciplined. her foot heals, and she loses 50 lbs. Her physician tells her to stop her metformin (Glucophage) and return to the office for follow-up in 3 months. What blood test can the physician order that will reflect her blood glucose control over 3 months? Glycohemoglobin or HbA1c test reflects 2-3 months. Normal is 45-65. The American Diabetic Association recommends less than 7% as a goal for people with diabetes.

Diabetes is diagnosed when the fasting plasma glucose drawn 8 hours without eating is?

126 mg/dL or higher

Qualification for oral antidiabetic drugs

1: type-2 2: over 40 3: diagnosed less than 5 years 4: fasting glucose less than or equal to 200 5: req. less than 40 units of insulin per day 6: normal kidney/liver function

Diabenese (chlorpropramide)

1st Gen. Sulfonylurea antidiabetic

Orinase (tolbutamide)

1st Gen. Sulfonylurea antidiabetic

Tolinase (tolazamide)

1st Gen. Sulfonylurea antidiabetic

Glucotrol (glipizide)

2nd Gen. Sulfonyurea antidiabetic

Monitoring glucose: if on insulin must check blood sugar how many times a day?

4 times a day, before meals and at bedtime

The nurse instructs a patient about how insulin affects blood glucose. Arrange the events in sequence. 1. Beta cells are stimulated to release insulin. 2. Glucose enters the bloodstream. 3. Glycogen is converted to glucose by alpha cells (glycogenesis). 4. Glycogen is stored in the liver. 5. Insulin transports glucose to muscle cells.

2 1 5 4 3

At what point after injection does the peak action of regular insulin occur?

2-5 hours

A patient has come to the doctor's office after finding out that her blood glucose level was 135 mg/dL. She states that she had not eaten before the test and was told to come and see her doctor. She asks you if she has diabetes. The nurse responds: 1. "Having a fasting serum glucose that high certainly indicates diabetes." 2. "That test indicates that we need to do more tests that are specific for diabetes." 3. "How do you feel? Do you have any other signs of diabetes?" 4. "Do you have a family history of diabetes, stroke, or heart disease? We need to know before making a diagnosis."

2. "That test indicates that we need to do more tests that are specific for diabetes The nurse needs to answer the patient's question in a way that gives information and is not misleading. Although 135 is high, there may be a nonpathologic explanation. More tests should be done to evaluate the patient.

A patient has come into the emergency room with her friend. Her friend states that she had been acting very strangely and confused. The friend states that the patient has diabetes and takes insulin. The nurse knows that signs and symptoms of hypoglycemia include: 1. slow pulse rate and low blood pressure. 2. irritability, anxiety, confusion, and dizziness. 3. flushing, anger, and forgetfulness. 4. sleepiness, edema, and sluggishness.

2. irritability, anxiety, confusion, and dizziness When blood sugar levels fall, hormones are activated to increase serum glucose. One of the hormones is epinephrine, which causes these symptoms.

The patient has been admitted with hyperglycemic hyperosmolar nonketotic syndrome (HHNKS). Her blood glucose level is very high (880 mg/dL on admission). The physician believes that her condition is to the result of large amounts of glucose solutions administered intravenously during kidney dialysis. The nurse would anticipate that the patient would exhibit: 1. a fruity breath and high level of ketones in her urine. 2. severe dehydration and hypernatremia caused by the hyperglycemia. 3. exactly the same symptoms and signs as diabetic ketoacidosis. 4. Kussmaul's respirations, nausea, and vomiting.

2. severe dehydration and hypernatremia caused by the hyperglycemia. IV solutions containing glucose will bypass the digestive system, so there is no trigger for the pancreas to release insulin, but there is just enough insulin to prevent the breakdown of fatty acids and the formation of ketones.

when should regular insulin be given?

20-30 minutes before a meal

high fasting blood glucose

200

A patient reports he is having one of his "attacks" of rapid heart rate, palpitations, diaphoresis, tremors and severe pounding headache. Assessment reveals BP 160/120mm Hg and heart rate of 116 beats per minute, other vitals are within normal limits. Preliminary blood tests indicate elevated blood glucose. Which other lab test should the nurse expect to be ordered to help diagnose the patient?

24-hour urine test for metanephrine and vanillylmandellic acid.

Diabeta (glyburide)

2nd Gen. Sulfonyurea antidiabetic

The patient, newly diagnosed with hypothyroidism, seems very anxious to begin her drug regimen. The nurse's instructions include: 1. "Be certain that no dose is skipped." 2. "If a dose is skipped one day, double the dose the next day." 3. "Know the signs and symptoms of hyperthyroidism." 4. "You will be able to notice the benefits of thyroid replacement therapy right away."

3. "Know the signs and symptoms of hyperthyroidism." Her enthusiasm may lead her to overdose on the thyroid replacement pills. She needs to be aware of the proper prescription and the reasons for following the prescribed dosage.

The patient asks about his lab test, which showed a high level of TSH and a low level of T4. You explain: 1. "It means that you have an inconsistency in your thyroid tests, and you will need more testing." 2. "I am sorry. You will have to ask your doctor about your lab results. We are not allowed to discuss them." 3. "The TSH is sending a message to your thyroid gland to increase production, but your thyroid isn't doing that." 4. "That means that you will have to go on hormone therapy for the rest of your life."

3. "The TSH is sending a message to your thyroid gland to increase production, but your thyroid isn't doing that." The test determines if the problem is in the pituitary or in the thyroid. In this case the high TSH is coming from the pituitary as it should but the thyroid is not responding.

The nurse instructs the patient is scheduled to have a radioactive iodine uptake test to: 1. watch for any signs of bleeding or swelling from the biopsy site. 2. avoid contact with others until notified otherwise. 3. wash hands with soap and water after each urination for 24 hours after the test. 4. this test demonstrates the effectiveness of the pituitary gland on the thyroid gland.

3. wash hands with soap and water after each urination for 24 hours after the test. Radiation dose is small and will not harm others.

Amaryl (glimipride)

3rd Gen. Sulfonyurea antidiabetic

When the Type 1 diabetic patient asks why his 7 AM insulin has been changed from NPH insulin to 70/30 premixed insulin, the nurse explains that 70/30 insulin: 1. is absorbed more rapidly into the bloodstream. 2. has no peak action time and lasts all day. 3. makes insulin administration easier and safer. 4. give a bolus of rapid-acting insulin to prevent hyperglycemia after breakfast. the morning meal.

4. give a bolus of rapid-acting insulin to prevent hyperglycemia after breakfast. 70/30 insulin is 30% rapid-acting and 70% intermediate-acting insulin. The rapid action of the 7 AM premixed insulin prevents hyperglycemia after the morning meal.

The type 1 diabetic patient has an insulin order for NPH insulin, 35 U, to be given at 7 AM. The patient is also NPO for laboratory work that will not be drawn until 10 AM. The nurse should: 1. give the insulin as ordered. 2. give the insulin with a small snack. 3. inform the charge nurse. 4. hold the insulin until after the blood draw.

4. hold the insulin until after the blood Holding the insulin for the NPO order is appropriate. The patient will not be getting food until after the blood draw, so will not need the insulin until then. Giving the insulin as ordered will create a possibility of hypoglycemia before the blood is drawn. Giving a snack to a patient who is NPO is inappropriate.

The nurse is drawing up a teaching plan for a patient who has type 1 diabetes. The doctor has ordered two types of insulin, 10 U of regular insulin and 35 U of NPH insulin. The proper procedure is to: 1. draw up the insulins in two separate syringes so that there can be no confusion. 2. draw up the regular insulin before drawing up the NPH insulin. 3. inject air into the NPH insulin, draw it up to 35 U, then inject air into the clear regular insulin and withdraw to 45 U. 4. inject 35 U air into the NPH insulin, inject 10 U air into the regular insulin, withdraw 10 U of the regular insulin, and withdraw 35 U of the NPH insulin.

4. inject 35 U air into the NPH insulin, inject 10 U air into the regular insulin, withdraw 10 U of the regular insulin, and withdraw 35 U of the NPH insulin. When drawing up two insulins, the vials are injected with air and the regular insulin is drawn first. This slow and time-consuming activity has been greatly reduced with the advent of premixed insulins.

Believed to be caused by an auto immune response, what percentage of diabetic patients have Type 1 diabetes?

5% - 10%

The nurse working on the endocrine unit is reviewing preprandial glucose results. Which results indicate the patients DM is well controlled?

51yr old female with glucose = 115 mg/dL 29yr old male with glucose = 84 mg/dL

While providing discharge instructions to a patient newly taking NPH insulin every morning, the nurse recognizes that teaching has been effective if the patient knows to observe for signs and symptoms of low blood sugar level at which of the following times?

6-12 hours after administration of insulin

Monitoring glucose: if on an insulin pump, how many times should you check a day?

6-8 times a day

Average fasting Blood glucose

60-100

What is an acceptable pre-meal blood sugar range for a patient with diabetes?

70-130 mg/dL

Otosclerosis

Abnormal bone formation in the osseous labyrinth of the temporal bone causing the footplate of the stapes to become fixed or immobile in the oval window. The result is a conductive hearing loss.

Morphine sulfate

????????

pregnancy category synthroid

A

Which of these characteristics should a nurse associate with a patient who has type 2 diabetes? (Select all that apply.) A) Exercise and diet may be sufficient treatment B) Is often obese with difficulty managing weight C) Prone to ketosis and ketoacidosis complications D) Genetics and strong familial links are causal factors E) Insulin resistance and inappropriate secretion

A B D E

When assessing the patient experiencing the onset of symptoms of type 1 diabetes, which question should the nurse ask? a. "Have you lost any weight lately?" b. "Do you crave fluids containing sugar?" c. "How long have you felt anorexic?" d. "Is your urine unusually dark-colored?"

A Rationale: Weight loss occurs because the body is no longer able to absorb glucose and starts to break down protein and fat for energy. The patient is thirsty but does not necessarily crave sugar- containing fluids. Increased appetite is a classic symptom of type 1 diabetes. With the classic symptom of polyuria, urine will be very dilute.

hyphema

Bleeding into the anterior chamber of the eye possibly the result of blunt eye trauma.

Dopamine Antagonist - Antiemetic

Blocks D2 receptors in CTZ

Onset, Peak and Duration of BASAL Insulins

EXAMPLE- Insulin glargine BRAND NAME- Lantus AE ONSET- 1-2 hrs PEAK- No Peak DURATION- Up to 24 hrs

Which nursing diagnosis is most appropriate for the patient admitted in addisonian crisis?

Addison's is associated with fluid loss so the nursing diagnosis would be Deficient fluid volume.

life-threatening emergency caused by insufficient adrenocortical hormones or sudden sharp decrease in these hormones

Addisonian crisis

Side Effects of Emollients (Stool Softeners)

Addominal Cramps, Nausea, vomiting, diarrhea

Gastric juice analysis

Aids in diagnosis of Zollinger-Ellison syndrome and atrophic gastritis. pH is measured, volume may also be.

Examples of Proton Pump Inhibitor or PPi's - Anti Ulcer

Esomeprazole, Lansoprazole, Omeprazole, Pantoprazole, Rebeprazole

most common cause of SIADH

malignancy, small cell lung cancer; head injury, stroke, brain tumors,

Vermox (mabendazole)

Anthelmintic agent

myopia

A condition in which the eyeball is elongated, causing the image to focus in front of the retina instead of on it. (nearsightedness)

astigmatism

A condition that develops with abnormal curvature of the cornea or eyeball, causing the image to focus at multiple points on the retina.

Hypothyroidism

A disorder in which the thyroid fails to produce sufficient thyroid hormone which causes a decrease in metabolism

Sulfasalazine (azulfidine)

Anti-inflammatory agent (DMARD)

Motility study: Radionuclide testing

A meal is tagged with radionucleotide markers and eaten. A scintiscanner measures the rate of passage.

Retinopathy

A microvascular disease caused by complication of DM

Amoxil (amoxicillin)

Antibiotic

Euthyroid state

A normal functioning thyroid gland.

spiriva

Anticholinergic Class: beta adrenergic. Generic: tiotropium. Dosage: 1 capsule/d. Route: inhale. Action: bronchodilates.

A patient on an American Diabetes Association diet receives a breakfast tray and does not care for the oatmeal. What is a food can the nurse substitute for a 1/2 cup of oatmeal?

A slice of wheat toast

A teaching plan for a patient who is taking lispro (Humalog) should include which of these instructions by the nurse? A) "Inject this insulin with your first bite of food because it is very fast acting." B) "The duration of action for this insulin is about 8 to 10 hours, so you'll need a snack." C) "This insulin needs to be mixed with regular insulin to enhance the effects." D) "To achieve tight glycemic control, this is the only type of insulin you'll need."

A) "Inject this insulin with your first bite of food because it is very fast acting." Lispro is a rapid-acting insulin and has an onset of 15 to 30 minutes with a peak action of about 2 hours, not 8 to 10 hours. Because of its rapid onset, it is administered immediately before a meal or with meals to control blood glucose rise after meals. Lispro insulin must be combined with intermediate- or long-acting insulin not regular insulin, which is also a short-duration insulin, for glucose control between meals and at night. To achieve tight glycemic control, patients must combine different types of insulin based on duration of action.

Nursing interventions for Activity intolerance related to fatigue secondary to hypothyroidism

Assist with activities of daily living (ADLs), allow rest between activities, protect skin from breakdown related to immobility

At 5 PM a patient who is taking NPH insulin develops hunger, shakiness, and sweating. A nurse assesses the medication administration record (MAR) and should recognize that the patient's symptoms are related to an injection of NPH insulin at which of these times? A) 2 AM B) 8 AM C) 1 PM D) 3 PM

A) 2 AM The patient is exhibiting symptoms of hypoglycemia at 5 PM. NPH has a peak action of 8 to 10 hours after administration. Based on the duration of action of NPH insulin, the patient's hypoglycemic symptoms are from the 8 AM injection of NPH insulin. An injection of NPH insulin at 2 AM, 1 PM, or 3 PM would not cause hypoglycemic symptoms based on the average duration of action from NPH insulin.

A patient is scheduled to start taking insulin glargine (Lantus). On the care plan a nurse should include which of these outcomes related to the therapeutic effects of the medication? A) Blood glucose control for 24 hours B) Mealtime coverage of blood glucose C) Less frequent blood glucose monitoring D) Peak effect achieved in 2 to 4 hours

A) Blood glucose control for 24 hours Insulin glargine is administered as a once-daily subcutaneous injection for patients who have type 1 diabetes. It is used for basal insulin coverage, not mealtime coverage. It has a prolonged duration up to 24 hours with no peaks. Blood glucose monitoring is still an essential component to achieve tight glycemic control.

The nurse receives a lab report indicating that the phenytoin (Dilantin) level for the patient she saw in the clinic yesterday is 16 mcg/mL. Which intervention is most appropriate? A) Continue as planned since the level is within normal limits. B) Tell the patient to hold today's dose and return to the clinic. C) Consult the prescriber to recommend an increased dose. D) Have the patient call 911 and meet the patient in the emergency department.

A) Continue as planned since the level is within normal limits

Which of these instructions should the nurse provide when teaching a patient to mix regular insulin and NPH insulin in the same syringe? A) Draw up the clear regular insulin first, followed by the cloudy NPH insulin. B) It is not necessary to rotate the NPH insulin vial when it is mixed with regular insulin. C) The order of drawing up insulin does not matter as long as the insulin is refrigerated. D) Rotate each day subcutaneous injection sites among the arm, thigh, and abdomen.

A) Draw up the clear regular insulin first, followed by the cloudy NPH insulin. To ensure a consistent response, only NPH insulin is appropriate for mixing with a short-acting insulin. Unopened vials of insulin should be refrigerated; current vials can be kept at room temperature for up to 1 month. Drawing up the regular insulin into the syringe first prevents accidental mixture of NPH insulin into the vial of regular insulin, which could alter the pharmacokinetics of subsequent doses taken out of the regular insulin vial. NPH insulin is a cloudy solution, and it should always be gently rotated to evenly disperse the particles before loading the syringe. Subcutaneous injections should be made using one region of the body (e.g., the abdomen or thigh) and rotated within that region for 1 month.

The telemetry unit nurse is reviewing laboratory results for a patient who is scheduled for an operative procedure later in the day. The nurse notes on the laboratory report that the patient has a serum potassium level of 6.5 mEq/L, indicative of hyperkalemia. The nurse informs the physician of this laboratory result because the nurse recognizes hyperkalemia increases the patient's operative risk for: A. Cardiac problems B. Bleeding with anemia C. Fluid imbalances D. Infection

A) Hyper/hypokalemia increases the patient's risk for cardiac problems. A decrease in the hematocrit and hemoglobin level may indicate the presence of anemia or bleeding. An elevated WBC occurs in the presence of infection. Abnormal urine constituents may indicate infection or fluid imbalances.

Which agent below is most likely to cause serious respiratory depression as a potential adverse reaction? A) Morphine (Duramorph) B) Pentazocine (Talwin) C) Hydrocodone (Lortab) D) Nalmefene (Revex)

A) Morphine (Duramorph) Morphine is a strong opioid agonist and as such has the highest likelihood of respiratory depression. Pentazocine, a partial agonist, and hydrocodone, a moderate to strong agonist, may cause respiratory depression but not as often and serious as morphine. Nalmefene is an opioid antagonist and would be used to reverse respiratory depression with opioids.

A postoperative patient has an epidural infusion of morphine sulfate (Astramorph). The patient's respiratory rate declines to 8 breaths/min. Which medication would the nurse anticipate administering? A) Naloxone (Narcan) B) Acetylcysteine (Mucomyst) C) Methylprednisolone (Solu-Medrol) D) Protamine sulfate

A) Naloxone (Narcan) Naloxone is a narcotic antagonist that can reverse the effects, both adverse and therapeutic, of opioid narcotic analgesics.

Upon assessment, a patient reports that he drinks 5-6 bottles of beer every evening after work. Based upon this information, the nurse is aware that the patient may require: A. Larger doses of anesthetic agents and larger doses of postoperative analgesics. B. Larger doses of anesthetic agents and lower doses of postoperative analgesics. C. Lower doses of anesthetic agents and larger doses of postoperative analgesics. D. Lower doses of anesthetics agents and lower doses of postoperative analgesics.

A) Patients with a larger habitual intake of alcohol require larger doses of anesthetic agents and postoperative analgesics, increasing the risk for drug-related complications.

A patient who has type 2 diabetes has a glycated hemoglobin (HbA1c) result of 10%. A nurse should make which of these changes to the nursing care plan? A) Refer to a diabetic educator, there is poor glycemic control. B) Glycemic control is adequate, no changes are needed. C) Hypoglycemia is a risk, teach the patient the symptoms. D) Instruct the patient to limit activity and weekly exercise.

A) Refer to a diabetic educator, there is poor glycemic control.

A client has been newly diagnosed with hypothyroidism and will take levothyroxine (Synthroid) 50 mcg/day by mouth. As part of the teaching plan, the nurse emphasizes that this medication: A) Should be taken in the morning B) May decrease the client's energy level C) Must be stored in a dark container D) Will decrease the client's heart rate

A) Should be taken in the morning

Serotonin Receptor Antagonists - Antiemetic

Blocks serotonin receptors in CTZ and vagal nerve terminals in upper GI tract Used in cancer chemo - Golden Child

The nurse is providing teaching to a patient regarding pain control after surgery. The nurse informs the patient that the best time to request pain medication is: A. Before the pain becomes severe. B. When the patient experiences a pain rating of 10 on a 1-to-10 pain scale. C. After the pain becomes severe and relaxation techniques have failed. D. When there is no pain, but it is time for the medication to be administered.

A) The question states that the patient is being instructed on when to "request" pain medication. If a pain medication is ordered PRN, the patient should be instructed to ask for the medication before the pain becomes severe.

The nurse is preparing to send a patient to the operating room for an exploratory laparoscopy. The nurse recognizes that there is no informed consent for the procedure on the patient's chart. The nurse informs the physician who is performing the procedure. The physician asks the nurse to obtain the informed consent signature from the patient. The nurse's best action to the physician's request is to: A. Inform the physician that is his responsibility to obtain the signature. B. Obtain the signature and ask another nurse to co-sign the signature. C. Inform the physician that the nurse manager will need to obtain the signature.

A) The responsibility for securing informed consent from the patient lies with the person who will perform the procedure. The nurse's best action is to inform the physician that it is his responsibility to obtain the signature.

A client who is started on metformin and glyburide would have initially present with with symptoms? A. Polydipsia, polyuria, and weight loss B. Weight gain, tiredness, and bradycardia C. irritability, diaphoresis and tachycardia D. Diarrhea, abdominal pain and weight loss

A) symptoms of hyperglycemia included polydispia, polyuria, and weightloss. Metformin and sulfonylureas are commonly ordered medications. Weight gain, tiredness, and bradycardia are symptoms of hypothyroidism. Irritability, diaphoresis, and tachycardia are symptoms of hypoglycemia. Symptoms of Crohn's disease include diarrhea, abdominal pain, and weight loss.

Select all that apply. Advantages of laser surgery include diminished A. bleeding. B. swelling. C. tissue damage. D. postoperative pain. E. postoperative infection.

A, B, C, D, & E (All of the above) Laser surgery offers the benefits of diminished bleeding, swelling, tissue damage, and postoperative pain and infection.

The nurse is conducting discharge teaching related to a new prescription for phenytoin (Dilantin). Which statements are appropriate to include in the teaching for this patient and his family? Select all that apply. A) "Be sure to call the clinic if you or your family notice increased anxiety or agitation." B) "You may have some mild sedation. Do not drive until you know how this drug will affect you." C) "This drug may cause easy bruising. If you notice this, call the clinic immediately." D) "It is very important to have good oral hygiene and visit your dentist regularly." E) "You may continue to have wine with your evening meals but only in moderation."

A, B, D Patients receiving an antiepileptic drug are at increased risk for suicidal thoughts and behavior beginning early in their treatment. The U.S. Food and Drug Administration (FDA) advises that patients, families, and caregivers be informed of the signs that may precede suicidal behavior and be encouraged to report these immediately. Mild sedation can occur in patients taking phenytoin even at therapeutic levels. Carbamazepine(Tegretol), not phenytoin, increases the risk for hematologic effects, such as easy bruising. Phenytoin causes gingival hyperplasia in about 20% of patients who take it. Dental hygiene is important. Patients receiving phenytoin should avoid alcohol and other central nervous system depressants because they have an additive depressant effect.

Which content about self-care should the nurse include in the teaching plan of a client who has genital herpes? (Select all that apply.) A. Encourage annual physical and Pap smear. B. Take antiviral medication as prescribed. C. Use condoms to avoid transmission to others. D. Warm sitz baths may relieve itching. E. Use Nystatin suppositories to control itching. F. Douche with weak vinegar solutions to decrease itching.

A,B,C,D. (E) is specific for Candida infections and (F) is used to treat Trichomonas.

21. The client scheduled for electroconvulsive therapy tells the nurse, "I'm so afraid. What will happen to me during the treatment?" Which of the following statements is most therapeutic for the nurse to make? A. "You will be given medicine to relax you during the treatment." B. "The treatment will produce a controlled grand mal seizure." C. The treatment might produce nausea and headache. d. You can expect to be sleepy and confused for a time after the treatment.

A. The patient will receive medication that relaxes skeletal muscles and produces mild sedation.

A pt has returned from surgery with a tracheostomy tube in place. After about 10 minutes in postoperative recovery, the pt begins to have noisy, increased respirations and an elevated heart rate. What action should the RN take immediately? A. Suction the tracheostomy. B. Readjust the tracheostomy tube and tighten the ties. C. Preform a complete respiratory assessment.

A. Noisy, increased respiration & increased pulse are signs that the pt needs immediate suctioning to clear the airway of secretions. A complete respiratory assessment may then be completed.

The reason pts are sent to a PACU after surgery is: A. to be monitored while recovering from anesthesia. B. to remain near the surgeon immediately after surgery. C. to allow the medical-surgical unit time to prepare for transfer. D. to provide time for the pt to cope with the effects of surgery.

A. Pts are sent to a PACU to be monitored while they're recovering from anesthesia.

2. The nurse evaluates which of the following clients to be at risk for developing hypernatremia? A. 50-year-old with pneumonia, diaphoresis, and high fevers B. 62-year-old with congestive heart failure taking loop diuretics C. 39-year-old with diarrhea and vomiting D. 60-year-old with lung cancer and syndrome of inappropriate antidiuretic hormone (SIADH)

A. 50-year-old with pneumonia, diaphoresis, and high fevers. Explanation Diaphoresis and a high fever can lead to free water loss through the skin, resulting in hypernatremia. Loop diuretics are more likely to result in a hypovolemic hyponatremia. Diarrhea and vomiting cause both sodium and water losses. Clients with syndrome of inappropriate antidiuretic hormone (SIADH) have hyponatremia, due to increased water reabsorption in the renal tubules.

Select all that apply. A nurse is caring for patients on a medical-surgical unit. The nurse plans the patients' care and instructs the nursing assistant to assist in repositioning patients every 2 hours. Which patients are at the greatest risk for complications if not repositioned properly? A. A 20-year-old unconscious patient B. A 90-year-old frail patient C. A 65-year-old patient who is visually impaired D. A 40-year-old patient who has paraplegia

A. A 20-year-old unconscious patient B. A 90-year-old frail patient (&) D. A 40-year-old patient who has paraplegia Patients who are at the greatest risk for complications if not properly repositioned are those who are unconscious, frail, or paralyzed.

A male client with arterial peripheral vascular disease (PVD) complains of pain in his feet. Which instruction should the nurse give to the UPA to quickly relieve the client's pain? A. Help the client to dangle his legs. B. Apply compression stockings. C. Assist with passive leg exercises. D. Ambulate three times daily.

A. A client who has arterial PVD may benefit from a dependent position which can be achieved by dangling by improving blood flow and relieving pain. (B) is indicated for venous insufficiency and (C) is indicated for bed rest. (D) is indicated to facilitate collateral circulation and may improve long term complaints of pain.

The nurse is observing an unlicensed assistive personnel (UPA) who is performing morning care for a bedfast client with Huntington disease. Which care measure is most important for the nurse to supervise? A. Oral care B. Bathing C. Foot care D. Catheter care

A. A client with Huntington disease experiences problems with motor skills such as swallowing and is at high risk for aspiration. (B, C, D) do not pose life-threatening consequences.

The nurse is performing hourly neurological check for a client with a head injury. Which new assessment finding warrants the most immediate intervention by the nurse? A. A unilateral pupil that is dilated and nonreactive to light. B. Client cries out when awakened by a verbal stimulus. C. Client demonstrates a loss of memory to the events leading up to the injury. D. Onset of nausea, headache, and vertigo.

A. Any changes in pupil size and reactivity is an indication of increasing ICP and should be reported immediately. (B) is normal for being awakened. (C & D) are common manifestations of head injury and less of an immediacy than (A).

In the operating room, a patient tells a circulating nurse that he is going to have the cataract in his left eye removed. If the nurse notes that the consent form indicates that surgery is to be performed on the right eye, what should be the nurse's first action? A. Ask the patient his name. B. Notify the surgeon and anesthesiologist. C. Check to see whether the patient has received any preoperative medications. D. Assume that the patient is a little confused because he is older and has received midazolam intramuscularly.

A. Ask the patient his name. Ensuring proper identification of a patient is a responsibility of all members of the surgical team. In a specialty surgical setting where many patients undergo the same type of surgery each day, such as cataract removal, it is possible that the patient and the record do not match. Nurses do not assume in the care of their patients. The priority is with the nurse identifying the patient and the patient's consent form before the physicians are notified.

A client diagnosed with angina pectoris complains of chest pain while ambulating in the hallway. Which action should the nurse implement first? A. Support the client to a sitting position. B. Ask the client to walk slowly back to the room. C. Administer a sublingual nitroglycerin tablet. D. Provide oxygen via nasal cannula.

A. Assist in safely repositioning and then administer (C & D). Then the client can be escorted back to the room via wheelchair or stretcher (B).

Which of the following preoperative assessment findings should be reported to a surgeon for preoperative treatment? A. Excessive thirst B. Gradual weight gain C. Overwhelming fatigue D. Recurrent blurred vision

A. Excessive thirst. The classic clinical manifestations of diabetes mellitus are increased frequency of urination (polyuria); increased thirst and fluid intake (polydipsia); and as the disease progresses, weight loss despite increased hunger and food intake (polyphagia). Weakness, fatigue, and recurrent blurred vision are associated with diabetes mellitus but are not considered priority manifestations because of the generalization of these complaints being applied to other disease processes. Weight loss is the cardinal sign related to the depletion of water, glycogen, and triglyceride stores.

The nurse is admitting a patient to the same day surgery unit. The patient tells the nurse that he was so nervous he had to take kava last evening to help him sleep. Which of the following nursing actions would be most appropriate? A. Inform the anesthesiologist of the patient's ingestion of kava. B. Tell the patient that using kava to help sleep was a good idea. C. Tell the patient that the kava should continue to help him relax before surgery. D. Inform the patient about the dangers of taking herbal medicines without consulting his health care provider.

A. Inform the anesthesiologist of the patient's ingestion of kava. Kava may prolong the effects of certain anesthetics. Thus the anesthesiologist needs to be informed of recent ingestion of this herbal supplement.

Select all that apply. A nurse is caring for a surgical patient in the preoperative area. The nurse obtains the patient's informed consent for the surgical procedure. Which statements are true regarding informed consent? A. Informed consent must be signed while the patient is free from mind-altering medications. B. Informed consent must be witnessed. C. Informed consent may be withdrawn at any time. D. Informed consent must be signed by patients age 16 and older. E. Informed consent must be obtained by the physician. F. Informed consent must be obtained from the family even in a life-threatening emergency.

A. Informed consent must be signed while the patient is free from mind-altering medications. B. Informed consent must be witnessed. An informed consent must be signed while the patient is free from mind-altering medications and must be witnessed after it has been determined that the patient has received all of the necessary information needed to make an informed decision. An informed consent may be withdrawn at any time before the procedure and must be signed by patients age 18 and older. A parent or guardian's signature is required for minors. The informed consent may be obtained by the physician or the nurse and is not required in the event of a life-threatening emergency.

Select all that apply. Which of the following best describes a consent form? A. May be signed by an emancipated minor. B. Protects the health care facility but not the physician C. Signifies that the patient understands all aspects of the procedure. D. Signifies that the patient and family have been told about the procedure E. Must be signed by the patient or responsible party at the health care facility, and that consent may not be obtained by phone or fax

A. May be signed by an emancipated minor. (&) C. Signifies that the patient understands all aspects of the procedure. A consent form may be signed by an emancipated minor, and consent may be obtained by fax or phone with appropriate witnesses. Only in the cases of underage children or unconscious or mentally incompetent people must a family member be aware of the procedure. The document protects the surgeon and the health care facility in that it indicates that the patient knows and understands all aspects of the procedure.

Which of the following nursing interventions should receive highest priority when a patient is admitted to the postanesthesia care unit? A. Positioning the patient B. Observing the operative site C. Checking the postoperative orders D. Receiving report from operating room personnel.

A. Positioning the patient. A patient is received in the postanesthesia care unit on a bed or stretcher. Proper positioning is necessary to ensure airway patency in a sedated, unconscious, or semiconscious patient. Observation of the operative site, receiving report from operating room personnel, and checking postoperative orders are interventions made after proper positioning of the patient.

9. The nurse is caring for a bedridden client admitted with multiple myeloma and a serum calcium level of 13 mg/dl. Which of the following is the most appropriate nursing action? A. Provide passive ROM exercises and encourage fluid intake B. Teach the client to increase intake of whole grains and nuts C. Place a tracheostomy tray at the bedside D. Administer calcium gluconate IM as ordered

A. Provide passive ROM exercises and encourage fluid intake Explanation A client who has a serum calcium of 13 mg/dl has hypercalcemia. Normal serum calcium is 9 to 11 mg/dl. Fluid intake promotes renal excretion of excess calcium. ROM exercises promote reabsorption of calcium into bone. Placing a tracheostomy at the bedside is a nursing intervention for hypocalcemia. Although calcium gluconate may be administered in hypocalcemia, it is never administered IM.

In assessing an older client with dementia for sundowning syndrome, what assessment technique is best for the nurse to use? A. Observe for tiredness at the end of the day. B. Perform a neurologic exam and mental status exam. C. Monitor for medication side effects. D. Assess for decreased gross motor movement.

A. Sundowning syndrome is a pattern of agitated behavior in the evening, believed to be associated with tiredness at the end of the day combined with fewer orienting stimuli, such as activities and interactions. (B, C, & D) with not provide information about this syndrome.

A client with hypertension has been receiving ramipril (Altace) 5 mg PO daily for 2 weeks and is scheduled to receive a dose at 0900. At 0830 the client's blood pressure is 120/70. Which action should the nurse take? A. Administer the dose as prescribed. B. Hold the dose and contact the healthcare provider. C. Hold the dose and recheck the blood pressure in 1 hour. D. Check the healthcare provider's prescription to clarify the dose.

A. The BP is WNL and indicates that the medication is working. (B & C) would be indicated if the BP was low (systole below 100). (D) is not required because the dose is within manufacture's recommendations.

Ritalin, Stratera

ADHD children ADD adults CNS STIMULANT; schedule II stratera (nonscheduled drug, decreases hyperactivity)

During assessment of a client in the intensive care unit, the nurse notes that the client's breath sounds are clear upon auscultation, but jugular vein distention and muffled heart sounds are present. Which intervention should the nurse implement? A. Prepare the client for a pericardial tap. B. Administer intravenous furosemide (Lasix). C. Assist the client to cough and deep breathe. D. Instruct the client to restrict oral fluid intake.

A. The client is exhibiting symptoms of cardiac tamponade that results in reduced cardiac output. Treatment is pericardial tap. (B) is not a treatment. (C) is not priority. (D) Fluids are frequently increased but this is not as priority as (A).

A client with cirrhosis states that his disease was cause by a blood transfusion. What information should the nurse obtain first to provide effective client teaching? A. The year the blood transfusion was received B. The amount of alcohol the client drinks C. How long the client has had cirrhosis D. The client's normal coping mechanisms

A. The nurse should first verify the clients explanation (A) since it may be accurate due to prior to 1990 blood was not screened for Hep C and hep C can cause cirrhosis. Not all cirrhosis is caused is caused by alcoholism (B) (C & D) provide useful but less relevant information.

The nurse is caring for a client with a chest tube to water seal drainage that was inserted 10 days ago because of a ruptured bullae and pneumothorax. Which finding should the nurse report to the healthcare provider before the chest tube is removed? A. Tidal of water in the water seal chamber B. Bilateral muffled breath sounds at bases C. Temperature of 101 degrees F D. Absence of chest tube drainage for 2 days.

A. Tidal in the water seal chamber should be reported to the HPC to show that the chest tube is working properly. (B) may indicate hypoventilation from the chest tube and usually improves when the tube is removed. (C) indicates infection (D) is an expected finding.

A client is placed on a mechanical ventilator following a cerebral hemorrhage, and vecuronium bromide (Norcuron) 0.04 mg/kg every 12 hours IV is prescribed. What is the priority nursing diagnosis for this client? A. Impaired communication related to paralysis of skeletal muscles. B. Hight risk or infection related to increased ICP. C. Potential for injury related to impaired lung expansion. D. Social isolation related to inability to communicate.

A. To increase the client's tolerance of the endotracheal intubation and/or mechanical ventilation, a skeletal-muscle relaxant such as vecuronium is usually prescribed. (A) is a serious outcome because the client cannot communicate his/her needs. (D) is not as much of a priority. (B) infection is not related to ICP. (C) is incorrect because the ventilator will ensure that the lungs are expanded.

In caring for a person receiving an opioid analgesic through an epidural catheter, the nursing responsibility of prime importance is A. assessing for respiratory depression. B. establishing a baseline laboratory profile. C. inspecting the catheter insertion site hourly. D. ensuring that the patient remains on strict bed rest.

A. assessing for respiratory depression. Possible side effects of epidural opioids are pruritus, urinary retention, and delayed respiratory depression, occurring 4 to 12 hours after a dose. Establishing a baseline laboratory profile is outside the scope of practice for a nurse. Hourly inspection of the catheter insertion site is an unnecessary nursing intervention. In general, the site is assessed once a shift unless unexpected complications occur. Strict bed rest is not necessary for the patient with an epidural catheter; however, assistance with getting out of bed could be necessary related to effects of the opioid analgesic.

The primary goal of the circulating nurse during preparation of the operating room, transferring and positioning the patient, and assisting the anesthesia team is A. avoiding any type of injury to the patient. B. maintaining a clean environment for the patient. C. providing for patient comfort and sense of well-being. D. preventing breaks in aseptic technique by the sterile members of the team.

A. avoiding any type of injury to the patient. The protection of the patient from injury in the operating room environment is maintained by the circulating nurse by ensuring functioning equipment, preventing falls and injury during transport and transfer, monitoring asepsis, and being with the patient during anesthesia induction.

doxazosin/ Cardura

ADRENERGIC BLOCKER hypertension, Selective alpha1-blocker; also for BPH

carvedilol/ Coreg

ADRENERGIC BLOCKER hypertension; blocks both alpha and beta receptors; also for heart failure

when should NPH insulin be given?

AFTER the person has eaten

Symlin (pramlintide acetate)

Amylin analog

When administering low-molecular-weight heparin (LMWH) after an operation, a nurse should A. explain that the drug will help prevent clot formation in the legs. B. check the results of the partial thromboplastin time before administration. C. administer the dose with meals to prevent GI irritation and bleeding. D. inform the patient that blood will be drawn every 6 hours to monitor the prothrombin time.

A. explain that the drug will help prevent clot formation in the legs. Unfractionated heparin or LMWH is given as a prophylactic measure for venous thrombosis and pulmonary embolism. These anticoagulants work by inhibiting thrombin-mediated conversion of fibrinogen to fibrin. LMWH is injected subcutaneously with no relationship to meals. It has a more predictable dose response and less risk of bleeding complications. It does not require anticoagulant monitoring and dosage adjustments.

A male client who has never smoked but has had COPD for the past 5 years is now being assessed for cancer of the lung. The nurse knows that he is most likely to develop which type of lung cancer? A. Adenocarcinoma B. Oat-cell carcinoma C. Malignant melanoma D. Squamous-cell carcinoma

A. is the only lung cancer not related to cigarette smoking related to lung scarring and fibrosis from preexisting pulmonary diseases such as TB and COPD. (B& D) are related to smoking. (C) is a skin cancer

A patient is scheduled for a hemorrhoidectomy at an ambulatory day-surgery center. An advantage of performing surgery at an ambulatory center is a decreased need for A. laboratory tests and perioperative medications. B. preoperative and postoperative teaching by the nurse. C. psychologic support to alleviate fears of pain and discomfort. D. preoperative nursing assessment related to possible risks and complications.

A. laboratory tests and perioperative medications. Ambulatory surgery is usually less expensive and more convenient, generally involving fewer laboratory tests, fewer preoperative and postoperative medications, less psychological stress, and less susceptibility to hospital-acquired infections. However, the nurse is still responsible for assessing, supporting, and teaching the patient undergoing surgery, regardless of where the surgery is performed.

A client in the trauma ICU is experiencing deep, throbbing pain. The nurse will provide medication for this pain because: A.) The pain is being transmitted over C fibers and the enkephalins will not be effective to control the pain. B.) The pain is being transmitted over A beta fibers and beta-endorphins will not be effective to control the pain. c.) The pain is being transmitted over delta fibers and dynorphins will not be effective to control the pain. d.) The pain is being transmitted over A beta fibers and the dynophins will not be effective to control the pain.

A.) The pain is being transmitted over C fibers and the enkephalins will not be effective to control the pain.

enalapril/ Vasotec

ACE INHIBITOR hypertension, heart failure

captopril/ Capoten

ACE INHIBITOR Hypertension, Heart Failure, and myocardial infarction

Oral hypoglycemic agents ABSORPTION DELAYERS

ALPHA-GLUCOSIDASE INHIBITORS (AGIs)-Lower postprandial glucose by reducing rate of carb digestion and absorption EXAMPLES- acarbose (Precose) miglitol (Glyset) S/E- Flatulance, Bloating NURSING IMPLICATIONS: -Give at start of each meal -No weight gain or hypoglycemia risk -Multiple dosing less convenient -If used in combination with another drug and hypoglycemia occurs, treat with milk or glucose tablets, not table sugar

losartan/ Cozaar

ANGIOTENSIN-RECEPTOR BLOCKER hypertension, causes relaxation of smooth vascular muscles

The teaching plan for a diabetic is focused on smoking cessation and control of hypertension for the avoidance of microvascular complications, such as (select all that apply): 1. macular degeneration. 2. end-stage renal disease (ESRD). 3. coronary artery disease (CAD). 4. peripheral vascular disease (PVD). 5. cerebrovascular accident (CVA).

ANS: 1, 2 Macular degeneration and ESRD are both microvascular complications. CAD, PVD, and CVA are all macrovascular complications.

When the type 2 diabetic patient says, "Why in the world are they looking at my hemoglobin? I thought my problem was with my blood sugar." The nurse responds that the level of hemoglobin A1c: 1. shows how a high glucose level can cause a significant drop in the hemoglobin level. 2. shows what the glucose level has done for the last 3 months. 3. indicates a true picture of the patient's nutritional state. 4. reflects the effect of high glucose levels on the ability to produce red blood cells.

ANS: 2 By analyzing the amount of glucose bound to the hemoglobin, the level of blood glucose can be evaluated for the last 3 months, because the glucose stays bound to the hemoglobin for the life of the red blood cell (RBC).

The home health nurse is assessing a type 1 diabetic patient who has been controlled for 6 months. The nurse is surprised and concerned about a blood glucose reading of 52. This episode of hypoglycemia is probably caused by the patient's having: 1. taken a new form of birth control pill this morning. 2. used large amounts of sugar substitute in her tea this morning. 3. had a 2-hour long exercise class at the spa this morning. 4. underdosed herself with insulin this morning.

ANS: 3 Excessive exercise used up the glucose that was made available by the insulin taken by the patient. The patient now has too much insulin for the available glucose and has become hypoglycemic.

The patient with type 2 diabetes shows a blood sugar reading of 72 at 6 AM. Based on the reading of 72, the nurse should: 1. notify the charge nurse of the reading. 2. give regular insulin per sliding scale. 3. give him cup of milk. 4. administer the oral hyperglycemic tablet.

ANS: 3 milk The patient is hypoglycemic and needs an immediate source of glucose, such as milk or orange juice. The oral hyperglycemic agent will not work quickly enough. Notifying the charge nurse can be done later. Giving insulin per sliding scale would lower the blood sugar level.

When a newly diagnosed type 2 diabetes mellitus patient asks the nurse why she has to take a pill instead of insulin, you reply that in type 2 diabetes, the body makes insulin but: 1. overweight and underactive people simply cannot use the insulin produced. 2. metabolism is slowed in some people so they have to take a pill to speed up their metabolism. 3. sometimes the autoimmune system works against the action of the insulin. 4. the cells become resistant to the action of insulin. Pills are given to increase the sensitivity.

ANS: 4 Type 2 diabetes mellitus is a disease in which the cells become resistant to the action of insulin and the blood glucose level rises. Oral hyperglycemic agents make the cells more sensitive.

tinnitus

Perception of sound such as ringing, buzzing, or roaring in the ears

Histamine 2 Blockers- Anti Ulcer

Blocks the H2 receptor of the parietal cells in the stomach, therefore reducing gastric acid secretions and concentration Promotes healing by eliminating the cause.

Propacil or Tapazole

Blocks the utilization of iodineby interfering with the Iodothyrosines in the synthesis of thyroid hormones

Glucose in urine is called

Glycosuria

Colorless urine

Can be caused by dehydration which can cause you to go into shock

Upper GI fibroscopy

Esophageal, gastric, and duodenal mucosa are viewed through an endoscope.

A patient with hyperthyroidism is taking propylthiouracil (PTU). The nurse will monitor the patient for: A) gingival hyperplasia and lycopenemia. B) dyspnea and a dry cough. C) blurred vision and nystagmus. D) fever and sore throat.

D) fever and sore throat.

Amenorrhea

Absence of menses

Bulk-Forming Laxative

Absorb water into intestines, increasing bulk and peristalsis Results: 8 - 12 hours Admin - mix in glass of water or juice; stir; drink immediately and follow with a glass of water

Side Effects of Bulk-Forming Laxative

Abdominal cramps, Excess laxative, nausea, vomiting, gas, diarrhea

Criteria for metabolic or cardiometabolic syndrome

Abdominal obesity, elevated triglycerides, decreased HDLs, BP >130/85 mm Hg, fasting plasma glucose (FPG) >110 mg/dL

demecarium bromide/ Humorsol

ANTGLAUCOMA DRUGS THAT INCREASE THE OUTFLOW OF AQUEOUS HUMOR MIOTICS, CHOLINESTERASE INHIBITORS Ophthalmic solution; longer-acting medication (2-3) days

pilocarpine hydrochloride/Adsorbocarpine,Isopto carptine, Pilopine, others)

ANTGLAUCOMA DRUGS THAT INCREASE THE OUTFLOW OF AQUEOUS HUMOR MIOTICS, DIRECT-ACTING CHOLINERGIC AGENTS Ophthalmic solution; cholinergic agent; may be prescribed as an ocular therapeutic system, a slow release delivery method (Ocusert); Ocusert effects can last up to 7 days

latanoprose/ Xalatan

ANTGLAUCOMA DRUGS THAT INCREASE THE OUTFLOW OF AQUEOUS HUMOR PROSTAGLANDINS AND PROSTAMIDES ophthalmic solution; prostaglandin

tofranil/ Elavil

ANTIDEPRESSANTS TRICYCLIC ANTIDEPRESSANTS old drug; inhibit reuptake of NE and serotonin s.e: anticholinergic (Urinary retention, constipation) interact with many medical conditions heart, lung, GI conditions; orthostatic hypotension

brimonidine tartrate/Alphagan

ANTIGLAUCOMA DRUGS THAT DECREASE THE FORMATION OF AQUEOUS HUMOR ALPHA2-ADRENERGIC AGENTS Ophthalmc solution

timolot/ Timoptic, Timipotic

ANTIGLAUCOMA DRUGS THAT DECREASE THE FORMATION OF AQUEOUS HUMOR BETA-ADRENERGIC BLOCKERS ophthalmic solution; nonspecific beta blocker

brinzolamide/Azopt

ANTIGLAUCOMA DRUGS THAT DECREASE THE FORMATION OF AQUEOUS HUMOR CARBONIC ANHYDRASE INHHIBITORS Ophthalmic solutuion; sulfonamide

actazolamide/ Diamox

ANTIGLAUCOMA DRUGS THAT DECREASE THE FORMATION OF AQUEOUS HUMOR CARBONIC ANHYDRASE INHIBITORS Oral diuretic; sulfonamide; also for sezures, high altitude sickness, and renal impairment

isosorbide/ Ismotic

ANTIGLAUCOMA DRUGS THAT DECREASE THE FORMATION OF AQUEOUS HUMOR OSMOTIC DIURETICS Used before and after eye surgery

Side Effects of Stimulant Laxative (Irritants)

Abdominal Cramps, weakness, reddish brown urine, diarrhea

Type 2 Diabetes

Acquired; insulin resistance

Cannabinoids - Antiemetic

Active ingredient - marijuana Marinol

Which nursing diagnosis is most appropriate for a patient with weight gain and fatigue related to hypothyroidism.

Activity Intolerance related to fatique.

Anticholinergics - Antiemetic

Acts on the VC; decreases stimulation of the CTZ

Tranquilizer - Anti Ulcer

Acts on the vagus nerve to stop the production of acetylcholine. Decrease anxiety and vagal stimulation.

adrenocortical insufficiency

Addison's disease

all 3 classes of adrenal corticosteroids (glucocorticoids, mineralocorticoids, and androgens) are reduced

Addison's disease

What ethnicities are more at risk for getting type 2 diabetes?

African Americans, Mexican Americans, American Indians, Asian Americans, Pacific Islanders

The public health nurse is preparing to teach the members of the local swim club about care of the ears. Which of the following statements would be appropriate for the nurse to include? Use a clean cotton swab to dry the ear canal after swimming. A tight-fitting swim cap is preferred to earplugs for keeping water out of the ear. If there is debris in the ear canal, irrigate the ear with cold water. After an episode of acute inflammation, the swimmer should wait 7-10 days before returning to the water.

After an episode of acute inflammation, the swimmer should wait 7-10 days before returning to the water. Rationale: Guidelines include: Stay out of the water until the acute inflammatory process is completely resolved-ideally, 7-10 days before resuming water activities; and use silicone earplugs, and dry the outer ear with a towel, then use a hair dryer on the lowest setting several inches from the ear to dry the canal. Do not insert cotton swabs or other objects into the ear canal to dry it. A tight-fitting swim cap does not keep water out of the ear. Repeated exposure to cold water encourages the growth of exostoses in the ear canal.

36. The nurse is teaching the client with insulin-dependent diabetes the signs of hypoglycemia. Which of the following signs is associ- ated with hypoglycemia? A. Tremulousness B. Slow pulse C. Nausea D. Flushed skin

Answer A is correct. Tremulousness (a state of trembling or quivering) is an early sign of hypoglycemia. Answers B,C, and D are incorrect because they are symptoms of hyperglycemia.

The nurse is caring for a patient with type 2-DM who has taken an oral hypoglycemic agent who is now experiencing hunger, sweating, headache and agitation. which action should a nurse take first?

Give 4oz of OJ

What are other major risk factors for type 2 diabetes?

Age over 40 years with a BMI greater than 25 Inactive lifestyle Heredity link If you delivered a baby over 9 lbs Hypertensive with a BP over 140/90 Have any history of vascular disease Have metabolic syndrome X

presbycusis

Age-related loss of the ability to hear high-frequency sounds, may occur because of cochlear hair cell degeneration or loss of auditory neurons in the organ of Corti.

To remember adrenal cortex hormones think Salt, Sugar and Sex

Aldosterone promotes salt retention Cortisol affects sugar (carbohydrate) metabolism Androgens are sex hormones

Glyset (miglitol)

Alpha-glucosidase inhibitor antidiabetic

Precose (acarbose)

Alpha-glucosidase inhibitor antidiabetic

Example of Antacids - without Systemic Effects

Aluminum Salts - Amphojel Magnesium Salts - Combined - Aluminum Hydroxide and Magnesium Hydroxide - Maalox

Neomycin

Aminoglycoside antibiotic

Diabetes Mellitus

Disorder of pancreas where beta cells of islets of langerhans fail to produce adquate amount of insulin -Disease in which high blood glucose results from defective insulin secretiono r action * Juvenile or insulin dependent (adult onset can be non-insulin dependent)

Tylenol (acetominophen)

Analgesic, antipyretic

Mylanta (simethicone, magnesium hydroxide, and aluminum hydroxide combo)

Antacid

A nurse is caring for a cancer patient receiving subcutaneous morphine sulfate for pain. Which of the following nursing actions is most important in the care of this patient? A. Monitor urine output. B. Monitor respiratory rate. C. Monitor heart rate. D. Monitor temperature.

Answer: B Morphine sulfate can suppress respiration and respiratory reflexes, such as cough. Patients should be monitored regularly for these effects to avoid respiratory compromise. Morphine sulfate does not significantly affect urine output, heart rate, or body temperature.

Riopan (magaldrate oral)

Antacid

A patient arrives at the emergency department complaining of back pain. He reports taking at least 3 acetaminophen tablets every three hours for the past week without relief. Which of the following symptoms suggests acetaminophen toxicity? A. Tinnitus. B. Diarrhea. C. Hypertension. D. Hepatic damage.

Answer: D Acetaminophen in even modestly large doses can cause serious liver damage that may result in death. Immediate evaluation of liver function is indicated with consideration of N-acetylcysteine administration as an antidote. Tinnitus is associated with ASPIRIN overdose, not acetaminophen. Diarrhea and hypertension are not associated with acetaminophen.

Maalox (simethicone, magnesium hydroxide, and aluminum hydroxide combo)

Antacid

Coumadin

Anticoagulant. Trade names: Warfarin. It is the only oral anticoagulant prescribed today. Dosage: PO: 5 to 10 mg/d for 2 to 5 d; maint: 2 to 10 mg/d. Normal dose is 5 mg. Half Life: 1 to 3 days. PB: 99%. Peak: 1 to 3 d. Duration: 2.5 to 5 d. Action: To prevent blood clotting. Mode of Action: Depression of hepatic synthesis of vitamin K clotting factors (II [prothrombin], VII, IX and X) Side Effects: occult bleeding, anorexia, nausea, vomiting, diarrhea, abdominal cramps, rash, fever. Adverse reactions: Stomatitis. Contraindications: Blood dyscrasias, eclampsia. Anticoagulant antagonist: vitamin K. Therapeutic range: 2-3 (2= thick and 3= thin). Nursing Implications: Clients should be monitored closely for signs of bleeding. Laboratory testing of PT or INT should be scheduled at recommended intervals. Oral anticoagulants prolong clotting time by thinning the blood. Long half-life and duration, therefore must stop taking medication weeks before surgery. Platelet count should be monitored. Examine the client\'s mouth, nose, urine, and skin for bleeding. Check stools periodically for occult blood. Teaching: Instruct client to inform dentist when taking an anticoagulant. Use a soft toothbrush to prevent gums from bleeding. Shave with an electric razor. Have laboratory tests such as PT or INR performed. Suggest client to carry a medical ID card or wear a bracelet. Encourage client not to smoke. Aspirin should NOT be taken with coumadin. Advise client to avoid large amounts of foods with vitamin K (green, leafy vegetables, broccoli)

Glucovance

Antidiabetic combo

Imodium (loperamide hydrochloride)

Antidiarrheal agent

Lomotil (diphenoxylate and atropine combo)

Antidiarrheal agent

Compazine (prochlorperazine)

Antiemetic agent

Phenergan (promethazine)

Antiemetic agent

Tigan (trimethobenzamide hydrochloride)

Antiemetic agent

Mycostatin

Antifungal agent

Nilstat

Antifungal agent

Nystatin

Antifungal agent

Propylthiouracil (PTU)

Antithyroid agent

Tapazole (methimazole)

Antithyroid agent

Carafate (sucralfate)

Antiulcer agent

If you have metabolic syndrome, what other diseases are you more at risk for?

Arthrosclerosis CVA - stroke CAD - coronary artery disease Early death

Patient education for Fluid Volume Deficit related to Addison's disease

Assess knowledge base, teach hormone replacement (2/3 of dose in A.m. and 1/3 in p.m., as ordered), teach how to recognize stress and alter treatment during stressful times. Patient may need increased sodium intake during hot weather, should wear Medic Alert identification.

Electrogastrography

Assesses gastric motility, motor/nerve dysfunction. Electrodes on the abdomen record for up to 24 hrs.

Wellbutrin (Smoking cessation)

Atypical inhibit reuptake of serotonin; affect NE and dopamine

Nursing interventions: ACTH and corticotropin

Avoid admin of corticotropin to clients w/ adrenocortical hyperfunction. Corticotropin stimulates release of cortisol from adrenals. Monitor growth/development of child receiving corticotropin Observe weight. If gain occurs, check for edema. A side effect of ACTH is Na and water retention Watch carefully for adverse effects when corticotropin is discontinued. Dose should be tapered and not stopped abruptly, b/c adrenal hypofunction may result Check labs, especially electrolyte levels. Electrolyte replacement may be needed

When the patient who has undergone cataract extraction is preparing to go home, the nurse reinforces which of the following instructions? Remove the dressing at bedtime. A headache for the first few days after surgery should be expected. Avoid sleeping on the operative side. Set a new appointment with the surgeon for 1 month from today.

Avoid sleeping on the operative side. Rationale: The nurse should reinforce any limitation, such as avoiding reading, lifting, and strenuous activity, and sleeping on the operative side. The patient also needs to understand the importance of not disturbing the eye dressing, of follow-up appointments, and of manifestations of complications such as eye pain, change in vision, headache, or nausea.

A patient with newly diagnosed type 2 diabetes mellitus asks the nurse what "type 2" means in relation to diabetes. The nurse explains to the patient that type 2 diabetes differs from type 1 diabetes primarily in that with type 2 diabetes a. the patient is totally dependent on an outside source of insulin. b. there is decreased insulin secretion and cellular resistance to insulin that is produced. c. the immune system destroys the pancreatic insulin-producing cells. d. the insulin precursor that is secreted by the pancreas is not activated by the liver.

B Rationale: In type 2 diabetes, the pancreas produces insulin, but the insulin is insufficient for the body's needs or the cells do not respond to the insulin appropriately. The other information describes the physiology of type 1 diabetes.

A nurse caring for a patient who has diabetic ketoacidosis recognizes which of these characteristics in the patient? (Select all that apply.) A) Occurs mainly in type 2 diabetes patients B) Altered fat metabolism leading to ketones C) Arterial blood pH of 7.35 to 7.45 D) Sudden onset, triggered by acute illness E) Plasma osmolality of 300 to 320 milliosmoles/L

B D E

The nurse is teaching a patient who is newly diagnosed with epilepsy about her disease. Which statement made by the nurse best describes the goals of antiepilepsy medication therapy? A) "With proper treatment we can completely eliminate your seizures." B) "Our goal is to reduce your seizures to an extent that helps you live a normal life." C) "Epilepsy medication does not reduce seizures in most patients." D) "These drugs will help control your seizures until you have surgery."

B) "Our goal is to reduce your seizures to an extent that helps you live a normal life." Epilepsy is treated successfully with medication in a majority of patients. However, the dosages needed to completely eliminate seizures may cause intolerable side effects. Neurosurgery is indicated only for patients in whom medication therapy is unsuccessful.

The nurse is caring for a patient whose seizures are characterized by a 10- to 30-second loss of consciousness with mild symmetric eye blinking. Which seizure type does this most closely illustrate? A) Tonic-clonic B) Absence C) Atonic D) Myoclonic

B) Absence This scenario accurately describes absence seizures. Tonic-clonic seizures present with convulsions and muscle rigidity followed by muscle jerks. Patients may experience urinary incontinence and loss of consciousness. Atonic seizures cause sudden loss of muscle tone. Myoclonic seizures present with sudden muscle contractions that last but a second.

The nurse is preparing to give ethosuximide (Zarontin). The nurse understands that this drug is only indicated for which seizure type? A) Tonic-clonic B) Absence C) Simple partial D) Complex partial

B) Absence Absence seizures are the only indication for ethosuximide. The drug effectively eliminates absence seizures in approximately 60% of patients and effectively controls 80% to 90% of cases.

A patient admitted to the hospital with a diagnosis of pneumonia asks the nurse, "Why am I receiving codeine? I don't have any pain." The nurse's response is based on the knowledge that codeine also has which effect? A) Immunostimulant B) Antitussive C) Expectorant D) Immunosuppressant

B) Antitussive Codeine provides both analgesic and antitussive therapeutic effects.

A pediatric nurse is preparing a child for cleft palate repair surgery. The nurse recognizes that this type of surgery is categorized as: A. Transplantation surgery B. Constructive surgery C. Palliative surgery D. Reconstructive surgery

B) Cleft palate repair considered constructive surgery because the goal is to restore function in congenital anomalies. Reconstructive surgery serves to restore function to traumatized or malfunctioning tissues and includes plastic surgery or skin grafting. Transplant surgery replaces organs or structures that are diseased or malfunctioning, such as a liver or kidney transplant. Palliative surgery is not a curative and seeks to relieve or reduce the intensity of an illness, such as debridement or necrotic tissue.

Which of the following statements from a newly diagnosed client with diabetes indicates more instruction is needed? A.) i need to check my feet daily for sores B.) i need to store my insulin in the refrigerator C.) i can use my plastic insulin syringe more than once D.) i need to see my doctor for follow up exams

B.) i need to store my insulin in the refrigerator Insulin only needs to be stored in the refrigerator if it wont be used within 6 weeks, after being opened. It should be at room temperature when given to decrease pain and prevent lipodystrophy.

The nurse is planning care for a patient receiving morphine sulfate (Duramorph) by means of a patient-controlled analgesia (PCA) pump. Which intervention may be required because of a potential adverse effect of this drug? A) Administer cough suppressant. B) Insert Foley catheter. C) Administer antidiarrheal. D) Monitor liver function tests.

B) Insert Foley catheter. Morphine can cause urinary hesitancy and urinary retention. If bladder distention or the inability to void is noted, the prescriber should be notified. Urinary catheterization may be required. Morphine acts as a cough suppressant and an antidiarrheal, so neither of those drugs would need to be administered to counteract an adverse effect of morphine. Liver toxicity is not a common adverse effect of morphine.

Following heparin treatment for a pulmonary embolism, a client is being discharged with a prescription for warfarin (Coumadin). In conducting discharge teaching, the nurse advises the client to have which diagnostic test monitored regularly after discharge? A) Perfusion scan B) Prothrombin Time (PT/INR) C) Activated partial thromboplastin (APTT) D) Serum Coumadin level (SCL

B) Prothrombin Time (PT/INR)

A nurse assesses a patient who is taking pramlintide (Symlin) with mealtime insulin. Which of these findings should require immediate follow-up by the nurse? A) Skin rash B) Sweating C) Itching D) Pedal edema

B) Sweating Pramlintide is a new type of antidiabetic medication that is used as a supplement to mealtime insulin in type 1 and 2 diabetes. Hypoglycemia, which is manifested by sweating, tremors, and tachycardia, is the adverse reaction of most concern. Skin rash, itching, and edema are not adverse effects of pramlintide.

The nurse is working on a postoperative unit where pain management is part of routine care. Which statement below is the most helpful in guiding clinical practice in this setting? A) At least 30% of the U.S. population is prone to drug addiction and abuse. B) The development of opioid dependence is rare when opioids are used for acute pain. C) Morphine is a common drug of abuse in the general population. D) The use of PRN (as needed) dosing provides the most consistent pain relief without risk of addiction.

B) The development of opioid dependence is rare when opioids are used for acute pain.

The operating room is aware that which of the following patients are at a greater risk related to a surgical procedure? A. 34 yr old female B. 83 yr old female C. 48 yr old male D. 8 yr old male

B)Infants and older adults are at greatest risk from surgery than are children and young or middle-aged adults. Physiologic changes associated with aging increase the surgical risk for older patients.

A patient returning to the floor after orthopedic surgery is complaining of nausea. The nurse is aware that an appropriate intervention is to: A. Hold all medications. B. Avoid strong smelling foods. C. Avoid oral hygiene until the nausea subsides. D. Provide clear liquids with a straw.

B)Nursing care for a patient with nausea includes avoiding strong smelling foods. Providing oral hygiene, administering prescribed medications (especially if medications ordered are anti-nausea/antiemetics), and avoid the use of a straw.

A patient taking Dilantin (phenytoin) for a seizure disorder is experiencing breakthrough seizures. A blood sample is taken to determine the serum drug level. Which of the following would indicate a sub-therapeutic level? A. 15 mcg/mL. B. 4 mcg/mL. C. 10 mcg/dL. D. 5 mcg/dL.

B. 4 mcg/mL. The therapeutic serum level for Dilantin is 10 - 20 mcg/mL. A level of 4 mcg/mL is sub-therapeutic and may be caused by patient non-compliance or increased metabolism of the drug. A leve of 15 mcg/mL is therapeutic. Choices C and D are expressed in mcg/dL, which is the incorrect unit of measurement.

functional loss - definition

BCVA < 20/60 or sig central field defect

propranolol/ Inderal, InnoPran XL

BETA-ADRENERGIC BLOCKERS (CLASS II) dysrhythmias; also for hypertension, prevention of MI, angina, and migraines

The nurse is planning the care for a client who is admitted with the syndrome of inappropriate antidiuretic hormone secretion (SIADH). Which interventions should the nurse include in this client's plan of care? (Select all that apply.) A. Salt-free diet B. Quiet environment C. Deep tendon reflex assessments D. Neurologic checks E. Daily weights F. Unrestricted intake of free water

B, C, D, E. SIADH results in water retention and dilutional hyponatremia, which causes neurologic change when serum sodium levels are less than 115 mEq/L. The nurse should maintain a quiet environment (B) to prevent overstimulation that can lead to periods of disorientation, assess deep tendon reflexes (C) and neurologic checks (D) to monitor for neurologic deterioration. Daily weights (E) should be monitored to assess for fluid overload: 1 kg weight gain equals 1 L of fluid retention, which further dilutes serum sodium levels. (A and F) contribute to dilutional hyponatremia.

Untreated hyperglycemia may lead to all of the following complications except: a. Hyperosmolar syndrome b Vitiligo c. Diabetic ketoacidosis d. Coma

B. Excessively high blood sugar or prolonged hyperglycemia can cause diabetic ketoacidosis, the condition in which the body breaks down fat for energy and ketones spill into the urine. Diabetic hyperosmolar syndrome occurs when blood sugar is excessively high and available insulin is ineffective. In this case, the body cannot use glucose or fat for energy and glucose is excreted in the urine. Without immediate medical attention, both conditions may result in coma or death.

Mr. Jessie Ray, a newly admitted patient, has a seizure disorder which is being treated with medication. Which of the following drugs would the nurse question if ordered for him? A. Phenobarbitol, 150 mg hs B. Amitriptylene (Elavil), 10 mg QID. C. Valproic acid (Depakote), 150 mg BID D. Phenytoin (Dilantin), 100 mg TID

B. Elavil is an antidepressant that lowers the seizure threshold, so would not be appropriate for this patient. The other medications are anti-seizure drugs.

A 20-year-old client presents to the clinic with complaints of breast tenderness, nausea, vomiting, and absence of menses for 2 months. She has a history of a seizure disorder well controlled with carbamazepine (Tegretol). She tells the nurse that she has been taking her oral contraceptives as directed, but she wonders if she might be pregnant. The nurse's best response to her concern should be which of the following? A. "You can't be pregnant if you have been taking your oral contraceptives correctly." B. "Carbamazepine can decrease the effectiveness of oral contraceptive drugs, so we need to do a pregnancy test." C. "There is no need to worry. Oral contraceptives are very effective." D. "Taking antiseizure drugs with oral contraceptives significantly decreases your risk of getting pregnant."

B. "Carbamazepine can decrease the effectiveness of oral contraceptive drugs, so we need to do a pregnancy test.

Seconal 0.1 gram PRN at bedtime is prescribed for rest. The scored tablets are labeled grain 1.5 per tablet. How many tablets should the nurse plan to administer? A. 1/2 tablet B. 1 tablet C. 1 1/2 tablet D. 2 tablets

B. 15 gr = 1 g, 0.1 x 15 = 1.5 grains

The nurse is interviewing a client who is taking interferon-alfa-2a (Roferon-A) and ribavirin (Virazole) combination therapy for hepatitis C. The client reports experiencing overwhelming feelings of depression. What action should the nurse implement first? A. Recommend mental health counseling. B. Review the medications actions and interactions. C. Assess for the client's daily activity level. D. Provide information regarding a support group.

B. Alpha-interferon and ribavirin combination therapy can cause severe depression. (A, B, C) may be implemented after physiological aspect of the situation are assessed.

A 43-year-old homeless, malnourished female client with a history of alcoholism is transferred to the ICU. She is placed on telemetry, and the rhythm strip shown is obtained. The nurse palpates a heart rate of 160 beats/min, and the client's blood pressure is 90/54. Based on these finding, which IV medication should the nurse administer? A. Amiodarone (Cordarone) B. Magnesium sulfate C. Lidocaine (Xylocaine) D. Procainamide (Pronestyl)

B. Because the client has chronic alcoholism, she is likely to have hypomagnesium. (B) is the recommended drug for torsades de pointes (AHA, 2005), which is a form of polymorphic ventricular tachycardia (VT), usually associated with a prolonged QT interval that occurs with hypomagnesemia. (A and D) increase the QT interval, which can cause the torsades to worsen. (C) is the antiarrhythmic of choice in most cases of drug-induced monomorphic VT, not torsades.

A client with chronic asthma is admitted to postanesthesia complaining of pain at level 8 of 10, with a BP of 124/78, pulse of 88 beats/min, and respirations of 20 breaths/min. The postanesthesia recovery prescription is, "Morphine 2 to 4 mg IV push while in recovery for pain level over 5." What intervention should the nurse implement? A. Give the medication as prescribed to decrease the client's pain. B. Call the anesthesia provider for a different medication for pain. C. Use nonpharmacologic techniques before giving the medication. D. Reassess pain level in 30 Minutes and medicate if it remains elevated.

B. Call for a different medication because morphine and meperidine (Demerol) have histamine-releasing narcotics and should be avoided when a client has asthma. (A) puts the client at risk for asthma attack. (C & D) disregard the clients prescription and pain relief.

The nurse is reviewing the routine medications taken by a client with chronic angle closure glaucoma. Which medication prescription should the nurse question? A. An antianginal with a therapeutic effect of vasodilation. B. An anticholinergic with a side effect of pupillary dilation. C. An antihistamine with a side effect of sedation. D. A corticosteroid with a side effect of hyperglycemia.

B. Clients with angle closure glaucoma should not take medications that dilate the pupil (B) because this can precipitate acute and severely increased intraocular pressure. (A, C, D) do not cause increased intraocular pressure, which is the primary concern.

61. A nurse caring for a group of clients reviews the electrolyte laboratory results and notes a sodium level of 130 mEq/L on one client's laboratory report. The nurse understands that which client is at highest risk for the development of a sodium value at this level? A. The client with renal failure B. The client who is taking diuretics C. The client with hyperaldosteronism D. The client who is taking corticosteroids

B. Cllient taking diuretics ...Hyponatremia can occur in the client taking diuretics. The client taking corticosteroids and the client with renal failure or hyperaldosteronism are at risk for hypernatremia. Test-Taking Strategy: Use the process of elimination. First, determine that the client is experiencing hyponatremia. Next, you must know the causes of hyponatremia to direct you to option B. Review the normal serum sodium level and the causes of hyponatremia if you had difficulty with this question.

3. A client is admitted with diabetic ketoacidosis who, with treatment, has a normal blood glucose, pH, and serum osmolality. During assessment, the client complains of weakness in the legs. Which of the following is a priority nursing intervention? A. Request a physical therapy consult from the physician B. Ensure the client is safe from falls and check the most recent potassium level C. Allow uninterrupted rest periods throughout the day D. Encourage the client to increase intake of dairy products and green leafy vegetables.

B. Ensure the client is safe from falls and check the most recent potassium level (Correct Answer) Explanation In the treatment of diabetic ketoacidosis, the blood sugar is lowered, the pH is corrected, and potassium moves back into the cells, resulting in low serum potassium. Client safety and the correction of low potassium levels are a priority. The weakness in the legs is a clinical manifestation of the hypokalemia. Dairy products and green, leafy vegetables are a source of calcium.

Which of the following may be left in place when a patient is sent to the operating room? A. Wig B. Hearing aid C. Engagement ring D. Well-fitting dentures

B. Hearing aid If a patient is wearing a hearing aid, the perioperative nurse should be notified. Leaving the hearing aid in place enhances communication in the operating room. The nurse should make certain to record that the appliance is in place. Wigs, engagement rings, and dentures are not necessary items to facilitate quality patient care in the operating room.

A client diagnosed with chronic kidney disease (CDK) 2 years ago is regularly treated at a community hemodialysis facility. In assessing the client before his scheduled dialysis treatment, which electrolyte imbalance should the nurse anticipate? A. Hypophosphatemia B. Hypocalcemia C. Hyponatremia D. Hypokalemia

B. Hypocalcemia develops in CKD due to chronic hyperphosphatemia not (A). (C & D) incorrect you would find hypernatremia and hyperkalemia

Metformin (glucophage)

Biguinide antidiabetic

A 25-year-old client was admitted yesterday after a motor vehicle collision. Neurodiagnostic studies showed a basal skull fracture in the middle fossa. Assessment on admission revealed both halo and Battle signs. Which new symptom indicates that the client is likely to be experiencing a common life-threatening complication associated with basal skull fracture? A. Bilateral jugular vein distention. B. Oral temperature of 102 degrees F. C. Intermittent focal motor seizures. D. Intractable pain in the cervical region.

B. Increased temp indicates meningitis. (C & D) these symptoms may be exhibited but are not life threatening. (A) JVD is not a typical complication of basal skull fractures.

Which condition should the nurse anticipate as a potential problem in a female client with a neurogenic bladder? A. Stress incontinence. B. Infection. C. Painless, gross hematuria. D. Peritonitis.

B. Infection is the major complication resulting from stasis of urine and subsequent catheterization. (A) is the involuntary loss of urine through an intact urethra as a result of suddenly increased pressure. (C) is the most common symptom of bladder cancer. (D) is the most common and serious complication of peritoneal dialysis.

When assigning clients on a medical-surgical floor to a RN and a LPN, it is best for the charge nurse to assign which client to the LPN? A. A child with bacterial meningitis with recent seizures. B. An older adult client with pneumonia and viral meningitis. C. A female client in isolation wiht meningococcal meningitis. D. A male client 1 day post-op after drainage of a brain abscess.

B. Is the most stable. A, C, D have an increased risk for elevated ICP.

A 58-year-old client, who has no health problems, asks the nurse about taking the pneumococcal vaccine (Pneumovax). Which statement give by the nurse would offer the client accurate information about this vaccine? A. "The vaccine is given annually before the flue season to those over 50 years of age." B. "The immunization is administered once to older adults or persons with a history of chronic illness." C. "The vaccine is for all ages and is given primarily to those person traveling overseas to infected areas." D. "The vaccine will prevent the occurrence of pneumococcal pneumonia for up to 5 years."

B. It is usually recommended that persons over 65 years of age and those with a history of chronic illness should receive the vaccine once in a lifetime. (A) the influenza vaccine is given annually. (C) travel is not the main rationale for the vaccine. (D) The vaccine is usually given once in a lifetime.

Which abnormal lab finding indicates that a client with diabetes needs further evaluation for diabetic nephropathy? A. Hypokalemia B. Microalbuminauria C. Elevated serum lipids D. Ketonuria

B. Microalbuminuria is the earliest sign of nephropathy and indicates the need for follow-up evaluation. Hyperkalemia (A) is associated with end stage renal disease caused by diabetic nephropathy. (C) may be elevated in end stage renal disease. (D) may signal the onset of DKA.

The nurse would be alerted to the occurrence of malignant hyperthermia when the patient demonstrates: A. Hypocapnia B. Muscle rigidity C. Decreased body temperature D. Confusion upon arousal from anesthesia

B. Muscle rigidity Malignant hyperthermia is a metabolic disease characterized by hyperthermia with rigidity of skeletal muscles occurring secondary to exposure to certain anesthetic agents in susceptible patients. Hypoxemia, hypercarbia, and dysrhythmias may also be seen with this disorder.

If a 77-year-old patient who is NPO after surgery has dry oral mucous membranes, which of the following is the most appropriate nursing intervention? A. Increase oral fluid intake. B. Perform oral hygiene frequently. C. Swab the inside of the mouth with petroleum. D. Increase the rate of IV fluid administration.

B. Perform oral hygiene frequently. Frequent oral hygiene will help alleviate discomfort for a patient who is NPO. IV fluid rate is prescribed by the physician. Petroleum is always inappropriate intraorally. Oral fluid intake is contraindicated in a patient who is NPO.

A family member was taught to suction a client's tracheostomy prior to the client's discharge from the hospital. Which observation by the nurse indicates that the family member is capable of correctly performing the suctioning technique? A. Turns on the continuous wall suction to -190 mm Hg B. Inserts the catheter until resistance or coughing occurs C. Withdraws the catheter while maintaining suctioning D. Re-clears the tracheostomy after suctioning the mouth

B. indicates correct technique for performing suctioning. Suction pressure should be between -80 and -120 (A). The catheter should be withdrawn 1-2 cm at a time with intermittent suction (C). (D) introduces pathogens.

The nurse is caring for a client who has been in good health up to the present and is admitted with cellulitis of the hand. The client's serum potassium level was 4.5 mEq/L yesterday. Today the level is 7 mEq/L. Which of the following is the next appropriate nursing action? A. Call the physician and report results B. Question the results and redraw the specimen C. Encourage the client to increase the intake of bananas D. Initiate seizure precautions

B. Question the results and redraw the specimen (Correct Answer) Explanation A client who has been in good health up to the present is admitted for cellulitis of the hands. When the serum potassium goes from 4.5 mEq/L to 7.0 mEq/L with no risk factors for hyperkalemia, false high results should be suspected because of hemolysis of the specimen. The physician would likely question results as well. Bananas are a food high in potassium. Seizures are not a clinical manifestation of hyperkalemia.

An older female client with dementia is transferred from a long term care unit to an acute care unit. The client's children express concern that their mother's confusion is worsening. How should the nurse respond? A. "It is to be expected that older people will experience progressive confusion." B. "Confusion in an older person often follows relocation to new surroundings." C. "The dementia is progressing rapidly, but we will do everything we can to keep your mother safe." D. "The acute care staff is not as experienced as the long-term care staff at dealing with dementia."

B. Relocation often results in confusion among older clients and is stressful to clients of all ages. (A) is an inaccurate stereotype. (C) is most likely false there are many factors that cause increased temporary confusion. (D) may be true but does not offer the family a sense of security about the care.

Which of the following preoperative assessment findings should be reported to a surgeon for preoperative treatment? A. Serum sodium level of 140 mEq/L B. Serum potassium level of 3 mEq/L C. Hb concentration of 13.5 mg/dl D. Partial thromboplastin time of 25 seconds

B. Serum potassium level of 3 mEq/L. Electrolyte imbalances increase operative risk. Preoperative laboratory results should be checked to see whether they are within the normal range. The normal potassium level is 3.5 to 5.0 mEq/L. A low serum potassium level puts the patient at risk for cardiac dysrhythmias. A serum sodium level of 140 mEq/L is considered a normal value. An Hb concentration of 13.5 mg/dl is considered a normal value by most laboratory standards and does not interfere with operative decisions. A partial thromboplastin time of 25 seconds is a normal value and conducive to proceeding with a surgical procedure.

An older male client comes to the geriatric screening clinic complaining of pain in his left calf. The nurse notices a reddened area on the calf of his right leg that is warm to touch and the nurse suspects that the client may have thrombophlebitis. Which addition assessment is most important for the nurse to perform? A. Measure calf circumference. B. Auscultate the client's breath sounds. C. Observe for ecchymosis and petechiae. D. Obtain the client's blood pressure.

B. Since the client may have a pulmonary embolus secondary to the thrombophlebitis. A. Would support the nurses assessment. C. Least helpful since bruising is not associated with thrombophlebitis. D. Less important then auscultation.

Which of the following is most likely to be effective in meeting a patient's teaching/learning needs preoperatively? A. Teaching only the patient B. Teaching the patient and family C. Using brief verbal instructions D. Using only written instructions

B. Teaching the patient and family. A nurse should determine learning needs preoperatively and teach both the patient and the family before surgery. Using only written instructions does not provide the opportunity for evaluation for learning. Brief verbal instructions are often forgotten. Teaching only the patient limits learning preoperatively because the patient can be anxious and not receptive to new information.

The nurse assesses a postoperative client. Oxygen is being administered at 2 L/min and a saline lock is in place. Assessment shows cool, pale, moist skin. The client is very restless and has scant urine in the urinary drainage bag. What intervention should the nurse implement first. A. Measure urine specific gravity. B. Obtain IV fluids for infusion protocol. C. Prepare for insertion of a central venous catheter. D. Auscultate the client's breath sounds.

B. The client is at risk for hypovolemic shock and is exhibiting early signs. Start IV to restore tissue perfusion. (A, C, D) are all important but less of a priority.

A female client with a nasogastric tube attached to low suction states that she is nauseated. The nurse assesses that there has been no drainage through the nasogastric tube in the last 2 hours. Which action should the nurse take first? A. Irrigate the nasogastric tube with sterile normal saline. B. Reposition the client on her side. C. Advance the nasogastric tube 5 cm. D. Administer an intravenous antiemetic as prescribed.

B. The priority is to determined if the tube is functioning correctly, which would relieve the client's nausea. The least invasive intervention is to reposition the client (B), should be attempted first, followed by (A & C) if these are unsuccessful then (D).

A 55-year-old male client is admitted to the coronary care unit having suffered an acute myocardial infarction (MI). Within 24 hours of the occurrence, the nurse can expect to find which systemic sign? A. Elevated serum amylase level B. Elevated CM-MB level C. Prolonged prothrombin time (PT) D. Elevated serum BUN and creatinine

B. Tissue damage in the myocardium causes the release of cardiac enzymes into the blood system. An elevated CM-MB is a recognized indicator of an MI. It peaks 12 - 24 hours and returns to normal within 48 - 78 hours. (A) would indicate pancreatitis or a gastric disorder. (D) Although an elevated BUN might be related to an acute MI it is usually associated with dehydration, high protein intake or gastrointestinal bleeding and creatine levels indicate renal damage. (C) Indicates effective anticoagulation therapy.

Which description of symptoms is characteristic of a client with diagnosed with trigeminal neuralgia (tic douloureux)? A. Tinnitus, vertigo, and hearing difficulties. B. Sudden, stabbing, severe pain over the lip and chin. C. Unilateral facial weakness and paralysis. D. Difficulty in talking, chewing, and swallowing.

B. Trigeminal neuralgia is characterized by paroxysms of pain, similar to an electric shock, in the area innervated by one or more branches of the trigeminal nerve. A. Characteristic of Meniere's C. Characteristic of Bell palsey D. Characteristic of disorders of the hypoglossal (12th cranial nerve)

The nurse observes ventricular fibrillation on telemetry and upon entering the clients bathroom finds the client unconscious on the floor. What intervention should the nurse implement first? A. Administer an antidysrhythmic medication. B. Start cardiopulmonary resuscitation. C. Defibrillate the client at 200 joules. D. Assess the client's pulse oximetry.

B. Ventricular fibrillation is a life-threatening dysrhythmia and CPR should be started immediately. A & C are appropriate but B is the priority. D does not address the seriousness of the situation.

Debilitating anginal pain can be decreased in some clients by the administration of beta-blocking agents such as nadolol (Corgard). Which client requires the nurse to use extreme caution when administering Corgard? A. A 56-year-old air traffic controller who had bypass surgery 2 years ago. B. A 47-year-old kindergarten teacher diagnosed with asthma 40 years ago C. A 52-year-old unemployed stock broker who refuses treatment for alcoholism D. A 60-year-old retired librarian who takes a diuretic daily for hypertension.

B. asthma must be carefully monitored because beta blockers because it can induce cardiogenic shock and reduce bronchodilation efforts. (A & D) this medication is indicated and (C) it is not contraindicated.

A patient has the following preoperative medication order: morphine 10 mg with atropine 0.4 mg IM. The nurse informs the patient that this injection will A. decrease nausea and vomiting during and after surgery. B. decrease oral and respiratory secretions, thereby drying the mouth. C. decrease anxiety and produce amnesia of the preoperative period. D. induce sleep, so the patient will not be aware during transport to the operating room.

B. decrease oral and respiratory secretions, thereby drying the mouth. Atropine, an anticholinergic medication, is frequently used preoperatively to decrease oral and respiratory secretions during surgery, and the addition of morphine will help to relieve discomfort during the preoperative procedures. Antiemetics decrease nausea and vomiting during and after surgery, and scopolamine and some benzodiazepines induce amnesia. An actual sleep state is rarely induced by preoperative medications unless an anesthetic agent is administered before the patient is transported to the operating room.

Conscious sedation is being considered for a patient undergoing a cervical dilation and endometrial biopsy in the health care provider's office. The patient asks the nurse, "What is this conscious sedation?" The nurse's response is based on the knowledge that conscious sedation A. can be administered only by anesthesiologists or nurse anesthetists. B. enables the patient to respond to commands and accept painful procedures. C. is so safe that it can be administered by nurses with direction from health care providers. D. should never be used outside of the OR because of the risk of serious complications.

B. enables the patient to respond to commands and accept painful procedures. Conscious sedation is a moderate sedation that allows the patient to manage his or her own airway and respond to commands, and yet the patient can emotionally and physically accept painful procedures. Drugs are used to provide analgesia, relieve anxiety and/or provide amnesia. It can be administered by personnel other than anesthesiologists, but nurses should be specially trained in the techniques of conscious sedation to carry out this procedure because of the high risk of complications resulting in clinical emergencies.

Oral hypoglycemic agents INSULIN SENSITIZERS

BIGUANIDE- Decreases glucose production by liver; Increases glucose uptake by muscle EXAMPLES- metformin (Glocophage, Fortamet, Riomet, Glumetza) S/E- Nausea, diarrhea, decreased appetite, less likely to cause hypoglycemia than other agents NURSING IMPLICATIONS: -Give with meals -May enhance weight loss -Withhold if patient is having test with contrast dye -Contraindicated in renal and hepatic disease and heart failure -Monitor serum creatnine level -Notify physician of early symptoms of lactic acidosis; hyperventilation, myalgia, malaise

Lithium

BIPOLAR- MANIC/DEPRESSIVE mood stabilizer Lithium levels q 1-3 days initially, then 2-3 months; 0.6-1.5 mEq/L Acts like sodium in body: sweating, polyuria, diuretic-> Lithium toxicity Lithium toxicity with dehydration/ diarrhea/polyuria/hypothyroid

Lower GI Study

Barium is instilled in the rectum and x-rays are obtained.

Acoustic neuroma

Benign tumor of cranial nerve VIII.

Diabetic ketoacidosis or ketoacidosis is an acute complication risk for persons with type 1 DM if...

Blood glucose is uncontrolled and insulin is deficient

Before giving insulin, the nurse always checks which result?

Blood sugar level

Manifestations of Dilutional Hyponatremia

Bounding pulse Elevated or Normal BP Muscle weakness Headache Personality changes Nausea Diarrhea Convulsions Coma

Causes of Constipation

Bowel Obstruction, Fecal Impaction Chronic Laxative use, Ignoring the Urge Neurologic Disorders Side Effects of Drugs, Lack of Exercise, Fiber, Water

Symptoms of hypothyroidism

Bradycardia; cold intolerance; lethargy; memory loss; dry skin; brittle dry hair; decreased appetite; weight gain; constipation; increased serum lipids; decreased libido; erectile dysfunction

A patient enters a clinic with possible Cushing's syndrome. Which of the following physical examination findings support this diagnosis?

Buffalo hump and easy bruising

Side Effects of Dopamine Antagonist - Antiemetic

Butyrophenones - hypotension, EPS Metaclopramide - Sedation, diarrhea, EPS

pregnancy category of Prednisone

C

During a clinic visit 3 months following a diagnosis of type 2 diabetes, the patient reports following a reduced-calorie diet. The patient has not lost any weight and did not bring the glucose-monitoring record. The nurse will plan to obtain a(n) a. fasting blood glucose level. b. urine dipstick for glucose. c. glycosylated hemoglobin level. d. oral glucose tolerance test.

C Rationale: The glycosylated hemoglobin (Hb A1C) test shows the overall control of glucose over 90 to 120 days. A fasting blood level indicates only the glucose level at one time. Urine glucose testing is not an accurate reflection of blood glucose level and does not reflect the glucose over a prolonged time. Oral glucose tolerance testing is done to diagnose diabetes, but is not used for monitoring glucose control once diabetes has been diagnosed.

hyperactivity of the thyroid gland with sustained increase in synthesis and release of thyroid hormones

Hyperthyroidism

11. A client with a serum sodium of 115 mEq/L has been receiving 3% NS at 50 ml/hr for 16 hours. This morning the client feels tired and short of breath. Which of the following interventions is a priority? A. Turn down the infusion B. Check the latest sodium level C. Assess for signs of fluid overload D. Place a call to the physician

C Assess for signs of fluid overload Assessment should be done first

The healthcare provider prescribes naproxen (Naproxen) twice daily for a client with osteoarthritis of the hands. The client tells the nurse that the drug does not seem to be effective after three weeks. Which is the best response for the nurse to provide? A) The frequency of the dosing is necessary to increase the effectiveness. B) Therapeutic blood levels of this drug are reached in 4 to 6 weeks. C) Another type of nonsteroidal antiinflammatory drug may be indicated. D) Systemic corticosteroids are the next drugs of choice for pain relief

C) Another type of nonsteroidal antiinflammatory drug may be indicated.

The PACU has received a semiconscious patient from the operating room and reviews the chart for orders related to positioning of the patient. There are no specific orders on the chart related to specific orders for the patient's position. In this situation, in what position will the nurse place the patient? A. Trendelenburg position B. Prone position C. Side-lying position D. Supine position

C) If the patient is not fully conscious, place the patient in the side-lying position, unless there is an ordered position on the patient's chart.

A nurse administers naloxone (Narcan) to a postoperative patient experiencing respiratory sedation. What undesirable effect would the nurse anticipate after giving this medication? A) Drowsiness B) Tics and tremors C) Increased pain D) Nausea and vomiting

C) Increased pain Naloxone is a medication that reverses the effects of narcotics. Although the patient's respiratory status will improve after the administration of naloxone, pain will be more acute.

The nurse is preparing a patient for a colonoscopy. The nurse is familiar with the colonoscopy procedures at the hospital and is aware that which type of anesthesia os commonly used for his procedure? A. Spinal anesthesia B. Nerve block C. Conscious sedation D. Epidural anesthesia

C) Moderate sedation/analgesia is also known as conscious sedation or procedural sedation and is used for short-term and minimally invasive procedures such as a colonoscopy.

A patient is taking glipizide (Glucotrol) and a beta-adrenergic medication. A nurse is teaching hypoglycemia awareness and should tell the patient that which of these symptoms may not occur? A) Vomiting B) Muscle cramps C) Tachycardia D) Chills

C) Tachycardia Glipizide is a sulfonylurea oral hypoglycemic medication that acts to promote insulin release from the pancreas. Beta-adrenergic blockers can mask early signs of sympathetic system responses (most importantly, tachycardia) to hypoglycemia, which is the most common adverse effect of glipizide. Vomiting, muscle cramps, and chills are not symptoms of activation of the sympathetic nervous system that arise when glucose levels fall.

In the administration of a drug such as levothyroxine (Synthroid), the nurse should teach the client: A) That therapy typically lasts about 6 months. B) That weekly laboratory tests for T4 levels will be required. C) To report weight loss, anxiety, insomnia, and palpitations. D) That the drug may be taken every other day if diarrhea occurs.

C) To report weight loss, anxiety, insomnia, and palpitations.

What is the correct location for the placement of the hand for manual chest compressions during CPR on the adult client. A. Just above the xiphoid process on the upper third of the sternum. B. Below the xiphoid process midway between the sternum and the umbilicus. C. Just about the xiphoid process on the lower third of the sternum. D. Below the xiphoid process midway between the sternum and the first rib.

C.

Which statement should be stressed while giving instructions after adrenalectomy? A. Stop taking medication when pts physical appearance improves. B. Pt should take steroids on an empty stomach. C. Pt should take the prescribed medication as directed.

C. The pt should take prescribed medication as directed. Sudden withdrawl of steroids can precipitate adrenal crisis.

Which statement about diabetes mellitus is false? A. Type 2 diabetes commonly occurs in adults <40 yr. old. B. Type 1 diabetes usually occurs before age 30. C. Type 1 diabetes is treatable with exercise, meal planning, and antidiabetic drugs. D. An increasing number of adolescents are being diagnosed with type 2 diabetes.

C. Type 1 diabetes is treated with insulin and dietary management.

11. A nurse is preparing the client's morning NPH insulin dose and notices a clumpy precipitate inside the insulin vial. The nurse should: A. draw up and administer the dose B. shake the vial in an attempt to disperse the clumps C. draw the dose from a new vial D. warm the bottle under running water to dissolve the clump

C. The nurse should always inspect the vial of insulin before use for solution changes that may signify loss of potency. NPH insulin is normally uniformly cloudy. Clumping, frosting, and precipitates are signs of insulin damage. In this situation, because potency is questionable, it is safer to discard the vial and draw up the dose from a new vial.

Patients may experience which problem 24-48 hrs post-op as a result of anesthetics? A. colitis B. Stomatitis C. Paralytic ileus D. Gastrocolic reflux

C. After surgery, pts are clients are at risk for paralytic ileus as a result of anesthesia.

1. Which of the following statements made by a client taking phenytoin indicates understanding of the nurse's teaching? A. "I will increase the dose if my seizures don't stop." B. "I don't need to contact my health care provider before taking an over-the-counter cold remedy." C. "I will take good care of my teeth and see my dentist regularly." D. "I cannot take this drug with food."

C. "I will take good care of my teeth and see my dentist regularly."

The home health nurse is assessing a male client being treated for Parkinson disease with levodopa-carbidopa (Sinemet). The nurse observes that he does not demonstrate any apparent emotions when speaking and rarely blinks. Which intervention should the nurse implement? A. Perform a complete cranial nerve assessment. B. Instruct the client that he may be experiencing medication toxicity. C. Document the presence of these assessment findings. D. Advise the client to seek immediate medical evaluation.

C. A mask-like expression and infrequent blinking are common clinical features of Parkinsonism. The nurse should document the findings. (A & D) are not necessary. Signs of toxicity (B) are dyskinesia, hallucinations, and psychosis.

A central venous catheter has been inserted via a jugular vein and a radiography has confirmed placement of the catheter. A prescription has been received for stat medication but IV fluids have not yet been started. What action should the nurse take prior to administering the prescribed medication? A. Assess for signs of jugular vein distention. B. Obtain the needed intravenous solution. C. Administer a bolus of normal saline solution. D. Flush the line with heparinized saline.

C. A medication can be administered central line without IV fluids, flush with normal saline to remove heparin that may counteract with the medication. (B) is used following the medication and a second saline bolus. (A) will not impact the the med administration and is not a priority. (B) Administration of the stat medication is more of a priority than (B).

Before admitting a patient to the operating room, the nurse recognizes that which of the following must be attached to the chart of all patients? A. A functional status evaluation B. Renal and liver function tests C. A physical examination report D. An electrocardiogram

C. A physical examination report. It is essential to have a physical examination report attached to the chart of a patient going for surgery. This document explains in detail the overall status of the patient for the surgeon and other members of the surgical team.

What would be the most effective way for a nurse to validate "informed consent"? A. Ask the family whether the patient understands the procedure. B. Check the chart for a completed and signed consent form. C. Ask the patient what he or she understands regarding the procedure. D. Determine from the physician what was discussed with the patient.

C. Ask the patient what he or she understands regarding the procedure. Informed consent in the health care setting is a process whereby a patient is informed of the risks, benefits, and alternatives of a certain procedure, and then gives consent for it to be done. The piece of paper is simply evidence that the informed consent process has been done.

A client with heart failure is complaining of nausea. The client has received IV furosemide (Lasix), and the urine output has been 2500 ml over the past 12 hours. The client's home drugs include metoprolol (Lopressor), digoxin (Lanoxin), furosemide, and multivitamins. Which of the following are the appropriate nursing actions before administering the digoxin? Select all that apply. A. Administer an antiemetic prior to giving the digoxin B. Encourage the client to increase fluid intake C. Call the physician D. Report the urine output E. Report indications of nausea

C. Call the physician D. Report the urine output E. Report indications of nausea Explanation Potassium is lost during diuresis with a loop diuretic such as furosemide (Lasix). Hypokalemia can cause digitalis toxicity, which often results in nausea. The physician should be notified, and digoxin should be held until potassium levels and digoxin levels are checked. Peaked T waves and widened QRS are manifestations of hyperkalemia.

As the nurse is preparing a patient for surgery, the patient refuses to remove a wedding ring. Which of the following is the most appropriate action by the nurse? A. Note the presence of the ring in the nurse's notes of the chart. B. Insist the patient remove the ring. C. Explain that the hospital will not be responsible for the ring. D. Tape the ring securely to the finger.

C. Explain that the hospital will not be responsible for the ring. It is customary policy to tape a patient's wedding band to the finger and make a notation on the preoperative checklist that the ring is taped in place.

Laparoscopy

CO2 is injected into the peritoneal cavity, a small incision is made, fiberoptic laparoscope is interted. Direct visualization, biopsy may be taken, excision may be performed. General anesthesia, bowel decompression.

During the change of shift report, the charge nurse reviews the infusions being received by the clients on the oncology unit. The client receiving which infusion should be seen first?

C. Has the highest risk for respiratory depression and therefor should be seen first. (A) Risk of hypotension. (B) Lowest risk. (D) Risk of nephrotoxicity and phlebitis.

A patient has just been diagnosed with diabetes mellitus. His doctor has requested glucagon for emergency use at home. The nurse instructs the patient that the purpose of this drug is to treat: A. Hyperglycemia from insufficient insulin injection. B. Hyperglycemia from eating a large meal. C. Hypoglycemia from insulin overdose. D. Lipohypertrophy from inadequate insulin absorption.

C. Hypoglycemia from insulin overdose. Glucagon is for emergency use for insulin overdose. The patient will usually arouse within 20 minutes if unconscious. The family should also be instructed how to use the glucagon injection as well.

The nurse know that normal lab values expected for an adult may vary in an older client. Which data would the nurse expect to find when reviewing laboratory values of an 80-year-old man who is in good overall health. A. Complet blood count reveals increased WBC and decreased RBC counts. B. Chemistries reveal an increased serum bilirubin with slightly increased liver enzymes. C. Urinalysis reveals slight protein in the urine and bacteriuria with pyuria. D. Serum electrolytes reveal a decreased sodium level with an increased potassium level.

C. In older adults the protein found in urine is slightly risen as a result of kidney changes or subclinical UTIs and the client frequently experiences asymptomatic bacteriuria and pyuria as a result of incomplete bladder emptying. (A, B, D) are not normal findings.

Which of the following should be included in the plan of care for a patient who had spinal anesthesia? A. Elevating the head of the bed to decrease nausea B. Elevating the patient's feet to increase blood pressure C. Instructing the patient to remain flat in bed for 6 hours D. Administering oxygen to reduce hypoxia produced by spinal anesthesia

C. Instructing the patient to remain flat in bed for 6 hours. In addition to interventions designed to replace fluids and indirectly replace lost spinal fluid after administration of spinal anesthesia, the patient is instructed to lie flat for 6 to 8 hours. Elevating the head of the bed after spinal anesthesia can precipitate "spinal headache" or nausea related to losses of cerebrospinal fluid or changes in ICP. Elevating the patient's feet or administering oxygen are not necessary interventions unless the patient becomes hypotensive or hypoxic.

Which information should be given to the client taking phenytoin (Dilantin)? A. Taking the medication with meals will increase its effectiveness. B. The medication can cause sleep disturbances C. More frequent dental appointments will be needed for special gum care. D. The medication decreases the effects of oral contra- ceptives.

C. More frequent dental appointments will be needed for special gum care. Gingival hyperplasia is a side effect of phenytoin. The client will need more frequent dental visits. Answers A, B, and D do not apply to the medication; therefore, they are incorrect.

Five minutes after receiving a preoperative sedative medication by IV injection, a patient asks the nurse to get up to go to the bathroom to urinate. Which of the following is the most appropriate action for the nurse to take? A.) Assist patient to bathroom and stay next to door to assist patient back to bed when done. B. Allow patient to go to the bathroom since the onset of the medication will be more than 5 minutes. C. Offer the patient to use the urinal/bedpan after explaining the need to maintain safety. D. Ask patient to hold the urine for a short period of time since a urinary catheter will be placed in the operating room.

C. Offer the patient to use the urinal/bedpan after explaining the need to maintain safety. The prime issue after administration of either sedative or opioid analgesic medications is safety. Because the medications affect the central nervous system, the patient is at risk for falls and should not be allowed out of bed, even with assistance.

The nurse is completing an admission inter for a client with Parkinson disease. Which question will provide addition information about manifestations the client is likely to experience? A. "Have you ever experienced and paralysis of your arms or legs?" B. " Do you have frequent blackout spells?" C. "Have you ever been 'frozen' in one spot, unable to move?" D. "Do you have headaches, especially ones with throbbing pain?"

C. Parkinson clients frequently experience difficulty in initiating, maintaining, and performing motor activities. They may even experience being rooted, unable to move. (A, B, D) Does not typically occur in Parkinson.

18. A client with pneumonia presents with the following arterial blood gases: pH of 7.28, PaCO2 of 74, HCO3 of 28 mEq/L, and PO2 of 45, which of the following is the most appropriate nursing intervention? A. Administer a sedative B. Place client in left lateral position C. Place client in high-Fowler's position D. Assist the client to breathe into a paper bag

C. Place client in high-Fowler's position

Which of the following is most appropriate after administration of preoperative medications? A. Confirming that the patient has voided B. Monitoring vital signs every 15 minutes C. Placing the patient in bed with the rails up D. Transporting the patient immediately to the operating room

C. Placing the patient in bed with the rails up. After administration of preoperative medications, a nurse should instruct a patient not to get up without assistance because medications can cause drowsiness or dizziness. Confirming that the patient has recently voided should be done before preoperative medications are administered. Monitoring vital signs every 15 minutes is not a necessary intervention unless prescribed by the physician. Transporting the patient immediately to the operating room is not necessary unless the patient is called for.

While performing preoperative teaching, the patient asks when she needs to stop drinking water before the surgery. Based on the most recent practice guidelines established by the American Society of Anesthesiologists, the nurse tells the patient that: A. She must be NPO after breakfast. B. She needs to be NPO after midnight. C. She can drink clear liquids up to 2 hours before surgery. D. She can drink clear liquids up until she is taken to the OR.

C. She can drink clear liquids up to 2 hours before surgery. Practice guidelines for preoperative fasting state the minimum fasting period for clear liquids is 2 hours. Evidenced-based practice no longer supports the long-standing practice of requiring patients to be NPO after midnight.

The nurse is caring for a critically ill client with cirrhosis of the liver who has a nasogastric tube draining bright red blood. The nurse notes that the client's serum hemoglobin and hematocrit are decreased. What additional change in lab data should the nurse expect? A. Increased serum albumin B. Decreased serum creatinine C. Decreased serum ammonia D. Increased liver function tests

C. The breakdown of glutamine in the intestine and the increased activity of colonic bacteria from the digestion of proteins increases the ammonia levels in the clients with advanced liver disease, so removal of blood, a protein source, from the intestines results in reduced ammonia. (A, B, D) will not be significantly impacted by the removal of blood.

What is the correct procedure for performing an ophthalmoscopic examination on a client's right eye? A. Instruct the client to look at the examiner's nose and not move his/her eyes during the exam. B. Set ophthalmoscope on the plus 2 to 3 lens and hold it in front of the examiner's right eye. C. From a distance of 8 to 12 inches and slightly to the side, shine the light into the client's pupil. D. For optimum visualization, keep the ophthalmoscope at least 3 inches for the client's eye

C. The client should focus on a distant object in order to promote pupil dilation. The ophthalmoscope should be set on the 0 lens to begin (creates no correction) and should be held in front of the examiner's left eye when examining the client's right eye and kept 1" from the client's eye for optimum visualization. (A, B, D) are incorrect procedures.

During report, the nurse learns that a client with tumor lysis syndrome is receiving an IV infusion containing insulin. Which assessment should the nurse complete first? A. Review the client's history for diabetes mellitus. B. Observe the extremity distal to the IV site. C. Monitor the client's serum potassium and blood glucose. D. Evaluate the client's oxygen saturation and breath sounds.

C. The client with tumor lysis syndrome may experience hyperkalemia, therefor it is important to monitor serum potassium and blood glucose levels. (A, B, D) are not as priority.

4. A client with a potassium level of 5.5 mEq/L is to receive sodium polystyrene sulfonate (Kayexalate) orally. After administering the drug, the priority nursing action is to monitor A. urine output. B. blood pressure. C. bowel movements. D. ECG for tall, peaked T waves.

C. bowel movements. (Correct Answer) Explanation Kayexalate causes potassium to be exchanged for sodium in the intestines and excreted through bowel movements. If client does not have stools, the drug cannot work properly. Blood pressure and urine output are not of primary importance. The nurse would already expect changes in T waves with hyperkalemia. Normal serum potassium is 3.5 to 5.5 mEq/L

In the absence of postoperative vomiting, GI suctioning, and wound drainage, the physiologic responses to the stress of surgery are most likely to cause A. diuresis. B. hyperkalemia. C. fluid overload. D. impaired blood coagulation.

C. fluid overload. Secretion and release of aldosterone and cortisol from the adrenal gland and ADH from the posterior pituitary as a result of the stress response cause fluid retention during the first 2 to 5 days postoperatively, and fluid overload is possible during this time. Aldosterone causes renal potassium loss with possible hypokalemia, and blood coagulation is enhanced by cortisol.

The nurse is creating a pain management plan for a client with a previous history of substance abuse. Which of the following should be included in this plan? a.) Ask the physician to prescribe short-acting analgesics. b.) Ask the physician to prescribe a medication similar to the one the client abused. c.) Ask the physician to prescribe all analgesics for the oral route. d.) Keep a dose of Narcan at the bedside.

C.) Ask the physician to prescribe all analgesics for the oral route. Extended-release and long-acting analgesics are recommended for clients with a history of abuse. Specific interventions should avoid analgesics similar to the abused drug, utilize long-acting analgesics, avoid Narcan, and administer medications through the oral route.

diltiazem/ Cardizem

CALCIUM CHANNEL BLOCKERS Dilates CORONARY ARTERIES; affects calcium channels in both heart and blood vessels; also for angina and specific dysrhythmias

nifedipine/ Adalat, Procardia

CALCIUM CHANNEL BLOCKERS Selective for calcium channels in blood vessels; decreased peripheral vascular resistance and increases cardiac output; also for angina

amlodipine/ Norvasc

CALCIUM CHANNEL BLOCKERS Works primarily on PERIPHERAL circulation; reduces systolic, diastolic, and mean blood pressure; also for angina

verapamil/ Calan, Isoptin

CALCIUM CHANNEL BLOCKERS affects calcium channels in both heart and blood vessels; available for specific dysrhythmias, angina and migraines

digoxin/ Lanoxin

CARDIAC GLYCOSIDE heart failure, increases cardiac outputl; larger dose is given to initiate therapy; alsoused for dysrhythmias

clonidine/ Catapres

CENTRAL-ACTING AGENT central-acting alpha2-adrenergic agent, hypertension

Xanax/ Ativan

CNS DEPRESSANTS BENZODIAZAPINE (more) Short Rx slow neutral activity from relaxation to sedation to anesthesia agitation

Side Effects of Opiates and Opiate Related Drugs - Antidiarrheal

CNS depression, constipation, decreased GI motility. Decreases SNS and respiratory Drive

Side Effects of Pepsin Inhibitors - Anti Ulcer

Can cause constipation (non-absorbable) pooped out

What are some side effects of Metformin, Glucaphage?

Can cause lactic acidosis in patients with renal insufficiency Teach patient to report: unusual muscle pain, fatigue, shortness of breath, dizziness, irregular heart beats

Treatment for hyperthyroidism

Can't treat with meds; kill it with radiotherapy or excision

lesion of the hypothalamus, infundibular stem, or posterior pitutiary interfers with ADH synthesis, transport, or release; most common form of DI

Central (neurogenic) DI

Meniere's disease

Chronic disorder of unknown cause characterized by recurrent attacks of vertigo with tinnitus and a progressive unilateral hearing loss.

Example of Histamine 2 Blocker - Anti Ulcer

Cimetidine, Famotidine, Nizatidine, Ranitidine

aricept

Class: ACHE inhibitor (alzheimer's). Generic: donepezil. Dosage: 5-10 mg/hs (bedtime). Route: PO. Action: inhibits ACHE, increase ACh

zantac

Class: Antiulcer: hist 2 blocker. Generic: ranitidine. Dosage: 150 mg PO, 50 mg IV/IM. Action: inhibit gastric secretion by inhibiting hist2 receptors. Contra: hypersensitivity, renal/liver disease. Side effects: confusion, vertigo, depression, GI upset. Nurse: monitor electrolytes; urinary PH, Ca+ P. Teaching: report pain, bleeding; drink 2 o2 H20 after taking drug, take 1-3 hrs after meals and bedtime, avoid food with increased vitamin D, decrease stress level.

adderall

Class: amphetamines. Generic: amp. sulfate. Dosage: Narcolepsy (5-60 mg/d) ADHD (5mg/d).

heparin

Class: anticoagulant Dose: A: subQ: 5000 U q8-12h; IV: bolus: 5000 u; inf: 20,000-40,000 U over 24 h based on APTT C: IV: 50 U/kg bolus; then 5-100 units/kg q4h based on APTT Action: inhibit thrombin: prevents convert of fibrinogen to fibrin Contra: bleeding disorder, peptic ulcer, severe hepatic/renal disease, hemophila, CVA sides: itching, burning; Adverse: bleeding, ecchymoses, hemorrhage Antidote: protamine sulfate

lipitor

Class: antihyperlipidemic. Generic: atirvastatin. Dosage: 10-40mg/d. Route: PO. Contra: liver disease, pregnant. Action: inhibits HMG-Co-A reductase, enzyme needed for hepatic production of cholesterol. Side effects: headache, rash, constipation. Nurse: patient fasts 12-14 hrs before taking chol. level, normal cholesterol < 200 mg/dL, hyperlipid > 240 mg/dL. Teaching: report visual changes, monitor BGL, take nicotinic acid with meals, increase fluid intake, decrease fat diet.

cytotec

Class: antiulcer. Generic: misoprostol. Dosage: 100-200 mcg qid. Route: PO. Action: prevention of NSAID-induced gastric ulcer. Side effects: GI upset

demerol

Class: opioid. Generic: meperidine. Dosage: 50-150 mg PRN. Route: PO/subQ/IM/IV. Schedule II, treats GI pain, mod-severe pain. Contra: chronic pain, liver, dysfunction, sickle cell, seizures, CAD, dysrhythmias. Side effects: decrease BP. Teaching: don't take with a/c or sedative-hypnotics.

Atropine

Class: parasympatholytic Dose: A: PO/IM/IV: 0.4-0.6 mg q4-6h PRN C: PO/IM/IV: 0.01 mg/kg/dose; max: 0.4 mg/dose, q4-6h PRN Action: inhibit ACh by occupying receptors; up HR by blocking vagus stim; promote dilation of pupils by blocking iris sphincter muscle Contra: narrow-angle glaucoma, obstructive GI disorder, paralytic ileus, ulcerative colitis, tachycardia, benign prostate hypertrophy, myasthenia gravis, myocardial ischemia Caution: renal/hepatic disorder, COPD, heart fail Side: dry mouth, nausea, headache, constipation, dry skin, flushing, mydriasis, blurred vision, photophobia, abdom distension, palpitations, urinary retention Adverse: tachycardia, hypotension, paralytic ileus, coma, V-fib

Examples of Emollients (Stool Softeners)

Colace Peri-colace

What is a symptom of hypoglycemia?

Cold sweats and Tremors

ERCP

Combination of endoscope and x-rays to view biliary, pancreatic, and hepatic ducts.

Pepsin Inhibitor - Mucosal Protective Drug - Anti Ulcer

Combines with protein to form a viscous substance that covers the ulcer and protects it from HCl and Pepsin. Does not neutralize acid or decrease acid secretions (Just covers)

glaucoma

Condition characterized by increased intraocular pressure of the eye and a gradual loss of vision.

Chronic otitis media

Condition involving permanent perforation of the tympanic membrane, with or without recurrent pus formation and often accompanied by changes in the mucosa and bony structures (ossicles) of the middle ear.

Side Effects of Non-Systemic Antacids

Constipation for Aluminum Diarrhea for Magnesium

Upper GI study: Upper GI flouroscopy

Contrast is introduced and fluoroscopy is used to examine upper GI structures.

Lysodren (mitotane)

Cortisol-inhibiting / Adrenal cytotoxic agent

excess of corticosteroids, particuarly glucocorticoids

Cushing syndrome

Which disorder results from too much cortisol secretion?

Cushing's syndrome (regardless of exogenous or endogenous, it is caused by excess cortisol)

A patient screened for diabetes at a clinic has a fasting plasma glucose level of 120 mg/dl (6.7 mmol/L). The nurse will plan to teach the patient about a. use of low doses of regular insulin. b. self-monitoring of blood glucose. c. oral hypoglycemic medications. d. maintenance of a healthy weight.

D Rationale: The patient's impaired fasting glucose indicates prediabetes and the patient should be counseled about lifestyle changes to prevent the development of type 2 diabetes. The patient with prediabetes does not require insulin or the oral hypoglycemics for glucose control and does not need to self-monitor blood glucose.

A college student who has type 1 diabetes normally walks each evening as part of an exercise regimen. The student now plans to take a swimming class every day at 1:00 PM. The clinic nurse teaches the patient to a. delay eating the noon meal until after the swimming class. b. increase the morning dose of neutral protamine Hagedorn (NPH) insulin on days of the swimming class. c. time the morning insulin injection so that the peak occurs while swimming. d. check glucose level before, during, and after swimming.

D Rationale: The change in exercise will affect blood glucose, and the patient will need to monitor glucose carefully to determine the need for changes in diet and insulin administration. Because exercise tends to decrease blood glucose, patients are advised to eat before exercising. Increasing the morning NPH or timing the insulin to peak during exercise may lead to hypoglycemia, especially with the increased exercise.

A type I diabetic patient comes to the clinic for a follow-up appointment. The patient is taking NPH insulin, 30 units every day. A nurse notes that the patient is also taking metoprolol (Lopressor). What education should the nurse provide to the patient? A) "You need to increase your insulin to allow for the agonist effects of metoprolol." B) "Metoprolol may potentiate the effects of the insulin, so the dose should be reduced." C) "Metoprolol has no effects on diabetes mellitus or on your insulin requirements." D) "Metoprolol may mask signs of hypoglycemia, so you need to monitor your blood glucose closely."

D) "Metoprolol may mask signs of hypoglycemia, so you need to monitor your blood glucose closely."

Upon admission for an appendectomy, the patient provides the nurse with a document that specifies instructions his healthcare team should follow in the event he is unable to communicate these wishes postoperatively. This document is best known as: A. An informed consent B. An insurance card C. A Patient's Bill of Rights D. An advance directive

D) An advance directive, a legal document, allows the patient to specify instructions for his or her healthcare treatment should he or she be unable to communicate these wishes postoperatively. The advance directive allows the patient to discuss his or her wishes with the family members in advance of the surgery. Two common forms of advance directives include living wills and durable powers of attorney for healthcare.

The nurse is assessing a patient receiving valproic acid (Depakene) for potential adverse effects associated with this drug. Which item represents the most common problem with this drug? A) Increased risk for infection B) Reddened, swollen gums C) Nausea, vomiting, and indigestion D) Central nervous system depression

D) Central nervous system depression Valproic acid is generally well tolerated. It does not cause hematologic effects resulting in increased risk for infection nor does it cause gingival hyperplasia. It causes minimal sedation. Gastrointestinal effects, which include nausea, vomiting, and indigestion, are the most common problems but tend to subside with use and can be lessened by giving with food.

A patient takes oxycodone (OxyContin), 40 mg PO twice daily, for the management of chronic pain. Which intervention should be added to the plan of care to minimize the gastrointestinal adverse effects? A) Take an antacid with each dose. B) Eat foods high in lactobacilli. C) Take the medication on an empty stomach. D) Increase fluid and fiber in the diet.

D) Increase fluid and fiber in the diet. Narcotic analgesics reduce intestinal motility, leading to constipation. Increasing fluid and fiber in the diet can help manage this adverse effect.

A patient who is newly diagnosed with type 1 diabetes asks a nurse, "How does insulin normally work in my body?" The nurse explains that normal insulin has which of these actions in the body? A) Stimulates the pancreas to reabsorb glucose B) Promotes synthesis of amino acids into glucose C) Stimulates the liver to convert glycogen to glucose D) Promotes the passage of glucose into cells for energy

D) Promotes the passage of glucose into cells for energy Insulin is a hormone that promotes the passage of glucose into cells, where it is metabolized for energy. Insulin does not stimulate the pancreas to reabsorb glucose or synthesize amino acids into glucose. It does not stimulate the liver to convert glycogen into glucose.

A nurse is assessing a pt with a closed chest tube drainage system connected to suction. Which finding would require additional evaluation in the post-operative period? A. 75ml of bright red drainage in the system. B. A column of water 20cm high in the suction control chamber. C. Constant bubbling in the water seal chamber.

D. Constant bubbling in the water seal chamber is indicative of an air leak. The nurse should assess the entire system to the pt to find the sourse of he leak. The leak may be with in the pts chest or at the insertion site. If it is, notify physician. This could cause the lung to collapse due to a buildup of air pressure within the plural cavity.

An adrenal crisis is characterized by all of the signs and symptoms except: A. weakness and fatigue B. nausea & vomiting C. hypotension D. sodium & fluid retention

D. Sodium and fluid retention are characteristics of Cushing's Syndrome. Adrenal crisis causes decreased sodium levels and hypotension.

The nurse know that a client taking diuretics must be assessed for the development of hypokalemia, and that hypokalemia will create changes in the client's normal ECG tracing. Which ECG change would be an expected finding in the client with hypokalemia? A. Tall, spiked T waves B. A prolonged QT interval C. A widening QRS complex D. Presence of a U wave

D. A U wave is a positive deflection following the T wave and is often present with hypokalemia. A, B, C indicate hyperkalemia.

While a nurse is caring for a patient who is scheduled to have surgery in 2 hours, the patient states, "My doctor was here and told me a lot of stuff I didn't understand and then I signed a paper for her." To fulfill the role of advocate, what is the best nursing action? A. Reassure the patient that the surgery will go as planned. B. Explain the surgery and possible outcomes to the patient. C. Complete her first priority, the preoperative teaching plan. D. Call the physician to return and clarify information for the patient.

D. Call the physician to return and clarify information for the patient. Examples of nursing advocacy include questioning doctors' orders, promoting patient comfort, and supporting patient decisions regarding health care choices.

A nurse has requested and gotten permission to observe a surgical procedure of interest in the hospital in which the nurse is employed. While the patient is being draped, the nurse notices that a break in sterile technique occurs. Which of the following actions on the nurse's part is most appropriate? A. Tell the surgeon before an incision is made. B. Tell the circulating nurse at the end of the surgery. C. Say nothing because someone else is likely to notice also. D. Point out the observation immediately to the personnel involved.

D. Point out the observation immediately to the personnel involved. Any break in sterile technique in the operating room should be immediately pointed out and remedied.

7. A client with chronic renal failure reports a 10 pound weight loss over 3 months and has had difficulty taking calcium supplements. The total calcium is 6.9 mg/dl. Which of the following would be the first nursing action? A. Assess for depressed deep tendon reflexes B. Call the physician to report calcium level C. Place an intravenous catheter in anticipation of administering calcium gluconate D. Check to see if a serum albumin level is available

D. Check to see if a serum albumin level is available (Correct Answer) Explanation A client with chronic renal failure who reports a 10 pound weight loss over 3 months and has difficulty taking calcium supplements is poorly nourished and likely to have hypoalbuminemia. A drop in serum albumin will result in a false low total calcium level. Placing an IV is not a priority action. Depressed reflexes are a sign of hypercalcemia. Normal serum calcium is 9 to 10.5 mg/dl.

A client who is receiving an ACE inhibitor for hypertension calls the clinic and reports the recent onset of a cough to the nurse. What action should the nurse implement? A. Advise the client to come to the clinic immediately for further assessment. B. Instruct the client to discontinue use of the drug, and make an appointment at the clinic. C. Suggest that the client lear to accept the cough as a side effect to a necessary prescription. D. Encourage the client to keep taking the drug until seen by the HCP.

D. Cough is a common s/e of ACE inhibitors and is not an indication to discontinue the medication. (A) immediate evaluation is not needed. (B) an antihypertensive should not be stopped abruptly. (C) is demeaning since the cough may be disruptive to the client and other medications may produce results without the s/e.

The nurse is assessing a client who presents with jaundice. Which assessment finding is the most significant indication that further follow up is needed? A. Urine specific gravity of 1.03 with a urine output of 500 ml in 8 hours B. Frothy, tea-colored urine C. Clay-colored stools and complaints of pruritus D. Serum amylase and lipase levels that are twice their normal levels

D. Obstructive cholelithiasis and alcoholism are the two major causes of pancreatitis, and an elevated serum amylase and lipase indicate pancreatic injury. (A) is a normal finding. (B & C) are expected findings for jaundice.

The nurse plans to help an 18-year-old developmentally disabled female client ambulate on the first postoperative day. When the nurse tells her it is time to get out of bed, the client becomes angry and yells at the nurse. "Get out of here! I'll get up when I'm ready." Which response should the nurse provide? A. "Your healthcare provider has prescribed ambulation on the first postoperative day." B. "You must ambulate to avoid serious complications that are much more painful." C. "I know how you feel; you're angry about having to do this, but it is required." D. "I'll be back in 30 minutes to help you get out of bed and walk around the room."

D. Returning in 30 minutes provides a cooling off period, is firm, direct, nonthreatening, and avoids argument with the client. B is threatening. C. assumes what the client is feeling. A. avoids the nurse's responsibility to ambulate the client.

The nurse is assessing a 75-year-old male client for symptoms of hyperglycemia. Which symptom of hyperglycemia is an older adult most likely to exhibit? A. Polyuria B. Polydipsia C. Weight loss D. Infection

D. S/Sx of hyperglycemia in older adults may include fatigue, infection, and neuropathy (such as sensory changes). (A, B, C) are classic symptoms and may be absent in the older adult.

A client with alcohol-related liver disease is admitted to the unit. Which prescription should the nurse question as possibly inappropriate for the client? A. Vitamin K1 (AquaMEPHYTON) 5 mg IM daily B. High-calorie, low-sodium diet C. Fluid restriction to 1500 ml/day D. Pentobarbital (Nembutal sodium) 50 mg at bedtime for rest

D. Sedatives such as Nembutal are contraindicated for clients with liver damage and can have dangerous consequences. (A) is often prescribed since normal clotting mechanism is damaged. (B) is needed to restore energy. (C) Fluids are restricted to decrease ascites which often accompanies cirrhosis, particularly in later stages of the disease.

The nurse witnesses a baseball player receive a blunt trauma to the back of the head with a softball. What assessment data should the nurse collect immediately? A. Reactivity of deep tendon reflexes, comparing upper to lower extremities. B. Vital signs readings, excluding blood pressure if need equipment is unavailable. C. Memory of events that occurred before and after the blow to the head. D. Ability to spontaneously open the eyes before any tactile stimuli are given.

D. The LOC should be immediately established immediately after the head injury has occurred. Spontaneous eye opening (D) is a simple measure of LOC. (A) is not the best indicator of LOC. (B) is important but not the best indicator of LOC. (C) can be assessed after LOC has been established by assessing eye opening.

Twelve hours after chest tube insertion for hemothorax, the nurse notes that the client's drainage has decreased from 50 ml/hr to 5 ml/hr. What is the best inital action for the nurse to take? A. Document this expected decrease in drainage. B. Clamp the chest tube while assessing for air leaks. C. Milk the tube to remove any excessive blood clot build up. D. Assess for kinks or dependent loops in the tubing.

D. The least invasive action should be performed to assess the decrease in drainage. (A) is completed after assessing for and problems causing the decreased drainage. (B) is no longer protocol because the increased pressure may be harmful for the client. (C) is an appropriate nursing action after the tube has been assessed for kinks or dependent loops.

18. A nurse is administering blood to a patient who has a low hemoglobin count. The patient asks how long to RBC's last in my body? The correct response is. A. The life span of RBC is 45 days. B. The life span of RBC is 60 days. C. The life span of RBC is 90 days. D. The life span of RBC is 120 days.

D. The life span of RBC is 120 days.

An older client is admitted with a diagnosis of bacterial pneumonia. The nurse's assessment of the client will most likely reveal which S/SX? A. Leukocytosis and febrile. B. Polycythemia and crackles. C. Pharyngitis and sputum production. D. Confusion and tachycardia.

D. The onset of pneumonia is the older may be signaled by general deterioration, confusion, increased heart rate or increased respiratory rate. (A, B, C) are often absent in the older with bacterial pneumonia.

Which of the following regimens offers the best blood glucose control for persons with type 1 diabetes? a. A single anti-diabetes drugs b. Once daily insulin injections c. A combination of oral anti-diabetic medications d. Three or four injections per day of different types of insulin.

D. Three or four injections per day of different types of insulin. Because persons with type 1 diabetes do not produce insulin, they require insulin and cannot be treated with oral anti-diabetic drugs. Several injections of insulin per day, calibrated to respond to measured blood glucose levels, offer the best blood glucose control and may prevent or postpone the retinal, renal, and neurological complications of diabetes.

hydrochlorothiazide/ Microzide

DIURETIC thiazide type acts by inhibiting sodium reabsorption in the distal tubule; decreases blood potassium levels

A physician is performing a sterile procedure at a patient's bedside. Near the end of the procedure, the nurse thinks that the physician has contaminated a sterile glove and the sterile field. The nurse should A. report the physician for violating surgical asepsis and endangering the patient. B. not say anything, because the nurse is not sure that the gloves and field were contaminated. C. ask the physician whether the contaminated glove and the sterile field have been contaminated. D. point out the possible break in surgical asepsis and provide another set of sterile gloves and fresh sterile field.

D. point out the possible break in surgical asepsis and provide another set of sterile gloves and fresh sterile field. It is the responsibility of the nurse to point out any possible break in surgical asepsis when others are unaware that they have contaminated the field. Reporting the physician is not indicated, nor does it protect the patient. Asking the physician may lead to infection if the physician is unaware of the break in technique that the nurse believes may have happened. Saying nothing does not protect the patient and is negligence on the part of the nurse.

To prevent airway obstruction in the postoperative patient who is unconscious or semiconscious, the nurse A. encourages deep breathing. B. elevates the head of the bed. C. administers oxygen per mask. D. positions the patient in a side-lying position.

D. positions the patient in a side-lying position. An unconscious or semiconscious patient should be placed in a lateral position to protect the airway from obstruction by the tongue. Deep breathing and elevation of the head of the bed are implemented to facilitate gas exchange when the patient is responsive. Oxygen administration is often used, but the patient must first have a patent airway.

A client in the ICU who sustained a traumatic abdominal injury 1 week ago continues to complain of severe pain. The nurse notes his vital signs are normal. Which of the following would be appropriate for the nurse to do? a.) Encourage early return to ambulation. b.) Offer nonnarcotic analgesics for pain. c.) Utilize distraction d.) Provide the client with pain medication.

D.) Provide the client with pain medication.

hhydralazine with isosorbide dinitrate/BiDil

DIRECT-ACTING VASODILATOR heart failure, increases hear rate and cardiac output; decreases myocardial oxygen consumption; hydralazine also for HTN and isosorbide dinitrate for angina

hydralazine/ Apresoline

DIRECT-ACTING VASODILATOR hypertension, diastolic response usually greater than systolic

HYPOFUNCTION of Adrenal Cortex Hormone

DISORDER- Adrenocortical insufficiency, Addison's disease SIGNS AND SYMPTOMS- Sodium and water loss, hypotension, hypoglycemia, fatigue DIAGNOSTIC TESTS- Serum and Urine Cortisol THERAPEUTIC MEASURES- Glucocorticoid and mineralocorticoid replacement PRIORITY NURSING DIAGNOSIS- Risk for Deficient Fluid Volume

HYPERFUNCTION of Adrenal Cortex Hormone

DISORDER- Cushing's syndrome SIGNS AND SYMPTOMS- Weight gain, sodium and water retention, hyperglycemia, buffalo hump, moon face DIAGNOSTIC TESTS- Serum and Urine Cortisol THERAPEUTIC MEASURES- Alter steroid therapy dose or schedule; surgery if tumor PRIORITY NURSING DIAGNOSIS- Risk for Excess Fluid Volume, Unstable Blood Glucose Level, Infection

Insufficient ADH

DISORDER- Diabetes Insipidus SIGNS AND SYMPTOMS-Polyuria, polydipsia, dehydration, dilute urine DIAGNOSTIC TESTS-Urine specific gravity, urine and plasma osmolality, water deprivation test THERAPEUTIC MEASURES- Synthetic ADH replacement PRIORITY NURSING DIAGNOSES- Deficient fluid volume, Risk for Ineffective Self Health Management

Excess PTH

DISORDER- Hyperparathyroidism SIGNS AND SYMPTOMS- Hypercalcemia, fatigue, pathological fractures DIAGNOSTIC TESTS- Serum PTH, calcium and phosphate THERAPEUTIC MEASURES- Calcitonin, parathyroidectomy. PRIORITY NURSING DIAGNOSIS- Risk for Injury related to bone demineralization

Insufficient PTH

DISORDER- Hypoparathyroidism SIGNS AND SYMPTOMS- Hypocalcemia, neuromuscular irritability, tetany, positive Chvostek's and Trousseau's signs DIAGNOSTIC TESTS- Serum PTH, calcium and phosphate THERAPEUTIC MEASURES- Calcium and Vitamin D replacement; High calcium, low phosphorous diet PRIORITY NURSING DIAGNOSIS- Risk for Injury related to tetany

Excess ADH

DISORDER- SIADH SIGNS AND SYMPTOMS-Fluid retention, weight gain, concentrated urine DIAGNOSTIC TESTS-Serum and urine sodium osmolality; water load tes THERAPEUTIC MEASURES- Treat cause PRIORITY NURSING DIAGNOSES- Excess fluid volume, Risk for Ineffective Self Health Management

torsemide/ Demadex

DIURETIC loop diuretic acts by inhibiting sodium and chloride reabsorption in the look of Henle and distal tubule

furosemide/ Lasix

DIURETIC loop diuretic decreased blood potassium levels; acts by inhibiting sodium and chloride reabsorption in the loop of Henle

spironolactone/ Aldactone

DIURETIC potassium-sparing acts by inhibiting aldosterone in the distal tubule

Oral hypoglycemic agents ABSORPTION DELAYERS

DPP-4 INHIBITOR - Inhibits DPP-4, an enzyme that breaks incretins (hormones secreted by GI in response to food), they reduce glucogon secretion and increase insulin synthesis and release EXAMPLES- sitagliptin (Januvia) S/E- Upper respiratory infection, headache; hypoglycemia when administered in combination with a sulfonylurea NURSING IMPLICATIONS: -Administer one a day -Only works when blood glucose is high, so does not cause hypoglycemia when used alone -Watch for allergic reactions

atropine sulfate/ Isopto Atropine, others

DRUGS FOR EYE ECAMINATIONS AND MOISTENING EYE MEDICATIONS CYCLOPLEGICS; ANTICHOLINERGICS Also provided as ointment; should not be administered to patients with glaucoma; effects may be prolonged

Examples of Glucocorticoids - Antiemetic

Decadron Solu-Medrol

Opiates and Opiate Related Drugs - Antidiarrheal

Decreases GI motility, and therefore peristalsis Atrophine decreases cramping, hyper-secretions, and intestinal motility Atrophine is contra-indictated in glaucoma PT.

Anticholergenics - Anti Ulcer

Decreases acetylcholine produced - which therefore inhibits HCI production by binding histamine and acetylcholine production Take with meals to decrease acid secretions

Prednisone

Deltasone adrenal hormone: glucocorticoid

prednisone

Deltasone Class: adrenal hormone: glucocorticoid Dose: A: PO: 5-60 mg/d in single or divided doses C: PO: 0.1-0.15 mg/kg/d in single or divided doses Action: suppression of inflammation and adrenal function Contra: hypersensitivity, psychosis, fungal infection Caution: diabetes Sides: nausea, diarrhea, abdominal distension, increased appetite, sweating, headache, depression, flush, mood changes Adverse: petechiae, ecchymosis, hypertension, tachycardia, osteoporosis, muscle wasting, GI hemorrhage, pancreatitis, circulatory collapse, thrombophlebitis, embolism

What is a major cause of Type 1 diabetes?

Destruction of beta cells in the Islets of Langerhans in the Pancreas

macular degeneration

Destructive changes in the macula due to injury or gradual failure of the outer pigmented layer of the retina (the retinal layer adjacent to the choroid), which removes cellular waste products and keeps the retina attached to the choroid.

Upper GI study: Enteroclysis

Detailed, double-contrast study of the small intestine with a continuous barium infusion observed by fluoroscopy.

CT

Multiple x-rays at different angles are taken and digitized for cross-sectional images. Radiation doses are considerable. With or without contrast.

Thyroid hormone: Replacement and Antithyroid drugs Assessment

Determine baseline vitals to compare to future data Check serum T3, T4, and TSH levels Thyroid Replacement: obtain drug history. Be aware that these enhance the action of oral anticoagulants, sympathomimetics, and antidepressants. They decrease action of insulin, oral hypoglycemics, and digoxin. Phenytoin and aspirin can enhance the effects of thyroid hormone. Antithyroid Drugs: assess for signs and symptoms of thyroid crisis, including tachycardia, cardiac dysrhythmias, fever, HF, flushing, apathy, confusion, behavioral changes, and later hypotension and vascular collapse. Thyroid crisis can result from thyroidectomy (b/c excess hormones released), abrupt antithyroid withdrawal, excess ingestion of thyroid hormone, or failure to give antithyroid meds before thyroid surgery.

Postprandial blood glucose tests are recommended to....

Determine the response of blood glucose to a meal

deficiency of production of or secretion of ADH or a decrease renal response to ADH

Diabetes insipidus

Nephropathy

Diabetic kidney disease

How does hyperglycemia happen?

Diabetics lack insulin secretion either totally - happens with type 1 when their pancreatic beta cells are completely destroyed, or their bodies are unable to use the insulin secreted because they lack receptor sensitivity - happens mostly with type 2.

Prediabetes

Diagnosed when fasting glucose levels are higher than normal (100 mg/dL) but lower than 126 mg/dL; Or when Glucose Tolerance Test is higher than normal (140 mg/dL) but lower than 199 mg/dL; Or when HbA1c levels are higher than normal (6.0%) but lower than 6.5% People with prediabetes usually develop type II diabetes within 10 years; Treat with weight loss and exercise to prevent onset of type II diabetes

Dysphagia

Difficulty swallowing

Januvia (sitagliptin phosphate)

Dipeptidyl peptidase-4 inhibitor (DPP-4), Incretin modifier antidiabetic

The nurse providing postoperative teaching for a patient who has undergone LASIK surgery advises the patient to contact the office if he experiences: Dry eyes. Undercorrection of visual acuity. Decreased night vision with halos. Drainage from the eye.

Drainage from the eye. Rationale: Following surgery, patients can experience a temporary loss of sharpness, over- or undercorrection of visual acuity, dry eyes, or temporarily decreased night vision with halos, glare, and starbursts. Bleeding or drainage could indicate a surgical complication, and should be reported immediately.

ptosis

Drooping of the eyelid.

Side Effects of Cannabinoids - Antiemetic

Drowsiness, dry mouth, impaired thinking, euphoria, mood changes, headache, confusion, depersonalization, nightmares, incoordination, memory lapse, anxiety

Side Effects of Miscellaneous - Antiemetic

Drowsiness, dry mouth, tachycardia, urine retention, constipation, diarrhea, blurred vision

diflucan

Drug class: Azde antifungal. Generic name: fluconazole. Dosage: 400 mg/d. Route: PO/IV. Action: Increase permeability of fungal cell membrane. Side effects: anorexia, GI problems, headaches. Contra: Hypersensitivity

zofran

Drug class: antiemetic. Generic: Ondasetron. Dosage: 8 mg q8h. Route: PO/IV. Action: blocks serotonin. Side effects: headache, constipation, diarrhea, fatigue. Contra: hypersensitivity to ondansentron and 5-HT3 antagonist. Nurse: Do not give as needed (not a PRN)

Drug Use in Pre-Op setting: Benzodiazepines and barbiturates

Drug used in pre-op setting for sedation and amnesia

Drug Use in Pre-Op setting: Opioids

Drug used in pre-op setting to decrease intraoperative anesthetic requirements and pain.

Drug Use in Pre-Op setting: Anticholinergics

Drug used in pre-op setting to reduce secretions.

Side Effects of Anticholinergics - Antiemetic

Dry Mouth, constipation, dizziness - similar to antihisamines

fentanyl

Duragesic, Sublimaze Class: short-acting opioid Dose: A:IV: 8-40 mcg/kg for surgery continuous inf: 0.5-1 mcg/kg/min Total Dose: 8-40 mcg/kg A: IM: 50-100 mcg q1-2h PRN C: 2-12y: IM: 1.7-3.3 mcg q1-2h prn Lozenge: A/C: PO: suck on 400 mcg lozenge til sedated trandermal patch: A: initial: 25 mcg/h patch q3d

The length of time insulin works is called its...

Duration

Onset, Peak and Duration of BASAL Insulins

EXAMPLE- Insulin detemir BRAND NAME- Levemir ONSET- 1hr PEAK- No Peak DURATION- Up to 24 hrs

Onset, Peak and Duration of VERY SHORT ACTING Insulins

EXAMPLE- Insulin lispro BRAND NAME- Humalog ONSET- 5-15 min PEAK- 30-90 min DURATION- 5 hrs or less EXAMPLE- Insulin aspart BRAND NAME- Novolog ONSET- 10-20 min PEAK- 1-3 hrs DURATION- 3-5 hrs EXAMPLE- Insulin glulisine BRAND NAME- Apidra ONSET- 15-20 min PEAK- 30-90 min DURATION- 6 hrs

INTERMEDIATE ACTING Insulins

EXAMPLE- NPH BRAND NAME- Humulin N, Novolin N ONSET- 1-2 hrs PEAK- 6-12 hrs DURATION- 18-26 hrs **Remember Rule of evens 2-12-24 hrs

Onset, Peak and Duration of SHORT ACTING Insulins

EXAMPLE- Regular BRAND NAME- Humulin R, Novolin R ONSET- 30 min PEAK- 2-5 hrs DURATION- 5-8 hrs **Remember Rule of Odds 1-3-5

Oral hypoglycemic agents COMBINATION AGENTS

EXAMPLES- metformin and glyburide (Glucovance) metformin and rosiglitazone (Advandamet) glipizide and metformin (Metaglip) S/E- See individual agents NURSING IMPLICATIONS: -See individual agents

Kussmaul's respirations are used to ....

Eliminate excess acidity in patients with diabetic ketoacidosis

Client teaching: thyroid drug replacement and diet related to hypothyroidism

Encourage client to take drug @ same time each day, preferably before breakfast. Food hampers absorption rate. Teach to check warnings on OTC drug labels. Avoid OTCs that caution against use by ppl w/ heart/thyroid disease Direct client to report symptoms of hyperthyroidism (tachycardia, chest pain, palpitations, excess sweating) caused by drug accumulation or overdosing. Suggest client carry a MedicAlert card with the health condition and drug used. Diet Caution to avoid foods that inhibit thyroid secretion--strawberries, peaches, pears, cabbage, turnips, spinach, kale, Brussel sprouts, cauliflower, radishes, and peas)

Goiter

Enlargement of the thyroid gland. Usually non-toxic. (called endemic goiter when caused by iodine deficiency or other environmental factors.) SIGNS AND SYMPTOMS- Enlarged thyroid, swelling at base of neck

A preschool-age child is undergoing bilateral myringotomy with tubes for the treatment of recurrent otitis media. The mother asks the nurse what the tubes will do. The nurse's best response is that the tubes will: Provide for drainage. Prevent future episodes of otitis media. Equalize pressures between the middle ear and the environment. Prevent the spread of infection to the mastoid bone.

Equalize pressures between the middle ear and the environment. Rationale: Myringotomy (an incision of the tympanic membrane) is performed to relieve pressure caused by the edema and swelling of otitis media, which impairs the eustachian tubes' ability to equalize pressures between the middle ear and the environment. Without relief of pressure, the tympanic membrane can rupture, the delicate structures of the middle ear can be damaged, and infection can spread to adjoining tissues and structures. Infection can still occur and spread despite myringotomy.

Rectal sensory function studies

Evaluate rectal sensory function, neuropathy. A catheter with a balloon inflates until the patient feels distention. Tone and pressure are measured.

Motility study: Colonic transit

Evaluates motility and obstruction. Patient swallows a capsule with radionucleotide markers. X-rays are taken every 24 hrs until they are passed.

Small bowel enteroscopy: capsule

Evaluates obscure GI bleeding. Patient swallows a capsule which has a wireless camera and light source and transmits images to a device worn on a belt. Patient passes it in 1-2 days.

Endoscopy through ostomy

Evaluates the anastomosis for recurrent disease or treats bleeding.

PillCam ESO

Evalutes esophagus only. Swallowed by the patient. Has 2 cameras which take 7 images per second.

Examples of Stimulant Laxative (Irritants)

Ex-Lax, Correctol, Dulcolax, Senokot, Castor Oil

Hyperplasia

Excessive increase in the number of normal cells

Polydipsea

Excessive thirst

Polydipsia

Excessive thirst

Polyuria

Excessive urination

Nocturia

Excessive urination at night

What does the nurse need to teach about exercise to the diabetic?

Exercise is very important to do for a diabetic, but there are risks - teach the patient to check their blood glucose before, during and after exercise and if their blood sugar is greater than 250, they need to check their urine for ketones. This indicate that their insulin levels are not adequate Exercise puts the diabetic at risk for hypoglycemia and they need to be taught to not exercise within one hour of taking insulin injections or to exercise during the peak time of the insulin's action. However, exercising is very important and the diabetic should be taught that they should start off slowly to improve their hypertension, promote cardio fitness, assist with weight loss and improve blood lipids

Tests for Diabetes are...

Fasting Blood Glucose Glucose Tolerance Test HbA1c

The nurse is assisting with community education fair on risks for diabetes and heart disease. What are criteria for metabolic syndrome and should be included in instruction?

Fasting plasma glucose = 100 mg/dL BP = 130/85 mm/Hg HDL cholesterol is <40 mg/dL for men and ,<50 mg/dL for woman

A 45 yr old with diabetes forgot to take a daily dose of glyburide (Micronase). What symptom should the nurse be vigilant about ?

Fatigue, thirst, blurred vision

Side Effects of Osmotic Laxative - Saline

Fluid and Electrolyte Imbalances Hypotension, weakness

Symptoms of syndrome of inappropriate antidiuretic hormone (SIADH)

Fluid overload evidenced by weight gain, hyponatremia, low serum osmolality, concentrated urine, muscle cramps and weakness due to electrolyte imbalance; if untreated, lethargy, confusion, seizures, coma, death

Goitrogens

Foods that can cause a goiter (ex. Turnips, cauliflower)

Byetta (exenatide)

Glucagon-like peptide-1 receptor agonist (GLP-1 agonist)

Victoza (liraglutide)

Glucagon-like peptide-1 receptor agonist (GLP-1 agonist)

pituitary secretions

GH, TSH

Reglan (metoclopramide)

GI stimulant

Esophagogastroduodenoscopy

GI tract is viewed through a scope, video may be taken.

Client teaching: adrenal hormones

General advise to take as prescribed. Caution not to abruptly stop. When drug is stopped, dose is tapered over 1-2wks For short-term use (less than 10 days), the drug still needs to be tapered. Prepare schedule for client to decrease over 4-5d Direct client not to take cortisone preparations during pregnancy unless necessary and prescribed by HCP. Can be harmful to fetus Inform client that certain herbal laxatives/diuretics may interact and may increase severity of hypokalemia Instruct client to avoid ppl with respiratory infections b/c these drugs suppress the IS. Especially important if client is on high dose. teach client to inform HCPs of all drugs taken, especially before surgery Encourage client to carry MedicAlert Self-admin instruct client how to use aerosol nebulizer. Warn against overuse to avoid possible rebound effect Side effects Teach client to report signs/symptoms of overdose or Cushing syndrome: moon face, puffy eyelids, edema in feet, increased bruising, dizziness, bleeding and menstrual irregularity Diet Instruct client to take cortisone preps at mealtime or with food. These can irritate gastric mucosa and cause a peptic ulcer Advise client to eat foods rich in K, like fresh/dried fruits, vegetables, meats and nuts. Prednisone promotes K loss and hypokalemia

Type 1 Diabetes

Genetic; inactive/absent B cells

Chalazion

Granulomatous cyst or nodule of the lid.

autoimmune disease of unknow etiology marked by diffused thyroid enlargment and excessive thyroid hormone secretin

Graves' disease

An excess of which hormone is responsible for acromegaly?

Growth hormone

Pepcid (famotidine)

H2 receptor antagonist

Tagamet (cimetidine)

H2 receptor antagonist

Zantac (ranitidine)

H2 receptor antagonist

(SIADH) syndrome of inappropriate ADH Secretion

HORMONE PROBLEM: -ADH excess MAJOR SIGNS & SYMPTOMS -Water retention

Addison's disease

HORMONE PROBLEM: -Deficient steroids MAJOR SIGNS & SYMPTOMS -Hypotension

Pheochromocytoma

HORMONE PROBLEM: -Epinephrine Excess MAJOR SIGNS & SYMPTOMS -Labile Hypertension

Acromegaly

HORMONE PROBLEM: -Excess Growth hormone (GH) MAJOR SIGNS & SYMPTOMS -Growing hands and feet -Curvature of spine ________________________________________(dopamine is an agonist to GH) (Can cause swallowing problems or patients may need speech therapy) THERAPEUTIC MEASURES-aimed to treat cause. Use vasopressins.

Short stature

HORMONE PROBLEM: -Growth hormone (GH) deficiency MAJOR SIGNS & SYMPTOMS -Failure to grow and develop

Graves' Disease

HORMONE PROBLEM: -High T3 and T4 MAJOR SIGNS & SYMPTOMS -Exophthalmos

Hyperparathyroidism

HORMONE PROBLEM: -High serum calcium MAJOR SIGNS & SYMPTOMS -Muscle weakness, brittle bones

Hypothyroidism

HORMONE PROBLEM: -Low T3 and T4 MAJOR SIGNS & SYMPTOMS -Weight gain and fatigue

Hypoparathyroidism

HORMONE PROBLEM: -Low serum calcium MAJOR SIGNS & SYMPTOMS -Tetany

Cushing's syndrome

HORMONE PROBLEM: -Steroid excess MAJOR SIGNS & SYMPTOMS -Moon Face

Diabeted Insipidus

HORMONE PROBLEM: -antidiuretic hormone (ADH) deficiency MAJOR SIGNS & SYMPTOMS -Polyuria

Lactulose

Has big sugar molecules that draw water into the bowel Also decreases ammonia levels in liver PT's AVOID WITH DIABETES

Autoimmune disorders in which the body attacks itself

Hashimoto's thyroiditis, Grave's disease

The nurse is caring for a patient with Type 2 diabetes mellitus who started oral hypoglycemic medication 2 months ago for fasting blood glucose level of 160 mg/dl. What would be a good indicator that the treatment was working?

HbA1c 5%

Side Effects of Serotonin Receptor Antagonist - Antiemetic

Headache, diarrhea, dizziness, fatigue

Side Effects of Proton Pump Inhibitor or PPi's - Anti Ulcer

Headache, dizziness, diarrhea, abdominal pain, rash, dry mouth flatulence

Tetany

Muscle spasms, numbness and tingling caused by changes in PH and low serum calcium.

Which of the following suggestions should be taught by the nurse to the patient with tinnitus to assist the patient with coping with the symptom to promote concentration and sleep? Earplugs Sleeping medication Hearing aid that produces a tone to mask tinnitus Bedtime use of nasal decongestant spray

Hearing aid that produces a tone to mask tinnitus Rationale: Options for masking tinnitus to promote concentrate and sleep include ambient noise from a radio or sound system; masking device or white noise machine; hearing aid that produces a tone to mask the tinnitus; and hearing aid that amplifies ambient sound. Earplugs will focus the patient's attention on the sound. The patient's issue is the noise, not a sleeping disorder; hence, sleeping medications are not indicated. The tinnitus is a result of neurosensory damage, not of congestion.

Retinitis pigmentosa

Hereditary degenerative disease characterized by retinal atrophy and loss of retinal function progressing from the periphery to the central region of the retina.

High or low: serum sodium levle

High; pure water loss

Acute, life threatening complication of uncontrolled type 2 diabetes

Hyperosmolar hyperglycemia

The nurse giving Humulin R 20 U at 7 AM is aware that this drug will peak in: 1. 15 minutes. 2. 30 minutes. 3. 1 hour. 4. 2 hours.

Humulin R has its onset in about 15 minutes, but its peak is in 2 hours.

Symptoms of Hypoglycemia

Hunger sweating, tremor, blurred vision, headache and irritability; If not treated, confusion, seizures and coma

Therapeutic measures for DM

Hydration medications (Hypotonic IV fluids) can be ordered to replace intravascular volume without adding excess sodium. Vasopressins - nasal spray

Osmotic Laxative - Saline

Hyper-osmolar salts pull water into the colon and increase water in the feces to increase bulk, which puts pressure on the colon walls and stimulates peristalsis Must monitor electrolytes (Mg and Na are absorbed)! Need good renal function. Saline are contra-indictated in PTs with CHF.

Too much sugar in the blood is called...

Hyperglycemia

What acute high blood glucose complication can occur as a result of uncontrolled type 2 DM?

Hyperosmolar Hyperglycemia

adequate ADH, but there is a decrease response in ADH in the kidney; lithium is the most common cause

Nephrogenic DI

1. A client with hypoparathyroidism complains of numbness and tingling in his fingers and around the mouth. The nurse would assess for what electrolyte imbalance?

Hypocalcemia

The nurse recognizes that teaching is effective if a patient with diabetes knows to use sub Q glucagon for an emergency episode of which of the following conditions?

Hypoglycemia

Too little sugar in the blood is called...

Hypoglycemia

A patient who reports frequent urination and excessive thirst is being evaluated for possible diabetes insipidus. During the assessment the nurse notes poor skin turgor, BP 95/65 mm Hg and pale urine. What complication should the nurse be most concerned about?

Hypovolemic shock

The nurse is assisting with the teaching for Addison's disease about new orders for hydrocortisone and fludrocortisone. This statements by the patient indicates that more teaching is necessary.

I will eat a low-sodium diet.

diagnostic studies for agromegaly

IGF-1 levels, IGF binding protein-3 levels, GH response to oral glucose challenge; OGTT (GH secretion is inhibited by glucose) GH levels does not fall below 3 ng/mL, MRI, CT

treatment of severe SIADH (serum sodium less than 120)

IV hypertonic saline solution 3-5%; infuse slowly!!!

When should you not use metformin, Glucophage?

If a patient has liver disease COPD, congestive heart failure, Peripheral vein disease also, if has had contrast material or dye or any surgical procedure requiring anesthesia, must hold for 48 hours!

Protein in the urine is a sign of which long-term complication of diabetes?

Nephropathy

Importance of Pituitary

Important for homeostasis. Regulates: growth, breast milk production, sex organs, thyroid, water

Somatostatin Analogs - Anti-diarrheal

Inhibits gastric acid, secretion, gastrin, pepsinogen, cholecystokinin, serotonin, and intestinal fluids. Use is for sever diarrhea from cancer.

Example of Somatostatin Analogs - Anti-diarrheal

Octreotide (Sandostatin)

What should initial treatment be for a type 2 diabetic?

Initial treatment should be DIET AND EXERCISE!

myringotomy

Incision of the tympanic membrane.

What dietary recommendations will reduce the risk of kidney stones in the patient with hyperparathyroidism?

Increase fluids

Stimulant Laxative (Irritants)

Increase peristalsis by irritating sensory nerve endings in intestinal mucosa Results - orally 6 -12 hours Suppository - 15 - 60 minutes

Prostaglandin Analogs - Anti Ulcer

Increases cytoprotective mucous in the GI tract - Synthetic Often added to NSAIDS to prevent gastric irritation; NO Pregnant women

In addition to stimulating insulin production, glyburide (Micronase) has what effects?

Increases tissue sensitivity to insulin

Nursing diagnoses: Pit hormones

Ineffective health maintenance related to lack of ability to maintain drug regimen Delayed growth/development related to deficient stimulation of GH

conjunctivitis

Inflammation of the conjunctiva.

Keratitis

Inflammation of the cornea.

Labyrinthitis

Inflammation of the inner ear.

otitis media

Inflammation or infection of the middle ear.

Client teaching: hyperparathyroidism

Instruct client to report signs/symptoms of hypercalcemia: bone pain, anorexia, nausea, vomit, thirst, constipation, lethargy, bradycardia, and polyuria Advise women to inform HCP about pregnancy status before taking calcitonin preparation Encourage client to check OTCs for possible Ca content, especially if client has elevated calcemia. Some vitamins/antacids have calcium. Tell client to contact HCP before taking drugs w/ calcium

Client teaching: antithyroids

Instruct client to take drugs w/ meals to decrease GI symptoms advise client about effects of iodine and presence in iodized salt, shellfish, and OTC cough meds Emphasize the importance of drug compliance; abruptly stopping could bring on thyroid crisis Teach signs/symptoms of hypothyroidism: lethargy, puffy eyelids and face, thick tongue, slow speech w/ hoarseness, lack of perspiration, and slow pulse. Hypothyroidism can result from treatment for hyperthyroidism. Advise client to avoid antithyroid drugs if pregnant/breastfeeding. Can cause hypothyroidism in fetus/infant

Post op care for pts having eye surgery?

Instruct pt: -avoid straining, no heavy lifting, avoid sneezing, coughing, vomiting, sex -bend from knee to pick things up -in usage of eye drops and eye shields (wear eye shield while showering) - encourage to sleep on unaffected side - wear sunglasses when outside - avoid aspirin bc of effects on blood clotting - teach proper cleansing of eye (from inner to outer canthus) - report s/s of hemorrhage: *sudden, sharp pain *pain in and around the eye *sudden brow pain *change in vision (double or blurred) - should not have any decrease in vision; tell pt that their best vision will happen after 4-6 weeks post op *abnormal BP or vitals - if pt has abnormals, recheck IMMEDIATELY, observe for symptoms - rapid pulse with low BP means possible hemorrhage *signs of infection

Humulin N

Intermediate-acting insulin

NPH (insulin isophane)

Intermediate-acting insulin

Novolin N

Intermediate-acting insulin

Lente

Intermediate-acting insulin. Action: contains protamine and zine, a protein that prolongs the action of insulin. Onset: 1 to 2 hrs. Peak: 6 to 12 h. Duration: 18 to 24 h. Mode of Action: Insulin promotes use of glucose by body cells. Effects: Confusion, agitation, tremors, headache, flushing, hunger, weakness, lethargy, fatigue, hunger, weakness, lethargy, fatigue, urticaria; redness, irritation, or swelling at insulin injection site. Adverse reactions: tachycardia, palpitations, hypoglycemic reactions, rebound hyperglycemia. Life-threatening: shock, anaphylaxis. Nursing Interventions: Monitor vital signs (tachycardia can occur). Determine blood glucose levels and report changes. Monitor the clients HbA1c. Prepare a teaching plan based on client\'s knowledge of health problem, diet, and drug therapy. Teaching: Instruct client to report symptoms of hypoglycemic (insulin) reaction: headache, nervousness, sweating, tremors, rapid pulse; and symptoms of hyperglycemic reaction (diabetic acidosis): thirst, increased urine output. Advise client that hypoglycemic reactions are more likely to occur during the peak action time. Explain that orange juice, sugar-containing drinks may be used when hypoglycemic reactions begin. Teach family members to administer glycagon by injection of client has a hypoglycemic reaction. Inform client that certain herbs may react with insulin and oral antidiabetic drugs. Instruct client about the necessity for compliance with prescribed insulin therapy and diet. HbA1c provides most accurate picture of optimal diabetic control. Advise client to carry a MedicAlert card, tag, or bracelet. Instruct client how to check blood glucose with a glucometer.

NPH

Intermediate-acting insulin. Action: contains protamine and zine, a protein that prolongs the action of insulin. Onset: 1 to 2 hrs. Peak: 6 to 12 h. Duration: 18 to 24 h. Mode of Action: Insulin promotes use of glucose by body cells. Effects: Confusion, agitation, tremors, headache, flushing, hunger, weakness, lethargy, fatigue, hunger, weakness, lethargy, fatigue, urticaria; redness, irritation, or swelling at insulin injection site. Adverse reactions: tachycardia, palpitations, hypoglycemic reactions, rebound hyperglycemia. Life-threatening: shock, anaphylaxis. Nursing Interventions: Monitor vital signs (tachycardia can occur). Determine blood glucose levels and report changes. Monitor the clients HbA1c. Prepare a teaching plan based on client\'s knowledge of health problem, diet, and drug therapy. Teaching: Instruct client to report symptoms of hypoglycemic (insulin) reaction: headache, nervousness, sweating, tremors, rapid pulse; and symptoms of hyperglycemic reaction (diabetic acidosis): thirst, increased urine output. Advise client that hypoglycemic reactions are more likely to occur during the peak action time. Explain that orange juice, sugar-containing drinks may be used when hypoglycemic reactions begin. Teach family members to administer glycagon by injection of client has a hypoglycemic reaction. Inform client that certain herbs may react with insulin and oral antidiabetic drugs. Instruct client about the necessity for compliance with prescribed insulin therapy and diet. HbA1c provides most accurate picture of optimal diabetic control. Advise client to carry a MedicAlert card, tag, or bracelet. Instruct client how to check blood glucose with a glucometer.

The patient with glaucoma has been prescribed travoprost (Travatan), a prostaglandin analog, to improve the outflow of aqueous humor and reduce ocular pressure. Of the following signs or symptoms, which will the nurse teach the patient to report immediately to the eye doctor? Burning in the eye when the eye drops are used Tearing in the eye when the eye drops are used Changes in the color of the eyes Intolerable pain in the eyes after the drops are used

Intolerable pain in the eyes after the drops are used Rationale: Prostaglandin analogs have some adverse effects, such as blurred vision and stinging, and, when used long-term, cause permanent darkening of the iris of the eye and eyebrows, increased growth of eyelashes, and conjunctival redness.

Which routes can Insulin be administered?

Intravenous (IV) Sub Q Intramuscular

Myxedema coma

Is caused by untreated hypothyroidism

Endogenous hormone

Is made by the body and does not need to be ingested or injected

Positive Trousseau sign

Is the indication of thumb and fingers spasm within 3 minutes of the sphygmomanometer is placed on the patients arm and pumped above the patients systolic pressure.

GoLytely - Polyethylene glycol (PEG)

Isotonic, has electrolytes, not absorbed Used for bowel preps for surgery or diagnostic tests Drink 2-4 liters of it over 3 hours - 240 mL q 15 minutes CAN put in NG tube

potassium

Kaochlor, Kaon Cl, Kay Ciel, Micro K, K Dur Class: electrolyte Dose: hypokalemia(maint): A: PO: 20 mEq in 1-2 doses hypokalemia(correction):PO: 40-80 in 3-4 doses IV: 20-40 diluted in 1 L of IV sol\\\'n Action: transmits/conducts nerve impulses; contracts skeletal, smooth, and cardiac m. Contra: renal insufficiency/failure, Addison\\\'s disease, hyperkalemia, severe dehydration, acidosis, K-sparing diuretic Side: vomit, diarrhea, abdom cramp, irritability, rash, phlebitis with IV admin Adverse: hyperkalemia, oliguria, ECG changes (peaked T waves, widened QRS complex, prolonged PR interval), GI ulceration, cardiac dysrhythmias, respiratory distress, cardiac arrest

Examples of Absorbents - Anti-diarrheal

Kaolin-Pectin (Kaopectate) Bismuth Salts (Pepto-Bismol)

Deep, sighing respiration's from diabetic acidosis are called...

Kussmaul's respirations

Digoxin

Lanoxin Class: cardiac glycoside Dose: A: PO: 0.5-1 mg initially in 2 doses; maint: 0.125-0.5/d IV: same but given over 5 min Oldies: 0.125 mg/d C: PO: 1mo-2y: 0.01-0.02 mg/kg in 3 doses; maint: 0.012 mg/kg/d in 2 doses IV: dosage varies C:2-10y:PO: 0.012-0.04 mg/kg in divided doses Pediatric doses are usually ordered in mcg in elixir form Action: inhibit Na-K ATPase, promoting increased contractility, CO, and tissue perfusion, decreases ventricular rate Contra: ventricular dysrhythmias, 2nd-3rd degree heart block Caution: AMI, renal disease, hypothyroidism, hypokalemia Side: anorexia, nausea, vomit, diarrhea, headache, blurred vision (yellow-green halos), diplopia, photophobia, drowsy, fatigue, confusion Adverse: bardycardia, visual disturbances, AV block, cardiac dysrhythmias Antidote: digibind Therapeutic level: 0.5-2.0 ng/mL

Lactulose

Laxative

drug of choice to treat hypothyroidism

Levothyroxine (Synthroid)

synthroid

Levothyroxine sodium Class: thryoid hormone Dose: PO: initial: 25-50 mcg/d; maint: 50-200/d IV/IM: 50-100/d C over 12: PO: 2-3 mcg/kg/d Action: increase BMR, O2 consumption, and body growth contra: thyrotoxicosis, MI, severe renal disease Caution: CVD, hypertension, angina pectoris Side: nausea, vomit, diarrhea, cramps, tremors, nervous, insomnia, headache, weight loss Adverse: tachycardia, hypertension, palpitations, thryoid crisis, angina pectoris, cardiac dysrhythmias, CV collapse

Example of Tranquilizer

Librax Roubinul

Corneal ulcer

Local necrosis of the cornea that may be caused by infection, exposure trauma, or the misuse of contact lenses.

Examples of Opiates and Opiate Related Drugs - Antidiarrheal

Lomotil (diphenozylate with atropine) Immodium (lacks CNS SE)

Lantus (insulin glargine)

Long-acting insulin

Levemir (insulin detemir)

Long-acting insulin

The nurse is collecting data on a patient who reports fatigue, weight gain, constipation and dry skin. The physician suspects primary hypothyroidism. Which of the following test results would the nurse expect to see.

Low T3 and T4 levels, elevated thyroid-stimulating hormone levels

Lovenox

Low-molecular-weight heparins. Trade names: Enoxaparin sodium. Dosage: SubQ: 30 mg b.i.d. for 10 to 14 d. Administered subQ once or twice a day. PB: UK. Half-life: 3 to 4 h. (two to four times longer than heparin. Heparin has a very short 1/2 life). Action: For thromboembolism. Prevents and treats DVT (deep vein thrombosis) and pulmonary embolism after orthopedic or abdominal surgery. Side effects: Bleeding. Nursing Interventions: Monitor CBC (complete blood count). Pregnancy category: B. Contraindications: for clients with strokes, peptic ulcers, and blood anomaines. These drugs should not be given to clients having eye, brain or spinal surgery. Nursing interventions: Frequent laboratory monitoring of aPTT is not required. Drug can be administered at home unlike heparin. Teaching: teach client or family member how to administer the subQ injection, which is usually given in the abdomen. Average treatment period lasts 7 to 14 days. Not to take antiplatelet drugs such as aspirin while taking LMWHs or heparin.

Emollients (Stool Softeners)

Lowers surface tension Promotes water accumulation in intestines Emulsifies and lubricates feces for easier passage Use: reduces straining post surgery or MI

Nardil

MAO Inhibitor old drug; inhibit MAO breakdown of NE, dopamine, serotonin; intensify neurotransmission (LAST RESORT) S.e: orthostatic hypotension; treat with Calcium channel blocker Serotonin Syndrome (SES) ocurs when taken with other meds that affect serotonin (Confusion, hypertension, tremors, coordination changes) avoid caffeine, alcohol, tyramine foods (cheese, beer, beef, yeast, chocolate, bananas, raisins) Used as last resort

ramelteon/ Rozerem

MELATONIN RECEPTOR AGENT 30 min onset and short duration

adenosine/ Adenocard, Adenoscan

MISCELLANEOUS DRUGS dysrhythmias; usually reserved for atrial dysrhythmias; half-life is only 10 seconds

MRI

Magnetic fields and radio waves produce images of the area. Oral contrast agents enhance the image. Ferromagnetic objects can cause injury.

What is the best way for patients to avoid long-term complications of diabetes?

Maintain pre-meal blood glucose levels under 1.30 mg/dL

Which of the following physical assessment maneuvers will allow the nurse to differentiate the ear pain of a patient with otitis externa from the ear pain of a patient with otitis media? Manipulation of the auricle Irrigating the ear canal with cool water Rinne and Weber tests Whisper test

Manipulation of the auricle Rationale: The pain of otitis externa can be differentiated from that associated with otitis media by the manipulation of the auricle. In external otitis, this maneuver increases pain, whereas the patient with otitis media experiences no change in pain perception. Rinne and Weber tests are used to diagnose sensorineural hearing loss. The caloric test (irrigation with cool water) evaluates vestibulo-ocular reflex. The whisper test evaluates hearing loss.

Defecography

Measures anorectal function using thick barium paste and fluoroscopy while the patient defecated the paste.

Manometry

Measures changes in intraluminal pressures and muscle activity (esophageal, gastroduodenal, small intestine, colonic, anorectal).

Goitrogenic

Medications like propylthiouracil, sulfonamides, lithium and salicylates that are goiter producing.

Prandin (repaglinide)

Meglitinide antidiabetic

Starlix (nateglinide)

Meglitinide antidiabetic

What is Meniere's Disease? What is the priority health problem for Meniere's Disease?

Meniere's Disease is characterized by: tinnitus, vertigo, 1 sided sensorineural hearing loss. Priority health prob is risk for injury - you need to keep the pt. safe!! Pt is at risk for fall because of the vertigo primarily.

Examples of Bulk-Forming Laxative

Metamucil Fiber-Con Citrucell Perdiem

What is the main drug in the Biguanides class?

Metformin, Glucaphage

Example of Prostaglandin Analogs - Anti Ulcer

Misoprostol (Cytotec)

Which of the following aspects for health teaching regarding acute and chronic conjunctivitis is the most important information to be emphasized to a group of preschool parents? Preventable cause of blindness Importance of early medical diagnosis Mode of transmission Pain management

Mode of transmission Rationale: The nurse should emphasize to the group ways to prevent the transmission of the infection. Conjunctivitis is highly contagious, and can be transmitted by direct contact with contaminated fomites (e.g., hands, tissues, towels, pillow cases), and some forms can be transmitted to an infant during vaginal delivery. Prevention is the most important aspect of conjunctivitis to be communicated to the group. Pain with acute conjunctivitis is not common.

Nursing Interventions for Diarrhea

Monitor BP and Respirations Report more than 10-15 mmHg decrease in BP Monitor frequency of BM and bowel sounds Assess for evidence of dehydration (electrolytes) Avoid fried foods, milk, and sedatives If diarrhea persists after 48 hours, notify MD

Interventions for Risk for Injury related to calcium imbalance secondary to hyperparathyroidism

Monitor calcium levels, encourage oral fluids, encourage weight-bearing exercises and ambulation with assistance, encourage smoking cessation, teach what symptoms to report

Evaluation: PTH insufficiency

Monitor effectiveness of drug therapy Continue to monitor for signs/symptoms of hypocalcemia (tetany) when commercially prepared calcitonin has been given

The nurse recognizes that opioid analgesics exert their action by interacting with a variety of opioid receptors. Drugs such as morphine act by activating

Mu and kappa

BuSpar

NON-BARBUTURATES se: dizzy, h/a, drowsiness

insulin pen, insulin pump

Name two alternate therapies for diabetes:

aspirin

Name: ASA, Bayer, Ecotrin Class: analgesic, antiinflamatory Dose: A: PO: 325-650 mg q4h; max: 4g/d C: PO: 40-65 mg/d or 10-15 mg/kg in 4-6 doses; max: 3.5 g/d TIA and thromboembolic condition: A:PO: 81-325/d Arthritis: A:PO:3.6-5.4/d in divided doses TDM: 15-30 mg/dL (150-300 mcg/mL) Action: inhibit prostaglandin syn; inhibit hypothalamic heat-regulator center Contras: hypersensitivity to salicylates or NSAIDs, flu/virus symptoms in kids, 3rd trimester Caution: renal/hepatic disorders Sides: anorexia, nausea, vomit, diarrhea, dizzy, confusion, hearing loss, heartburn, rash, stomach pain, drowsy Adverse: tinnitus, urticaria, ulceration, agranulocytosis, hemolytic anemia, bronchospasm, anaphylaxis, thrombocytopenia, hepatotoxicity, leukopenia

Epinephrine

Name: Adrenalin Class: sympathomimetic Action: action on 1+ adrenergic sites; promotio0n of CNS & cardiac stimulation & bronchodilation Dose:Severe anaphylaxis: A: subQ:0.1-0.5 mL of 1:1000 PRN IV: 0.1-0.25 mg 1:10,000 infused slowly over 5-10min subQ/IM: 0.3 mg EpiPen autoinjector Contra: cardiac dysrhythmias, cerebral arteriosclerosis, pregnancy, narrow-angle glaucoma, cardiogenic shock Caution: hypertension, prostatic hypertrophy, hyperthyroidism, diabetes mellitus (hyperglycemia may occur) Sides: anorexia, nausea, vomit, nervous, tremor, agitation, headache, pallor, insomnia, syncope dizzy Adverse: palpitation, tachycardia, dyspnea, V-fib, pumonary edema antidote: phentolamine mesylate (Regitine) 5-10 mg diluted in 10-15 mL of saline into area for IV extravasation of NE and dopamine

metformin

Name: Glucophage, Glucophage XR Class: biguanide Dose: initial: 500 mg 1-3x/d w/ or shortly after meals; increase gradually; max: 2550 mg/d PO:ER: 500 mg/d w/ evening meal; max: 2000 mg/d Action: increases binding of insulin receptors, improves tissue sensitivity to insulin, increases glu transport into skeletal m. and fatty tissue, decrease glu production in liver by reducing gluconeogenesis, reduces glu absorption from intestines Contra: hypersensitive, concurrent infection, hepatic/renal dysfunction, cardiopulmonary insufficiency, alcoholism caution: pregnancy, lactation side: dizzy, fatigue, confusion, headache, agitation, bitter/metallic taste, anorexia, nausea, vomit, diarrhea adverse: lactic acidosis, malabsorption of aminos, vitamin b12, and folate

lisinopril

Name: Prinivil, Zestril Class: ACE-I Dose: initial: PO: 10 mg/d; maint: 20-40 mg/d; max: 80/d contra: pregnancy, K-sparing diuretics, hyperkalemia sides: constant, irritated cough, nausea, vomit, diarrhea, headache, dizzy, fatigue, insomnia, hyperkalemia, tachycardia adverse: first-dose hypotension, hyperkalemia

Nexium

Name: esomeprazole Class: proton pump inhibitor Dose: A: PO: 20-40 mg/d Action: used in therapy for H. pylori Contras: hypomagnemesia, severe liver disease, osteoporosis, osteopenia Sides: dizzy, confusion, tachycardia, dystonia, jitters, diarrhea, muscle cramping/weakness, cough, seizure, headache, nausea, stomach pain, gas, constipation, dry mouth

Pepcid

Name: famotidine Class: antiulcer:antacid Dose: A: PO 20 mg bid or 40 hs; maint: 20 hs IV: 20 mg bid diluted Action: inhibits gastric secretion sides: headaches, dizzy, constipation, pruritus, skin rash, gynecomastia, decreased libido, impotence

Lasix

Name: furosemide, Delone Class: loop diuretic Dose: A:PO:20-80 mg/d single dose; may increase in 6-8h, 20-40 mg; max = 600 mg/d IM/IV: 20-40 mg over 1-2 min IV; repeat 20 mg in 2h C: PO: 2 mg/kg single dose; repeat in 6-8h; max: 6 mg/kg/d IM/IV: 1 mg/kg single dose; repeat 1 mg/kg in 6-12h Action: inhbition of Na and water reabs\\\'n from loop of Henle and distal renal tubes; K, Mg, Ca may be lost Contra: presence of severe electrolyte imbalance, hypovolemia, hepatic coma Sides: nausea, diarrhea, electrolyte imbalance, vertigo, cramping, rash, headache, weakness, ECG change, blurred vision, photosensitivity Adverse: severe dehydration; marked hypotension; renal failure, thrombocytopenia, agranulocytosis

Levaquin

Name: levofloxacin Class: antibacterial: fluoroquinolone Dose: PO: 500-750 mg/d for 7-14d; IV: 500/d inf over 60 min Action: interference w/ enzyme(DNA gyrase), needed for bacterial DNA syn; bactericidal Contra: severe renal disease, hypersensitivity to quinolones, pregnancy, breastfeeding Caution: seizure disorders, renal disorder, kid under 14, older adults, ppl taking theophylline Sides: nausea, vomit, diarrhea, abdom cramp, gas, headache, dizzy, fatigue, restless, insomnia, rash, flushing, tinnitus, photosensitivity Adverse: Stevens-Johnson, encephalopathy, seizures, pseudomembranous colitis, dysrhythmias

dilantin

Name: phenytoin Class: anticonvulsant: hydantoin Dose: A:PO: 100 mg, tid/qid IV: LD: 10-14 mg/kg/d; max 300 mg/d in divided doses C: 5 mg/kg/d in 2-3 doses therapeutic range: 10-20 mcg/mL; toxic: 30-50 Action: reduce motor cortex activity by alter ion transport Contra: hypersensitivity, heart block, psychiatric disorder, pregnant Side: headache, diplopia, confusion, dizzy, sluggish, decreased coordination, ataxia, slurred speech, rash, anorexia, nausea, vomit, hypotension (after IV), pink-red/brown urine Adverse: leukopenia, hepatitis, gingival hyperplasia, nystagmus, hirsutism, osteoporosis, aplastic anemia, thrombocytopenia agranulocytosis, Stevens-Johnson, hypotension, V-fib, encephelopathy

metoprolol

Name:Lopresor, Toprol SR Class: antihypertensive:B1-block Dose: Hypertension:A: PO:50-100 mg/d in 1-2 doses; maint:100-450 in divided dose; max = 450 mg/d in divided doses oldies: PO: 25 mg/d; maint: 25-300 mg/d SR: 50-100/d; max = 400 mg/d MI: A: PO 100 mg bid IV: 5 mg q2min x 3 doses Action: promotion of BP reduction via B1-blocking effect Contra: 2nd-3rd degree heart block, cardiogenic shock, HF, sinus bradycardia Caution: hepatic/renal/thyroid dysfunction, asthma, peripheral vascular disease, type 1 diabetes Sides: fatigue, weakness, dizzy, nausea, vomit, diarrhea, mental changes, nasal stuffiness, impotence, decreased libido, depression Adverse: bradycardia, thrombocytopenia, complete heart block, bronchospasm, agranulocytosis

Side effects of Prednisone

Nausea, diarrhea, abdom distention, increased appetite, sweating, headache, depression, flush, mood changes

nitroglycerin

Nitrostat, Nitro-Bid, Transderm-Nitro Patch Class: antianginal Dose: PO/SL: 0.3, 0.4, 0.6 mg; repeat q5min x 3 PRN; SR: 2.5-26 mg, 2-4 x d IV: initial: 5mcg/min; dose may increase Ointment: 2% 1-2 in to chest/thigh Patch: 2.5-15 mg/d to chest/thigh Action: decrease myocardial oxygen demand; decrease preload by vasodilation, indirectly decreasing afterload Contra: marked hypotension, AMI, increased ICP, severe anemia Caution: severe renal/hepatic disease, early MI Side: nausea, vomit, headache, dizzy, syncope, weakness, flush, confusion, palor, rash, dry mouth Adverse: hypotension, reflex tachycardia, paradox bradycardia; circulatory collapse

The patient who gets up to urinate at night has...

Nocturia

Lunesta, Ambien

Non-Benzodiazapine CNS Depressant Rapid onset/ Sleep

Examples of Antihistamines - Antiemetic

Non-Prescription - Dramamine, Benadryl, Pepto Bismol Prescription - Visaril, Phenergran

Side Effects of Antihistamines - Antiemetic

Non-Prescription - Drowsiness, dry mouth, constipation Prescription - Dry mouth blurred vision, drowsiness, tachycardia, urinary retention, photosensitivity, EPS

Antihistamines - Antiemetics

Non-prescription: acts directly on gastric mucosa to suppress vomiting Prescription: acts on VC and decreases stimulation of CTZ Infiltration of Phenergan cause necrosis

Myxedema

Nonpitting edema that occurs in hypothyroidism

Assessment: adrenal hormone

Note baseline vitals Assess lab results: especially serum electrolytes and glycemia. Serum K level usually decreases and glycemia increases when taken over extensive period Obtain client weight and urine output Assess medical/herbal history. Report if glaucoma, cataracts, peptic ulcer, psychiatric problems, or diabetes. These drugs can intensify these health problems

Assessment: PTH insufficiencies

Note serum Ca level Asses for signs of tetany in hypocalcemia: twitching of mouth, tingling/numbness in fingers, carpopedal spasm, spasmodic contractions, and larygeal spasm

Following thyroidectomy, the nurse watches carefully for which signs of tetany?

Numb fingers and muscle cramps (caused by change in PH and low serum calcium)

Symptoms of diabetic neuropathy

Numbness and pain in extremities, sexual dysfunction, gastroparesis

Symptoms and treatment of acute tetany

Numbness and tingling around mouth and fingers, muscle spasms, cardiac dysrythmias; treat with IV calcium gluconate

Treatment options for DM

Nutrition, exercise, insulin or oral hypoglycemic agents, blood glucose monitoring, education

Type 1 Diabetes

ONSET - Rapid AGE AT ONSET - Usually younger than 40 RISK FACTORS - Virus, autoimmune response, heredity USUAL BODY TYPE - Lean HIGH BLOOD GLUCOSE COMPLICATION - Ketoacidosis TREATMENT - Diet, exercise; Must have insulin to survive

Type 2 Diabetes

ONSET - Slow AGE AT ONSET - Usually older than 40 RISK FACTORS - heredity, obesity USUAL BODY TYPE - Obese HIGH BLOOD GLUCOSE COMPLICATION - Hyperosmolar hyperglycemia; may develop ketoacidosis TREATMENT - Diet, exercise; May need oral hypoglycemics or insulin to control blood glucose level

acetic acid and hydrocortisone/ Vosol HC

OTIC PREPARATIONS Combination of acetic acid and glucocorticoid; for general ear infections and inflammation; prescription medications

carbamide peroxide/ Debrox

OTIC PREPARATIONS to soften, loosen and remove excessive ear wax; OTC medication

One of the risk factors for type 2 diabetes is...

Obesity

Hypovolemic shock

Occurs if fluid deficit is not corrected

What is a breakfast menu must appropriate for a patient with diabetes?

One slice whole grain toast with peanut butter, skim milk, orange juice

cataract

Opacification (clouding) of the lens of the eye.

Osmolality

Osmotic concentration; ionic concentration of the dissolved substances per unit of solvent.

Mannitiol

Osmotic diuretic. Trade name: Osmitrol. Dosage: IV: Initially 1 g/kg of 20% to 25% sol as a bolus. Action: To treat increased intracranial pressure or intraocular pressure, cerebral edema. Mode of Action: Inhibition of reabsorption of electrolytes and water affecting pressure of glomerular filtrate. Half Life: 100 minutes. Duration: 6-8 h. Peak: 1 h. Side effects: Temporary volume expansion, hyponatrema/hypernatremia, hypokalemia/hyperkalemia, dehydration, blurred vision, dry mouth. Adverse reactions: Pulmonary congestion, fluid/electrolyte imbalances. Contraindications: Hypersensitivity, severe dehydration, pregnancy, breastfeeding, current intracranial bleeding. Nursing Implications: Must be given with extreme caution to clients who have heart disease and HF. Teaching: It should be immediately discontinued if the client develops HF or renal failure.

OxyContin

Oxycodone, Narcotic pain reliever, analgesic

Pharmacokinetics: synthroid

PB = 99 t1/2 = 6-7d excretion: bile and feces

Pharmacokinetics: prednisone

PB: 70-90% t1/2: 3.5h Excretion: urine

milrinone/ Primacor

PHOSPHODIESTERASE INHIBITORS heart failure

Child dosing of Prednisone

PO 0.1-0.15 mg/kg/d in single or divided doses

Dosage Synthroid: Child over 12

PO 2-3 mcg/kg/d

Adult dosing of Prednisone

PO 5-60 mg/d in single or divided doses

Dosage Synthroid: Adult

PO Initially: 25-50 mcg/d Maintenance: 50-200 mcg/d IV/IM 50-100 mcg/d

amiodarone/ Cordarone

POTASSIUM CHANNEL BLOCKERS (CLASS III) dysrhythmias; usually reserved for serious ventricular dysrhythmias

Pathophysiology of type 2 DM

Pancreas does not produce enough insulin or tissues are resistant to insulin, causing elevated blood glucose. heredity and obesity are major factors

The nurse develops the nursing diagnosis of Acute pain r/t bone demineralization for a patient with hypoparathyroidism. What goal is most appropriate?

Patient will verbalize acceptable pain level. (The nurse should address the pain)

primary goal of treatment for Cushings

normalize hormone secretion, surgical removal of pituitary tumor, drugs (ketoconazole and aminoglutethimide)

The time when insulin is working its hardest after injection is called...

Peak Action Time

Example of Anticholergenics

Pro-Banthine

Nephrogenic

Produced by the kidney

Laxatives

Promote soft stool Purpose is to decrease hyper-motility CAUTION - SHOULD NOT BE USED MORE THAN 2 DAYS OR IF FEVER IS PRESENT!

Antacids - Anti Ulcer

Promotes ulcer healing by neutralizing HCI and reducing pepsin activity Needs good renal function - has lots of electrolytes; avoid giving with food or meds - decreases absorption

Prevacid (lansoprazole)

Proton pump inhibitor

Prilosec (omeprazole)

Proton pump inhibitor

damages or destroys thyroid tissus, limiting thyroid hormone secretion

RAI therapy

Scintigraphy

Radioactive isotopes are used to evaluate bleeding, displaced structures, organ size, and lesions.

A patient with newly diagnosed diabetes asks the nurse what to take for low blood sugar. What would be appropriate for the nurse to suggest?

Raisins

Novolog

Rapid acting insulin. Route and Dosage: 0.5 to 1 unit/kg/d; dose individualized. Half-life: 26 to 81 min. Onset: 5 to 15 min. Peak: 1 to 3 h Duration: 3 to 5 h. Action: To control diabetes mellitus; to lower blood sugar. Mode of Action: Insulin promotes use of glucose by body cells. Effects: Confusion, agitation, tremors, headache, flushing, hunger, weakness, lethargy, fatigue, hunger, weakness, lethargy, fatigue, urticaria; redness, irritation, or swelling at insulin injection site. Adverse reactions: tachycardia, palpitations, hypoglycemic reactions, rebound hyperglycemia. Life-threatening: shock, anaphylaxis. Nursing Interventions: Monitor vital signs (tachycardia can occur). Determine blood glucose levels and report changes. Monitor the clients HbA1c. Prepare a teaching plan based on client\'s knowledge of health problem, diet, and drug therapy. Teaching: Instruct client to report symptoms of hypoglycemic (insulin) reaction: headache, nervousness, sweating, tremors, rapid pulse; and symptoms of hyperglycemic reaction (diabetic acidosis): thirst, increased urine output. Advise client that hypoglycemic reactions are more likely to occur during the peak action time. Explain that orange juice, sugar-containing drinks may be used when hypoglycemic reactions begin. Teach family members to administer glycagon by injection of client has a hypoglycemic reaction. Inform client that certain herbs may react with insulin and oral antidiabetic drugs. Instruct client about the necessity for compliance with prescribed insulin therapy and diet. HbA1c provides most accurate picture of optimal diabetic control. Advise client to carry a MedicAlert card, tag, or bracelet. Instruct client how to check blood glucose with a glucometer.

Humalog

Rapid acting insulin. Trade name: Lispro. Dosage: Varies according to client\'s blood sugar. Absorption: absorbed from subQ injection site. Onset: SubQ 5 to 15 min. Peak: 30 to 60 min. Duration: 3 to 4 hrs. Action: To control diabetes mellitus; to lower blood sugar. Mode of Action: Insulin promotes use of glucose by body cells. Side Effects: Confusion, agitation, tremors, headache, flushing, hunger, weakness, lethargy, fatigue, hunger, weakness, lethargy, fatigue, urticaria; redness, irritation, or swelling at insulin injection site. Adverse reactions: tachycardia, palpitations, hypoglycemic reactions, rebound hyperglycemia. Life-threatening: shock, anaphylaxis. Teaching: Humalog acts faster than regular insulin, so they MUST NOT be administered more than 5 minutes before mealtime. Clients who are insulin-dependent and who take rapid-acting insulin usually require intermediate-acting insulin as well.

nystagmus

Rapid involuntary eye movements.

Apidra (insulin glulisine)

Rapid-acting insulin

Humalog (insulin lispro)

Rapid-acting insulin

Novolog (insulin aspart)

Rapid-acting insulin

Pheochromocytoma

Rare tumor of he adrenal system that secretes catecholamines. Causes hypersecretion of epinephrine and norepinephrine Therapeutic measures-alpha-blocking meds phenoxybenzamine (Dibenzaline) to dilate blood vessels to control hypertension (blocks epinephrine)

Psychogenic

Related to the mind.

Miscellaneous - Anti-diarrheal

Replenish Normal GI flora Lactobacillus

Common causes/ risk factors: Atelectasis

Respiratory complication that may be the result of a blocked airway, diminished surfactant, or mucus plug. Recent general anesthesia, shallow breathing, respiratory muscle weakness and immobility are common risk factors.

Definition: Atelectasis

Respiratory complication when the alveoli within the lung becomes deflated, resulting in a complete or partial collapse of a lung.

Which of the following nursing diagnoses is the priority diagnosis for a patient with Ménière's disease? Disturbed Sensory Perception: Auditory Disturbed Sleep Pattern Risk for Trauma Social Isolation

Risk for Trauma Rationale: The risk for trauma in patients with inner ear disorders, such as Ménière's disease, is great. Disturbed Sensory Perception: Auditory is a priority nursing diagnosis for patients with hearing loss. Sleep Pattern Disturbance is a priority nursing diagnosis for patients with tinnitus. Risk for Trauma is the priority nursing diagnosis for patients with inner ear disturbances causing balance disorders. Social Isolation is a priority diagnosis for patients with otosclerosis or presbycusis.

Clozaril

SCHIZOPHRENIA ATYPICAL/DOPAMINE SYSTEM STABILIZER (DDS) 2nd generation DOC* treat+ and-sx; less EPS se: Report sore throat, Check WBCS

thorazine, Halodol

SCHIZOPHRENIA PHENOTHIAZIDES 1st generationl phenothiazide-like (neuroleptics) wide safety margin; OD unlikely Avoid caffeine se: EPS: Parkinson-like rigid shuffle (Rx with anticholinergics, dystonia, tardive dyskinesia (Lip smacking), akathisia (Pacing) photosensitivity

disorders of the posterior pituitary gland

SIADH and DI

Diabetes Summary

SIGNS AND SYMPTOMS: Polyuria Polydipsia Polyphagia Fatigue Blurred Vision Headache Abdominal pain DIAGNOSTIC TESTS: Fasting Plasma Glucose HbA1c (glycosylated hemoglobin) Oral glucose tolerance test Additional testing for complications THERAPUTIC MEASURES: Nutrition Therapy Exercise Insulin Oral hypoglycemic medication Self monitoring of blood glucose levels Education COMPLICATIONS: Hypoglycemia Hyperglycemia Diabetic ketoacidosis Hyperosmolar hyperglycemia Long-term complications PRIORITY NURSING DIAGNOSIS: Risk for Unstable Blood Glucose Level

Cymbalta, Effexor

SNRI selevtive NE reuptake inhibitor se: confusion, tremor, sweating (With too high dose) check liver failure, suicide, seizures (with too high dose) BLACK BOX WARNING-> suicide rate/ warnings

disopyramide/ Norpace

SODIUM CHANNEL BLOCKERS (CLASS I) dysrhythmias; Class 1A; usually reserved for serious ventricular dysrhythmias

lidocaine/ Xylocaine

SODIUM CHANNEL BLOCKERS (CLASS I) dysrhythmias; Class 1B; usually reserved for rapid control of ventricular dysrhythmias; also widely used as a local anesthetic

Celexa, Prozac, Zoloft, Paxil, Lexapro

SSRI most used 1970s; slows reuptake of serotonin; post synapse more sensitive; DOC* se: weight gain, 70% have sexual dysfunction; decrease libido NI: drink fluids; add fiber; AVOID ST. JOHN'S WORT

Oral hypoglycemic agents INSULIN STIMULATORS

SULFONYUREAS- Stimulate insulin secretion by pancreas, increase insulin receptor sensitivity EXAMPLES- glipizide (Glucotrol) glimepride (Amaryl) glyburide (Micronase, Diabeta) S/E- Hypoglycemia, weight gain, possible increased risk of cardiovascular disease NURSING IMPLICATIONS: -Monitor patient for hypoglycemia -Teach patient to avoid alcohol

Neuropathy

SYMPTOM- Burning pain in legs and feet

Infection

SYMPTOM- Fever

Gastroparesis

SYMPTOM- Food intolerance

Retinopathy

SYMPTOM- Impaired vision

Diabetic Ketoacidosis

SYMPTOM- Ketones in the blood and urine

Nephropathy

SYMPTOM- Microalbuminuria

Hyperosmolar hyperglycemia

SYMPTOM- Profound hyperglycemia without ketonemia

Examples of Osmotic Laxative - Saline

Salts Salines Lactulose* Glycerine GoLytely**

Example of Anticholinergics - Antiemetic

Scopolamine

vertigo

Sensation of whirling or rotation.

Retinal detachment

Separation of the retina or sensory portion of the eye from the choroid.

A patient treated for Hodgkin's disease and reports feeling better but has gained weight and has muscle cramps. The nurse knows that syndrome of inappropriate antidiuretic hormone can be a complication of cancer and cancer treatment. Which lab results would further assess the possibility?

Serum calcium and phosphate

Humulin R

Short-acting insulin

Novolin R

Short-acting insulin

Example of Pepsin Inhibitor - Anti Ulcer

Sucralfate (Carafate)

Endoscopic ultrasonography (EUS)

Small ultrasound transducer on the tip of an endoscope displays images of a targeted GI area. Gas, bone, and adipose tissue are not issues, as with traditional ultrasonography.

Which meal plan is best for the patient with reactive hypoglycemia?

Small, frequent meals

Example of Antacids - Systemic Effect

Sodium Bicarbonate - Alka Seltzer Calcium Carbonate - Tums

Side Effects of Systemic Antacids

Sodium Excess - Water Retention Metabolic Alkalosis, Acid Rebound

Cathartics

Soft to watery stool with cramping

overproduction and oversecretion of ADH

Syndrome of Inappropriate antidiuretic hormone (SIADH)

Causes of Diarrhea

Spicy or spoiled food Bacteria (E.Coili), Virus, Toxins Drug Reactions Fecal Impaction, Laxative Abuse Malabsorption disorders, bowel tumor, IBD Stress, Anxiety

Hordeolum (sty)

Staphylococcal abscess that may occur on either the external or internal margin of the lid.

What is the main drug in the Meglitinide Analouges class?

Starlix/Nateglinide: this is a drug that stimulates insulin secretion within 20 minutes of ingestion. Must give 20 minutes before eating.

Crestor-

Statin. Trade name: Rosuvastatin calcium. Dosage: PO: initially 5 to 10 mg/d; Maint: 5 to 40 mg/d. Action: Strong statin drug that cuts LDL in half. Reduces total cholesterol, LDL, and triglycerides. Increases HDL. Mode of Action: Inhibit the enzyme HMG CoA reductase in cholesterol biosynthesis. By inhibiting cholesterol synthesis int he liver, it decreases the concentration of cholesterol, decreases LDL, and slightly increases HDL cholesterol. Side effects: GI disturbances, headaches, muscle cramps, and tiredness. Nursing Implications: Serum liver enzymes should be monitored, and an annual eye examination is needed, because cataract formation may result. Teaching: The client should report immediately any muscle aches or weakness, which can lead to rhabdomyolysis, a muscle disintegration that can become fatal. The client should understand that antihyperlipidemic drug therapy is a lifetime commitment for maintaing a decrease in serum lipid levels. Abruptly stopping the statin drug could cause the client to have a threefold rebound effect that may cause death from an AMI.

Absorbents - Anti-diarrheal

Sticks to the toxins or bacteria that are causing the diarrhea. Coats the walls of GI tract. Combination agents - Parepectolin (paragoric, Koelin-pectin)

Enophthalmos

Sunken appearance to the eyes.

The patient is in the preoperative holding area, having been brought to surgery from the Emergency Department because of a detached retina. The tear has occurred in the right posterior aspect of the retina. The nurse places the patient in which of the following positions? At a 45-degree angle (semi-Fowler's position) At a 90-degree angle (high Fowler's position) Prone Supine

Supine Rationale: The patient's head is positioned so that gravity pulls the detached portion of the retina into closer contact with the choroid. In this example, the patient's head should be flat and turned slightly to the right.

Miscellaneous - Antiemetic

Suppresses impulses to CTZ

Proton Pump Inhibitor or PPis - Anti Ulcer

Supress gastric acid secretion by inhibiting the hydrogen/potassium ATPase enzyme system located in the gastric parietal cells. Blocks the final step of HCI production. Take before meals; pretty much wipe out HCl

A person with type 2 diabetes mellitus is scheduled for hip surgery and is upset because an infusion of glucose and insulin is being started; this is a change from the patients usual oral hypoglycemic therapy. What should the appropriate response from the nurse be?

Surgery is a stressor that can alter blood glucose control and the IV medications are likely only temporary.

tympanoplasty

Surgical reconstruction of the middle ear.

enucleation

Surgical removal of an eye.

Mastoidectomy

Surgical removal of infected mastoid air cells.

Labyrinthectomy

Surgical removal of the labyrinth.

Hypophysectomy

Surgical removal of the pituitary gland

Hypophysectomy

Surgical removal of the pituitary gland. POST-OP- Monitor dressing for CSF. (CSF has glucose so drainage checked by a BS test with a glucose strip Use stool softeners and antitussives to prevent straining

Oral hypoglycemic agents INSULIN SENSITIZERS

THIAZOLIDINEDIONES (glitazones)- reduce insulin resistancein muscles, Improve blood lipids, may lower blood pressure and improve cardiovascular risk EXAMPLES- pioglitazone (Actos) rosiglitizone (Avandia) S/E- Nausea, weight gain, fluid retention NURSING IMPLICATIONS: -Give with meals -Work well with obese patients -Avoid with liver disease; monitor liver enzymes -Monitor for heart failure -May alter effectiveness of some birth control pills.

Symptoms of thyrotoxic crisis

Tachycardia, high fever, hypertension, dehydration, restlessness, delirium, coma, death if untreated

Symptoms of hyperthyroidism

Tachycardia; heat intolerance; fatigue; restlessness; tremor; emotional instability; insomnia; warm, moist skin; increased appetite, weight loss; frequent stools; decreased serum lipids; decreased libido; erectile dysfunction

A spry 77-year-old female patient has been diagnosed with simple glaucoma. She lives alone, takes medications for asthma, is on a fixed income, and walks to Catholic Mass on Sunday mornings. The nurse must be certain that the ophthalmologist is aware of which of the aforementioned facts about the patient? Lives alone. Takes medications for asthma. Lives on a fixed income. Walks to Catholic Mass on Sundays.

Takes medications for asthma. Rationale: Living alone and a fixed income should be concerns addressed through a Social Services consultation. The patient could be listed on a clergy visit list if hospitalized. Most importantly, the nurse wants to be certain that the patient takes medications for asthma because beta-adrenergic blockers are used topically to treat glaucoma. In asthmatic patients, beta-adrenergic blockers can cause bronchospasm, and are contraindicated

Which of the following assessment findings in a patient with hyperthyroidism being treated fro pneumonia should the nurse report to the physician immediately?

Temperature of 104 degrees and Tachycardia

If a tumor is surgically removed...

Test electrolyte measures

In the surgical setting, where is the center of the sterile field?

The center of the sterile field is the site of the surgical incision.

diabetic retinopathy

The collective name for the changes in the retina that occur in the person with diabetes. The retinal capillary structure undergoes alterations in blood flow, leading to retinal ischemia and a breakdown in the blood retinal barrier.

sulfonylurea; hypoglycemia

The main implication for this type of oral antidiabetic is to ______. (two answers)

The nurse is caring for the patient 6-hours post-transsphenoidal hypophysectomy. What sign should the nurse report to the physician immediately?

The mustache dressing tests positive for glucose.

What are some common RN interventions to prevent/ minimize paralytic ileus?

The nurse can prevent/minimize paralytic ileus after surgery with pt positioning and early ambulation. Evidence of bowel function returning to normal includes auscultation of bowel sounds and passing of flatus and stool.

In the administration of a drug such as levothyroxine (Synthroid), the nurse must teach the client: (Select all that apply.) a.) Therapy could take three weeks or longer. b.) Periodic lab tests for T4 levels are required. c.) Report weight loss, anxiety, insomnia, and palpitations. d.) Jaundice

Therapy could take three weeks or longer. Periodic lab tests for T4 levels are required. Report weight loss, anxiety, insomnia, and palpitations. A,B,C

How do the Glucosidase inhibitors work, and what meds are included in this class?

These include Acarbose, Precose and Miglitol, Glyset. They slow intestinal digestion and absorption of carbohydrates and lower blood glucose levels. The side effects include diarrhea and abdominal discomfort.

How do the THZ type meds work and what meds are in this class?

These meds include Avandia and Actos. They make the body more sensitive to insulin - also known as "insulin sensitizers", however they can cause liver damage and so the patient will need to get a liver function studies test done before taking. These meds can also reduce the effectiveness of oral contraceptives

Actos (pioglitazone)

Thiazolidinedione antidiabetic

Describe "Somogyi Phenomenon"

This is a morning hyperglycemia secondary to an effective counterregulatory response to nighttime hypoglycemia. Treatment: give adequate dietary intake at bedtime and evaluate insulin dose and exercise programs that may have led to hypoglycemia. Must monitor blood glucose during the night!

Januvia

This medication addresses beta cell dysfunction and uncontrolled glucose production by the liver because of alpha and beta cell dysfunction.

exenetide (Byetta)

This new DM medication is in the class of incretin mimetics. It is often given with sulfonylureas and the biguanide: metformin

sitagliptin phosphate (Januvia)

This new diabetes medication inhibits the dipeptidyl peptidase-4 (DPP-4) enzyme which plays a major role in glucose metabolism.

pramlintide (Symlin)

This new diabetes medication is a synthetic analog of human amylin.

exenetide (Byetta)

This new diabetes medication is injected before morning and evening meals

pramlintide (Symlin)

This new diabetes medication is used with insulin when insulin alone hasn't achieved adequate glucose control.

exenetide (Byetta)

This new diabetes medication mimics the effects of naturally occurring hormones from the intestines.

pramlintide (Symlin)

This new diabetes medication slows gastric empyting, suppresses glucagon secretion, and is injected before meals.

rapid-acting (Humalog or novolog)

This type of insulin has an onset of action in 10-15 minutes.

very long-acting (lantus, levemir)

This type of insulin has an onset of action in 1h.

intermediate-acting (NPH, Novolin N, Humulin N)

This type of insulin has an onset of action in 2-4h.

short-acting (Novolin R, Regular)

This type of insulin has an onset of action in 30-60 minutes.

long-acting (Ultralente)

This type of insulin has an onset of action in 6-8 hours.

very long-acting (lantus, levemir)

This type of insulin has no peak.

intermediate-acting (NPH, Novolin N, Humulin N)

This type of insulin lasts 16-20 hours.

rapid-acting (Humalog or novolog)

This type of insulin lasts 2-4 hours.

long-acting (Ultralente)

This type of insulin lasts 20-30 hours.

very long-acting (lantus, levemir)

This type of insulin lasts 24 hours.

short-acting (Novolin R, Regular)

This type of insulin lasts 4-6 hours.

rapid-acting (Humalog or novolog)

This type of insulin peaks in 1 hour.

short-acting (Novolin R, Regular)

This type of insulin peaks in 2-3 hours.

long-acting (Ultralente)

This type of insulin peaks within 12-16 hours.

intermediate-acting (NPH, Novolin N, Humulin N)

This type of insulin peaks within 4-12 hours.

biguanide

This type of oral antidiabetic is associated with hypoglycemia and GI disturbances

alpha-glucosidase inhibitor

This type of oral antidiabetic is associated with hypoglycemia and GI side-effects, especially gas

non-sulfonylurea insulin secretagogues

This type of oral antidiabetic is associated with hypoglycemia and weight gain.

thiazolidinedione

This type of oral antidiabetic is associated with hypoglycemia, anemia, and weight gain

Sulfonylurea

This type of oral antidiabetic is associated with hypoglycemia, weight gain, and mild GI symptoms

thiazolidinedione

This type of oral antidiabetic is used in type II diabetics who already take insulin.

alpha-glucosidase inhibitors

This type of oral antidiabetic is used to delay absorption of complex carbohydrates.

biguanides

This type of oral antidiabetic is used to inhibit production of glucose by the liver.

thiazolidinediones

This type of oral antidiabetic is used to sensitize body tissues to insulin

1st generation sulfonylurea

This type of oral antidiabetic is used to stimulate beta cells to secrete insulin, but it's not used often anymore.

2nd generation sulfonylurea

This type of oral antidiabetic is used to stimulate beta cells to secrete insulin.

non-sulfonylurea insulin secretagogue

This type of oral antidiabetic is used to stimulate the pancreas to secrete insulin.

non-sulfonylurea insulin secretagogue

This type of oral antidiabetic should be taken with meals. It has a short half life.

alpha-glucosidase inhibitor

This type of oral antidiabetic should be taken with the first bite of food in order to be effective.

biguanide

This type of oral antidiabetic should not be given within 48 hours (Pre and post) of a contrast dye.

Examples of Dopamine Antagonists - Antiemetic

Thorazine, compazine, verpirin (Phenergran sometimes here) Butyrophenones - Haldol, Inspsine Metaclopramide - Reglan

The nurse is caring for a patient with Type 1 diabetes mellitus being treated for pneumonia. Which lab value should the nurse monitor most closely to identify potential complications early?

Urine Ketones

levothyroxine sodium

Thryoid Hormone Synthroid and T4

Thyroshield (SSKI)

Thyroid-blocking agent

The Diabetes Control and Complications Trial (DCCT) found that individuals who maintain _______ control of their diabetes will have fewer long-term complications.

Tight

Therapeutic effects/uses of Prednisone

To decrease inflammatory occurence; as an immunosupressant; to treat dermatological disorders Action: suppression of inflammation and adrenal function

Slow metabolism

To little T3 and T4 can lead to?

Goal in management of DM is...

To maintain sugar levels in the 70 -125 mg/dL range

Fast metabolism

To much T3 and T4 can lead to?

Therapeutic effects and uses of Synthroid

To treat hypothyroidism, myxedema, and cretinism Mode of Action: Increase metabolic rate, O2 consumption, and body growth

Treatment for glaucoma? What medicine do you avoid with glaucoma?

Treatment - meds like pilocarpine to decrease IOP, also miotics, diuretics like diamox; Surgery - peripheral iridectomy AVOID - mydriatics (dilation of pupil), heavy lifting, straining

Hypoglycemia symptoms

Tremor Irritability Sweating

Pheochromocytoma

Tumor in the adrenal medulla

A 26 yr old patient is admitted to the hospital with a new diagnosis of diabetes, a blood glucose of 690 mg/dL, and ketones in the blood and urine. Which type of diabetes should the nurse expect?

Type 1

Side Effects of Histamine 2 Blockers - Anti Ulcer

Vertigo / Dizziness, headache, nausea, diarrhea or constipation, depression, confusion, rash, blurred vision, impotence

What are the classic signs and symptoms of type 1 diabetes? How does it differ from type 2?

Type 1 diabetes usually occurs at a much younger age - generally younger than age 30. It only accounts for about 10% of the diabetes population and is an autoimmune response - destruction of pancreatic beta cells. Type 1s MUST take insulin as they have absolutely no way to make insulin at all. S/S: polyuria, polydipsia, polyphagia, weight loss, nausea, vomiting, ketones in urine because of fat breakdown; * if conversion of fatty acids to ketone bodies collect in blood this can cause metabolic acidosis * dehydration is also a symptom and leads to hemoconcentration of the blood and hyperviscosity, hypo-perfusion, which is a decreased circulation of blood to the tissues, and hypoxia, which is poor tissue oxygenation especially to the brain. All of this causes lactic acid to increase and causes more acidosis~!

Difference between Type 1 and Type 2 diabetes is...

Type 1-Must have insulin because body is not producing it Type 2- Body not producing enough and body needs the glucose for energy

Which type of diabetes is most common?

Type 2 Diabetes

Describe Type 2 diabetes

Type 2 diabetes usually has a gradual onset. It is most common among middle-aged and older adults who are obese or overweight. Heredity does play a role as offspring have a 15% greater chance of developing the disease - it is a dominant inheritance. The age of onset peaks in the 50s, and 60-80% are obese. - Insulin continues to be produced in the body, but the insulin receptors become desensitized and eventually do not respond to the insulin even though the body may be pumping out great amounts of insulin. OR there may be decreased insulin secretion as well. With a type 2 diabetic, it can go either way.

how to diagnosis SIADH

UA and serum osmolality

Causes of hypoglycemic episodes in diabetes

Under-eating, skipping a meal, too much medication, too much exercise, poor timing of medication, food, exercise

Contra-Indictation of Laxative Use

Undiagnosed Abdominal Pain Inflammatory disorders of GI tract (Appendictis, Diverticulitis, Ulcerative Colitis) Spastic Colon Bowel Obstruction Pregnancy

1. increase in serum T4 (T4 greater than 12; T3 greater than 220) 2. Low TSH

What are the diagnostic findings in hyperthyroidism?

The nurse is caring for a patient who has a hearing deficit. Which of the following techniques should be implemented by the nurse to improve communication? (Select all that apply.) Keep the patient's face in full light. Use a low voice pitch with normal loudness. Speak at a slower rate than usual. Use short sentences.

Use a low voice pitch with normal loudness. Use short sentences. Rationale: Techniques to improve communication with a patient with a hearing loss include keeping one's face in full light-placing light in the patient's face will prevent patient from seeing the speaker; using a low voice pitch with normal loudness; speaking at a normal rate, and not overarticulating; and using short sentences, and pausing at the end of each sentence.

Patient education for patients receiving low doses of radioactive iodine at home

Use agency guidelines for education. In general, teach patient to avoid close contact with family for about a week; to use careful handwashing after urinating; to avoid oral contact with others; to wash eating utensils carefully; to treat sore throat with acetaminophen or lozenges; to increase oral fluid intake. Teach signs and symptoms of hypo- and hyperthyroidism to report.

midazolam/ Versed

Use: conscious sedation; provides sedation of mechanically ventilated patients during anesthesia or in a critical care setting NI: monitor bp, pulse, and resp. Oxygen and resuscitative equipment should be immediately available Toxicity and overdose: if overdose occurs, monitor pulse, resp and bp continuously. maintain patent airway and assist ventilation as needed if hypotension occurs, treatment includes IV fluids, repositioning, and vasopressors

diazepam/ Valium

Use: relief of anxiety; sedation & skeletal muscle relaxation NI: monitor vitals, assess risk of falls, mental status & level of sedation. Toxicity and overdose: flumazenil is an adjunct in the management of toxicity or overdose (may induce seizures)

Benzodiazepines - Antiemetic

Used in combination with glucocorticoid and serotonin 5-HT3 receptor antagonist Used in cancer chemo (not as often)

Oral Glucose tolerance test, when is this test used to diagnose for diabetes?

Used when person is pregnant to diagnose gestational diabetes, however not the test of choice because it is costly and inconvenient compared with fasting blood glucose measures.

Glucocorticoids - Antiemetic

Used with Benzos in cancer chemo

Examples of Benzodiazepines - Antiemetic

Valium Ativan

doxycycline

Virbamycin Drug class: antibacterial: tetracycline Dose: systemic infection: A:PO/IV: 100 mg q12h on day 1, then 100/d C over 8: PO/IV: 4.4 mg/kg/d in 1-2 doses on day 1; then 2.2-4.4 mg/kg/d Action: inhibition of protein synthesis steps; bacteriostatic or cidal Contra: hypersensitivity, pregnancy, severe hepatic/renal disease Caution: history of allergies, renal/hepatic dysfunction, alcoholism, hypokalemia, antidysrhythmics, significant bradycardia Sides: nausea, vomit, diarrhea, rash, flatulence, abdom discomfort, headache, photosensitivity, pruritus, heartburn, color vision changes adverse: superinfections, severe photosensitivity, blood dyscrasias, hepatotoxicity, intracranial hypertension, pseudomembranous colitis

Fiberoptic colonoscopy

Virtual (CT) colonoscopy; colon is distended with air and CT scanning provides a computer-simulated perspective. The scope takes still and video images and may remove polyps.

Lower GI endoscopy

Visualization of the anus, rectum, sigmoid, and descending colon using a flexible scope to take still and video images and sometimes biopsies.

What assessment findings should the nurse monitor to detect the onset of thyrotoxicosis in a patient with hyperthyroidism?

Vital Signs (Thyrotoxicosis causes increases in BP, pulse, temp and respirations)

What data is important for the nurse to monitor in a patient with a pheochromocytoma?

Vital signs

Which of the following nursing assessments is most important in the patient with hyperthyroidism and risk for thyrotoxic crisis.

Vital signs - Elevated vital signs can signal the onset of thyrotoxic crisis

Examples of Miscellaneous - Antiemetic

Vontrol Tigan

Nursing interventions to prevent amputation in DM

Wash and dry feet daily in warm water, Apply non-alcohol-based lotion, Inspect for lesions daily, Wear cotton socks and leather shoes, Avoid crossing legs and wearing constricting clothing, Use care when cutting toenails, Have feet checked regularly by a health professional

When the visiting nurse is teaching the family of a patient with acute conjunctivitis regarding the disease process, the nurse emphasizes which of the following statements as most important? Wash hands before and after instilling eye drops. Cleanse the patient's contact lenses twice each day. Ask patient to gently rub eyes to circulate the eye drops after instillation. Use a cotton-tipped swab to cleanse the right eye and then the left eye.

Wash hands before and after instilling eye drops. Rationale: Acute conjunctivitis is highly contagious. Teach the family to wash hands thoroughly before and after instilling eye medication. Contact lens use should be avoided until the infection has cleared. Rubbing the eyes increases the risk of corneal trauma. Using a new swab between eyes prevents cross-contamination.

Purgatives

Watery stools with cramping

A 26-year old female patient is hospitalized for radioactive iodine treatment for hyperthyroidism. Which of the following precautions by the nurse is appropriate?

Wear gloves when changing bedside commode. (Output will be radioactive.)

Symptoms of Cushing's syndrome

Weight gain, truncal obesity, buffalo hump, moon face, glucose intolerance, muscle wasting, thin skin, osteoporosis, risk for infection, mental status changes, sodium and water retention

1. High fever (above 101.3F) 2. Extreme tachycardia (>130) 3. Exaggerated hyperthryoid symptoms 4. Delirium psychosis, coma

What are the clinical manifestations of a thyroid storm? (4)

1. Nervousness 2. Palpitations 3. Poor heat tolerance, diaphoresis 4. Elderly - dry skin and pruritis 5. Exophthalmos 6. Increased appetite

What are the clinical manifestations of hyperthyroidism? (6)

1. Extreme fatigue 2. Hair loss 3. brittle nails 4. dry skin

What are the clinical manifestations of hypothyroidism? (4)

Often secondary to: l Hashimoto's thyroiditis l Malfunction of the pituitary gland l Thyroidectomy l Use of radioactive iodine

What is hypothyroidism secondary to? (4)

1. Pharmacologic therapy -Use of irradiation by administration of the radioisotope 123IAntithyroid medications 2. Propacil or Tapazole 3. Surgery

What is the medical management of hyperthyroidism? (3)

1. Pharmacologic Therapy Synthroid or Levothroid 2. Prevention of Medication Interactions 3. Gerontologic Considerations 4. Modifying Activity 5. Monitoring Physical Status 6. Promoting Physical Comfort

What is the medical management of hypothyroidism? (6)

1. Airway- possible external/ internal bleeding (swelling at site, must be patent, trach at bedside

What is the number one management of post thryoidectomy?

3. Parathyroid Glad- may develop hypocalcemia, muscle spasm, hyperreflexia, patient calcium levels down , may develop seizures

What is the number three management of post thryoidectomy?

2. Bleeding-Circulation: prevent extension of neck (support neck, interlock hands)

What is the number two management of post thryoidectomy?

1. Monitor vs, I/O 2. Check for constipation 3. Encourage p.o. fluids (3L/day) 4. Encourage physical activity 5. High fiber diet (fresh fruits and vegetables) 6. Avoid use enemas or laxatives

What is the nursing management of hypothyroidism? (6)

100 mg

What is the recommended daily dose of Januvia?

Too weak, has to convert to T3 outside the thyroid

What needs to happen to T4 in order to start functioning?

T3 and T4

What regulate metabolism and are necessary to function, energy, and growth

25-50 mL of 50% dextrose IV

What's the inpatient treatment choice of someone with hypoglycemia?

extremely expensive

What's the problem with the two alternate therapies for diabetes?

When is action of glucagon initiated in the body?

When blood glucose falls below 70mg/dl

Hyperplasia

When excess tissue develops in the endocrine glands and produces too much hormone

Ectopic

When hormones are made outside of the endocrine gland.

Ectopic hormones

When hormones are produced outside of the endocrine gland where they are normally produced

Euthyroid state

When the thyroid gland is functioning normally

Malnutrition

Worldwide, most common sign of dwarfism.

Can type 2 diabetes be prevented?

YES! it can be prevented or delayed by weight loss and increased physical activity

Example of Serotonin Receptor Antagonists - Antiemetic

Zofran

Hb A1c

a better predictor of blood glucose; shows BG levels over past 3 months; less than 6 is good, 6-7 is iffy, and over 7 is diabetes

Def: Diabetes Mellitus

a carbohydrate metabolism problem; due to insufficient insulin secretion

nephropathy

a disease affecting the kidneys

A physician prescribes levothyroxine sodium (Synthroid), 0.15 mg orally daily, for a client with hypothyroidism. The nurse will prepare to administer this medication: a) in the morning to prevent insomnia b) only when the client complains of fatigue and cold intolerance c) at various times during the day to prevent tolerance from occurring d) three times daily in equal doses of 0.5 mg each to ensure consistent serum drug levels

a) in the morning to prevent insomnia Levothyroxine (Synthroid) is a synthetic thyroid hormone that increases cellular metabolism. Levothyroxine should be given in the morning in a single dose to prevent insomnia and should be given at the same time each day to maintain an adequate drug level. Therefore, options B, C, and D are incorrect.

A nurse is monitoring a client newly diagnosed with diabetes mellitus for signs of complications. Which of the following, if exhibited in the client, would indicate hyperglycemia and warrant physician notification? a) polyuria b) diaphoresis c) hypertension d) increased pulse rate

a) polyuria Classic symptoms of hyperglycemia include polydipsia, polyuria, and polyphagia. Options B, C, and D are not signs of hyperglycemia.

Blood sugar is well controlled when Hemoglobin A1C is: a. Below 7% b. Between 12%-15% c. Less than 180 mg/dL d. Between 90 and 130 mg/dL

a. Below 7% A1c measures the percentage of hemoglobin that is glycated and determines average blood glucose during the two to three months prior to testing. Used as a diagnostic tool, A1C levels of 6.5% or higher on two tests indicate diabetes. A1C of 6% to 6.5% is considered prediabetes.

A resident of the nursing home has quite severe arthritis. When administering an analgesic to this elderly resident, the nurse should: a. Give the medication before the activity session in the day room. b. Give the medication when the resident states the pain is at 6 or higher on a 1-10 pain scale. c. Give the pain medication at mealtime. d. Make sure that the medication is not a narcotic.

a. Give the medication before the activity session in the day room.

A patient is admitted to the hospital with a diagnosis of Cushing syndrome. On physical assessment of the patient, the nurse would expect to find a. HTN, peripheral edema, and petechiae b. weight loss, buffalo hump, and moon face with acne c. abdominal and buttock striae, truncal obesity, and hypotension d. anorexia, signs of dehydration, and hyper pigmentation of the skin

a. HTN, peripheral edema, and petechiae (rationale- The effects of glucocorticoid excess include weight gain from accumulation and redistribution of adipose tissue, sodium and water retention, glucose intolerance, protein wasting, loss of bone structure, loss of collagen, and capillary fragility. Clinical manifestations of corticosteroid deficiency include hypotension, dehydration, weight loss, and hyperpigmentation of the skin.)

When caring for a patient with primary hyperaldosteronism, the nurse would question a physician's order for the use of a. Lasix b. amiloride (midamor) c. spironolactone (aldactone) d. aminoglutethimide (cytadren)

a. Lasix37 (rationale- hyperaldosteronism is an excess of aldosterone, which is manifested by sodium and water retention and potassium excretion. Lasix is a potassium-wasting diuretic that would increase the potassium deficiency. Aminoglutethimide blocks aldosterone synthesis; amiloride is apotassium-sparing diuretic; and spironolactone blocks mineralocorticoid receptors in the kidney, increasing secretion of sodium and water and retention of potassium.)

In the administration of hydrocortisone (Aeroseb-HC, Alphadern, Cetacort), it is vital that the nurse recognize that this drug might mask which symptoms? a.) Signs and symptoms of infection b.) Signs and symptoms of heart failure c.) Hearing loss d.) Skin infections

a.) Signs and symptoms of infection

A patient with acromegaly is treated with a transphenoidal hypophysectomy. Postoperatively, the nurse a. ensures that any clear nasal drainage is tested for glucose b. maintains the patient flat in bed to prevent cerebrospinal fluid leak c. assists the patient with toothbrushing Q4H to keep the surgical area clean d. encourages deep breathing and coughing to prevent respiratory complications

a. ensures that any clear nasal drainage is tested for glucose (Rationale- a transphenoidal hypophysectomy involves entry into the sella turcica through an incision in the upper lip and gingiva into the floor of the nose and the sphenoid sinuses. Postoperative clear nasal drainage with glucose content indicates CSF leakage from an open connection to the brain, putting the patient at risk for meningitis. After surgery, the patient is positioned with the head elevated to avoid pressure on the sella turcica, coughing and straining are avoided to prevent increased ICP and CSF leakage, and although mouth care is required Q4H toothbrushing should not be performed for 7-10post sx.)

Preoperative instructions for the patient scheduled for a subtotal thyroidectomy includes teaching the patient a. how to support the head with the hands when moving b. that coughing should due avoided to prevent pressure on the incision c. that the head and neck will need to remain immobile until the incision heals d. that any tingling around the lips or in the fingers after surgery is expected and temporary

a. how to support the head with the hands when moving (rationale- to prevent strain on the suture line postoperatively, the head must be manually supported while turning and moving in bed, but range-of-motion exercise for the head and neck are also taught preoperatively to be gradually implemented after surgery. There is no contraindication for coughing and deep breathing, and they should be carrier out postoperatively. Tingling around the lips or fingers is a sign of hypocalcemia, which may occur if the parathyroid glands are inadvertently removed during surgery, and should be reported immediately.)

During care of a patient with syndrome of inappropriate ADH (SIADH), the nurse should a. monitor neurologic status Q2H or more often if needed b. keep the head of the bed elevated to prevent ADH release c. teach the patient receiving treatment with diuretics to restrict sodium intake d. notify the physician if the patient's blood pressure decreases more than 20mmHg from baseline

a. monitor neurologic status Q2H or more often if needed Rationale- the patient with SIADH has marked dilution hyponatremia and should be monitored for decreased neurologic function and convulsions every 2 hours. ADH release is reduced by keeping the head of the bed flat to increase left atrial filling pressure, and sodium intake is supplemented because of hyponatremia and sodium loss caused by diuretics. A reduction in blood pressure indicates a reduction in total fluid volume and is an expected outcome of treatment.)

Antidote for heparin overdose a. protamine sulfate b. vitamin K c. vitamin E d. cyanocobolamine

a. protamine sulfate

The nurse determines that the patient in acute adrenal insufficiency is responding favorably to treatment when a. the patient appears alert and oriented b. the patient's urinary output has increased c. pulmonary edema is reduced as evidenced by clear lung sounds d. laboratory tests reveal serum elevations of K and glucose and a decrease in sodium

a. the patient appears alert and oriented (rationale- confusion, irritability, disorientation, or depressioni s often present in the patient with Addison's dz, and a positive response to therapy would be indicated by a return to alertness and orientation. Other indication of response to therapy would be a decreased urinary output, decreased serum potassium, and increased serum sodium and glucose. The patient with Addison's would be very dehydrated and volume-depleted and would not have pulmonary edema.)

Naloxone (Narcan) is administered to a client with severe respiratory depression and suspected drug overdose. After 20 minutes, the client remains unresponsive. The most likely explanation for this is: a.) The client did not use an opioid drug. b.) The dose of naloxone was inadequate. c.) The client is resistant to this drug. d.) The drug overdose is irreversible.

a.) The client did not use an opioid drug. If opioid antagonists (Naloxone) fail to reverse symptoms of respiratory depression quickly, the overdose was likely due to a non-opioid substance.

treatment options for hyperthyroidism

antithyroid meds radioactive iodine therapy subtotal thyroidectomy

hyperglycemia

abnormally high blood sugar usually associated with diabetes

hypoglycemia

abnormally low blood sugar usually resulting from excessive insulin or a poor diet

ketoacidosis

acidosis with an accumulation of ketone bodies *Patient will have a sweet taste in the back of the mouth.

condition caused by excessive secretion of GH characterized by an overgrowth of the bones and soft tissues

acromegaly

disorder of the anterior pituitary gland

acromegaly

excess GH in adult

acromegaly

neuropathy

any pathology of the peripheral nerves

Myxedema

advanced thyroid deficiency

Client teaching: GH

advise athletes not to take GH b/c of side effects. GH can be effective for children whose height is markedly below the expected norm. B/c GH acts on newly forming bone, it should be admin before the epiphyses have fused inform client with diabetes to closely monitor glycemia. Insulin regulation may be needed. Suggest client or family monitor client's growth

Client teaching: ACTH

advise client to adhere to drug regiment: discontinuation of certain drugs can cause hypofunction of the gland being stimulated Direct client to decrease salt intake to decrease or avoid edema. K supplement may be needed. Instruct client to report side effects like muscle weakness, edema, petechiae, ecchymosis, decrease in growth, decreased wound healing, and menstrual irregularities

what is insulin lipohypertrophy?

an increased swelling of fat that occurs at the side of repeated insulin injections - treatment is to rotate injection sites!

diphenhydramine/ Benadryl

anaphylaxis, mild nighttime sedation NI: may cause sedation and confusion d/t increased sensitivit to anticholinergic effects assess for confusion, delirium and other anticholinergic side effects and fall risk.

macular degeneration - Tx

antioxidants, Vit A B and zinc, bright lights, magnifiers, large print, photodynamic therapy (pt should avoid direct sunlight for 5 days post-procedure)

patient teaching after thyroid surgery (subtotal thyroidectomy)

assess for signs of iodine toxicity, practice coughing and deep breathing, leg exercises, support head manually, ROM exercises

most common cause of Addison's disease is

autoimmune response

Hashimoto's Disease

autoimmune thyroiditis is the most common cause of hypothyroidism

Your patient has been switched from valproic acid (Depakote) to gabepentin (Neurontin). Which of the following is a false statement? a. Gabapentin (Neurontin) is also used for bipolar disorder therapy b. Gabapentin (Neurontin) requires more frequent hepatic monitoring c. Gabapentin (Neurontin) is also used for migraine therapy d. Gabapentin (Neurontin) should not be given concurrently with antacids containing magnesium

b. Gabapentin (Neurontin) requires more frequent hepatic monitoring

Myxedema, which includes fatigue, general weakness, and muscle cramps, is a symptom of which endocrine disorder treated with levothyroxine (Synthroid)? a. Hyperthyroidism b. Hypothyroidism c. Cushing's syndrome d. Addison's disease

b. Hypothyroidism

Two days after surgery, an elderly client refuses a PRN dose of analgesic dose for fear of becoming "hooked." The nurse should respond by stating that: a. It is impossible to become hooked on PRN narcotics. b. Short-term use of narcotics is not likely to cause a person to become dependent on them. c. Side effects that occur in the elderly mean that medications will be discontinued as soon as possible. d. The elderly are least likely to become dependent on narcotics.

b. Short-term use of narcotics is not likely to cause a person to become dependent on them.

Your patient is taking valproic acid (Depakote). Which of the following is a false statement? a. Valproic acid requires hepatic monitoring b. Valproic acid has the lowest seizure relapse rate when discontinued c. Valproic acid is also used in migraine therapy d. Valproic acid is also used in bipolar disorder therapy

b. Valproic acid has the lowest seizure relapse rate when discontinued

Antiplatelet agents include all of the following except a. acetylsalicylic acid b. acetaminophen c. ticlopidine (Ticlid) d. dipyridamole (Persantine)

b. acetaminophen

When the patient with parathyroid disease experiences symptoms of hypocalcemia, a measure that can be used to temporarily raise serum calcium levels is to a. administer IV normal saline b. have the patient rebreathe in a paper bag c. administer Lasix as ordered d. administer oral phosphorous supplements

b. have the patient rebreathe in a paper bag (rationale- rebreathing in a paper bag promotes carbon dioxide retention in the blood, which lowers pH and creates an acidosis. An academia enhances the solubility and ionization of calcium, increasing the proportion of total body calcium available in physiologically active form and relieving the symptoms of hypocalcemia. Saline promotes calcium excretion, as does Lasix. Phosphate levels in the blood are reciprocal to calcium and an increase in phosphate promotes calcium excretion.)

An appropriate nursing intervention for the patient with hyperparathyroidism is to a. pad side rails as a seizure precaution b. increase fluid intake to 3000 to 4000ml/day c. maintain bed rest to prevent pathologic fractures d. monitor the patient for Trousseau's phenomenon or Chvostek's sign

b. increase fluid intake to 3000 to 4000ml/day (Rationale-A high fluid intake is indicated in hyperparathyroidism to dilute hypercalcemia and flush the kidneys so that calcium stone formation is reduced.)

A patient with SIADH is treated with water restriction and administration of IV fluids. The nurses evaluates that treatment has been effective when the patient experiences a. increased urine output, decreased serum sodium, and increased urine specific gravity b. increased urine output, increased serum sodium, and decreased urine specific gravity c. decreased urine output, increased serum sodium, and decreased urine specific gravity d. decreased urine output, decreased serum sodium, and increased urine specific gravity

b. increased urine output, increased serum sodium, and decreased urine specific gravity (rationale- the patient with SIADH has water retention with hyponatremia, decreased urine output and concentrated urine with high specific gravity. improvement in the patient's condition reflected by increased urine output, normalization of serum sodium, and more water in the urine, decreasing the specific gravity.)

When caring for a patient with nephrogenic DI, the nurse would expect treatment to include a. fluid restriction b. thiazide diuretics c. a high-sodium diet d. chlorpropamide (DIabinese)

b. thiazide diuretics (Rationale- in nephrogenic Di the kidney is unable to respond to ADH, so vasopressin or hormone analogs are not effective. Thiazide diuretics slow the glomerular filtration rate in the kidney and produce a decrease in urine output. Low-sodium diets are also thought to decrease urine output. Fluids are not restricted, because the patient could become easily dehydrated.)

The most important nursing intervention during the medical and surgical treatment of the patient with a pheochromocytoma is a. administering IV fluids b. monitoring blood pressure c. monitoring I&O and daily weights d. administering B-adrenergic blocking agents

b. monitoring blood pressure38 (rationale- a pheochromocytoma is a catecholamine-producing tumor of the adrenal medulla, which may cause severe, episodic HTN; severe, pounding headache; and profuse sweating. Monitoring for dangerously high BP before surgery is critical, as is monitoring for BP fluctuation during medical and surgical tx.)

Your patient has been stabilized taking only primidone (Mysoline). Which drug besides primidone may be assayed during his stay in the hospital to monitor his therapy? a. pentobarbital b. phenobarbital c. valproic acid d. phenytoin

b. phenobarbital

A patient with hypothyroidism is treated with Synthroid. When teaching the patient about the therapy, the nurse a. explains that caloric intake must be reduced when drug therapy is started b. provides written instruction for all information related to the medication therapy c. assures the patient that a return to normal function will occur with replacement therapy d. informs the patient that medications must be taken until hormone balance is reestablished

b. provides written instruction for all information related to the medication therapy (rationale- because of the mental sluggishness, inattentiveness, and memory loss that occur with hypothyroidism, it is important to provide written instructions and repeat information when teaching the patient. Caloric intake can be increased when drug therapy is started, because of an increased metabolic rate, and replacement therapy must be taken for life. Although most patients return to a normal state with treatment, cardiovascular conditions and psychoses may persist.)

The nurse teaches the client relaxation techniques and guided imagery as an adjunct to medication for treatment of pain. The nurse explains that the major benefit of these techniques is that they: a.) Are less costly. b.) Allow lower doses of drugs with fewer side effects. c.) Can be used at home or in any environment. d.) Do not require self-injection.

b.) Allow lower doses of drugs with fewer side effects. When used concurrently with medication, non-pharmacologic techniques can allow for lower doses, and possibly fewer drug-related adverse effects. The other options also are advantages to guided imagery and relaxation, but not the major one.

Of what precautions should a client receiving radioactive iodine-131 be made aware? a.) Drink plenty of fluids, especially those high in calcium. b.) Avoid close contact with children or pregnant women for one week after administration of drug. c.) Be aware of the symptoms of tachycardia, increased metabolic rate, and anxiety. d.) Wear a mask if around children or pregnant women.

b.) Avoid close contact with children or pregnant women for one week after administration of drug.

When hydrocortisone use is discontinued, the nurse must recognize the possibility of what side effect, if this drug is stopped abruptly? a.) Development of myxedema b.) Circulatory collapse c.) Development of Cushing's syndrome d.) Development of diabetes insipidus

b.) Circulatory collapse

Nursing intervention for a client receiving opioid analgesics over an extended period of time should include: a.) Referring the client to a drug treatment center. b.) Encouraging increased fluids and fiber in the diet. c.) Monitoring for G.I. bleeding. d.0 Teaching the client to take her own blood pressure.

b.) Encouraging increased fluids and fiber in the diet. Opioids suppress intestinal contractility, increase anal sphincter tone, and inhibit fluids into the intestines, which can lead to constipation. There is nothing to indicate the drug is related to addiction problems. Opioids do not cause GI bleeding.

Which disease is characterized by increased body metabolism, tachycardia, increased body temperature, and anxiety, and treated with Prophylthiouracil (PTU)? a.) Hashimoto's thyroiditis b.) Graves' disease c.) Addison's disease d.) Cushing's syndrome

b.) Graves' disease

A client who is taking levothyroxine (Synthroid) begins to develop weight loss, diarrhea, and intolerance. The nurse should be aware that this might be an indication of what hormonal condition? a.) Addison's disease b.) Hyperthyroidism c.) Cushing's syndrome d.) Development of acromegaly

b.) Hyperthyroidism

The client is prescribed ketorolac tromethamine (Toradol) for treatment of pain following a surgical procedure. The nurse should question which of the following drug orders? a.) Toradol 10 mg p.o. b.i.d. b.) Toradol 20 mg p.o. b.i.d c.) Toradol 5 mg p.o. t.i.d. d.) Toradol 20 mg p.o q.i.d

b.) Toradol 20 mg p.o. b.i.d The maximum daily dose of Toradol is 40 mg.

propofol/ Dipravan

balanced anesthesia; sedation of intubated, mechanically ventilated patients in intensive care units NI: assess resp. status, pulse, bp continuously. frequently causes apnea >60 sec. maintain patent airway and adequate ventilation. should be used only by individuals experienced in endotracheal intubation, and equipment for this procedure should be readily available; monitor for propofol infusion syndrome (severe metabolic acidosis, hyperkalemia, lipidema) Toxicity and overdose: monitor vitals maintain airway and assist ventilation

Seconal-

barbituate. Trade name: Seconal Sodium. Dosage: PO: 100 to 300 mg/d Half Life: 15 to 40 hours. Peak: 0.5 to 1 h Action: To treat insomnia; To induce sleep for those who have difficulty falling asleep; used for sedation, preoperative medication. Mode of Action: Depression of the CNS, including the motor and sensory activities. Side effects: Lethargy, drowsiness, hangover, dizziness, paradoxical excitement in older adults. Adverse reactions: Drug dependence or tolerance. Life threatening- respiratory distress. Contraindications: Respiratory depression, severe hepatic disease, pregnancy, nephrosis, hypersensitivity. Caution: Liver or kidney dysfunction; older adults, children, and debilitated individuals. Nursing Implications: Vital signs (respiratory and blood pressure) should be closely monitored. Might result in drug abuse. Raise bedrails of older adults and clients- confusion may occur. Check client\'s skin for rashes. Teaching: These drugs may cause the person to awaken early in the morning. Teach client to use nonpharmacologic ways to induce sleep. Avoid alcohol and antidepressant, antipsychotic, and narcotic drugs. Certain herbs may interact with CNS depressants. Advise client to not drive a motor vehicle. Take hypnotic 30 minutes before bed. Abrupt cessation of the hypnotic may result in withdrawal symptoms. Hangover may occur.

importance of Adrenals

becomes stressed when sick and produces cortisol; regulates stress response

when check BG?

before eating

overproduction of growth hormone is usually caused by

benign pituitary tumor (adenoma)

An elderly client had abdominal surgery six hours earlier. When the nurse asks the client about pain, the client responds that there is none. The best intervention on the part of the nurse is: a. Administer a PRN dose of IV pain medication as ordered. b. Assist the client into a sitting position in preparation for ambulation. c. Question the client further about discomfort to assess the meaning of pain. d. Assess the abdominal dressing and consult the surgeon about findings.

c. Question the client further about discomfort to assess the meaning of pain.

Which of the following diabetes drugs acts by decreasing the amount of glucose produced by the liver? a. Sulfonylureas b. Meglitinides c. Biguanides d. Alpha-glucosidase inhibitors

c. Biguanides Biguanides, such as metformin, lower blood glucose by reducing the amount of glucose produced by the liver. Sulfonylureas and Meglitinides stimulate the beta cells of the pancreas to produce more insulin. Alpha-glucosidase inhibitors block the breakdown of starches and some sugars, which helps to reduce blood glucose levels

A patient with Addison's disease comes to the emergency department with complaints of N/V/D, and fever. The nurse would expect collaborative care to include a. parenteral injections of ACTH b. IV administration of vasopressors c. IV administration of hydrocortisone d. IV administration of D5W with 20mEq of KCl

c. IV administration of hydrocortisone (rationale- vomiting and diarrhea are early indicators of addisonian crisis and fever indicates an infection, which s causing additional stress for the patient. treatment of a crisis requires immediate glucocorticoid replacement, and IV hydrocortisone, fluids, sodium and glucose are necessary for 24hours. Addison's disease is a primary insufficiency of the adrenal gland, and ACTH is not effective, nor would vasopressors be effective with the fluid deficiency of Addison's. Potassium levels are increased in Addison's dz, and KCl would be contraindicated.)

Planning: PTH insufficiency

client's serum calcium level will be w/in normal range

Evaluation: antithyroids

evaluate effectiveness in decreasing signs/symptoms of hyperthyroidism if signs/symptoms persist after 2-3 wks of therapy, other methods may be necessary

A patient with DI is treated with nasal desmopression. The nurse recognize that the drug is not having an adequate therapeutic effect the the patient experiences a. headache and weight gain b. nasal irritation and nausea c. a urine specific gravity of 1.002 d. an oral intake greater than urinary output

c. a urine specific gravity of 1.002 (rationale- normal urine specific gravity is 1.003 to 1.030, and urine with a specific gravity of 1.002 is very dilute, indicating that there continues to be excessive loss of water and that treatment of DI is inadequate. H/A, weight gain, and oral intake greater the urinary output are signs of volume excess that occur with overmedication. Nasal irritation & nausea may also indicate overmedication.)

A classic drug interaction, greatly involving an increased bleeding time, involves warfarin and a. vitamin B-6 b. acetaminophen c. acetylsalicylic acid d. all of the above

c. acetylsalicylic acid

When providing discharge instructions to a patient following a subtotal thyroidectomy, the nurse advises the patient to a. never miss a daily dose of thyroid replacement therapy b. avoid regular exercise until thyroid function is normalized c. avoid eating foods such as soybeans, turnips, and rutabagas d. use warm salt water gargles several times a day to relieve throat pain

c. avoid eating foods such as soybeans, turnips, and rutabagas (Rationale- when a patient has had a subtotal thyroidectomy, thyroid replacement therapy is not given, because exogenous hormone inhibits pituitary production of TSH and delays or prevents the restoration of thyroid tissue regeneration. However, the patient should avoid goitrogens, foods that inhibit thyroid, such as soybeans, turnips, rutabagas, and peanut skins. REgular exercise stimulates the thyroid gland and is encourage. Salt water gargles are used for dryness and irritation of the mouth and throat following radioactive iodine therapy.)

Fosphenytoin (Cerebyx) a. is a controversial agent for depression b. is used to control tremors due to Parkinsonism c. can be administered intravenously d. is ineffective after 5 days of therapy

c. can be administered intravenously

SIADH: high or low- urine output

low

SIADh: high or low plasma osmolality

low

To prevent complications in the patient with Cushing syndrome, the nurse monitors the patient for a. hypotension b. hypoglycemia c. cardiac arrhythmias d. decreased cardiac output

c. cardiac arrhythmias (rationale- electrolyte changes that occur in Cushing syndrome include sodium retention and potassium excretion by the kidney, resulting in hypokalemia, which may lead to cardiac arrhythmias or arrest. Hypotension, hypoglycemia, and decreased cardiac strength and output are characteristic of adrenal insufficiency.)

A patient is admitted to the hospital in thyrotoxic crisis. On physical assessment of the patient, the nurse would expect to find a. hoarseness and laryngeal stridor b. bulging eyeballs and arrhythmias c. elevated temperature and signs of heart failure d. lethargy progressing suddenly to impairment of consciousness

c. elevated temperature and signs of heart failure (rationale- a hyperthyroid crisis results in marked manifestations of hyperthyroidism, with fever tachycardia, heart failure, shock, hyperthermia, agitation, N/V/D, delirium, and coma. Although exophthalmos may be present in the patient with Gravs' dz, it is not a significant factor in hyperthyroid crisis. Hoarsness and laryngeal stridor are characteristic of the tetany of hypoparathyroidism, and lethargy progressing to coma is characteristic of myxedema coma, a complication of hypothyroidism.

A patient is scheduled for bilateral adrenalectomy. During the postoperative period, the nurse would expect administration of corticosteroids to be a. reduced to promote wound healing b. withheld until symptoms of hypocortisolism appear c. increased to promote an adequate response to the stress of surgery d. reduced because excessive hormones are released during surgical manipulation of the glands

c. increased to promote an adequate response to the stress of surgery (rationale- although the patient with Cushing syndrome has excess corticosteroids, removal of the glands and the stress of surgery require that high doses of cortisone be administered postoperatively for several days. The nurse should monitor the patient postoperatively to detect whether large amounts of hormones were released during surgical manipulation and to ensure the healing is satisfactory.)

TH: Planning

client's signs/symptoms of hypothyroidism will be alleviated w/in 2-4 wks w/ prescribed TH replacement, and client won't experience side effects Client's signs/symptoms of hyperthyroidism will be alleviated in 1-3 wks w/ prescribed antithyroid drug

During assessment of the patient with acromegaly, the nurse would expect the patient to report a. infertility b. dry, irritated skin c. undesirable changes in appearance d. an increase in height of 2 to 3 inches per year

c. undesirable changes in appearance (Rationale- the increased production of growth hormone in acromegaly causes an increase in thickness and width of bones and enlargement of soft tissues, resulting in marked changes in facial features, oily and coarse skin, and speech difficulties. Height is not increased in adults with growth hormone excess because the epiphyses of the bones are closed, and infertility is not a common finding because growth hormone is usually the only pituitary hormone involved in acromegaly.)

Antidote for warfarin overdose a. protamine zinc insulin b. protamine sulfate c. vitamin K d. warfarin

c. vitamin K

A client who incurred an arm injury describes his pain as "sharp and localized to the lower arm." The nurse recognizes that this type of pain would be relieved best by administration of which type of medication? a.) Muscle relaxant b.) Acetaminophen c.) Narcotic analgesics d.) Ice packs

c.) Narcotic analgesics Injury to tissues produces nociceptor pain, which usually responds to conventional analgesic pain medications such as opiates or NSAIDS.

The client admitted with hepatitis B is prescribed Vicodin tabs 2 for treatment of pain. The appropriate nursing action is to: a.) Administer the drug as ordered. b.) Administer one tablet only. c.) Question the physician about the order. d.) Hold the drug until the physician arrives.

c.) Question the physician about the order. Vicodin is a combination drug of hydrocodone and acetaminophen. Acetaminophen can be hepatotoxic, and is contraindicated in liver disease.

Celecoxib (Celebrex) is added to the treatment regimen of a client with arthritis. The nurse explains that the major advantage of this drug is: a.) The drug is less expensive. b.) The drug has no known side effects. c.) The drug has anti-inflammatory properties. d.) The drug's effectiveness is the same as opioids.

c.) The drug has anti-inflammatory properties. Celecoxib (Celebrex) has anti-inflammatory properties. It is not less expensive, has many side effects, and is less potent than opioids.

A client in the ICU tells the nurse he is experiencing severe pain. Prior to administering a narcotic analgesic to this client, the nurse will conduct a pain assessment to include: a.) Pain b.) Nociception c.) Pain behaviors d.) Suffering

c.) pain behavior There is a theory that addresses pain as having four facets: nociception, pain, suffering, and pain behaviors. Of these four facets, only the fourth, pain behavior, can be observed. This nurse will only be able to assess the client's pain behavior in the pain assessment.

3 classifications of DI

central (neurogenic DI), Nephrogenic DI, and Primary (psychogenic DI)

Planning: pit hormones

client will be free of pituitary disorder with appropriate drug regimen

hyperparathyroidism: high or low serum phosphate

low

hydrocodone/acetaminophen/ Vicodin

combination with nonopioid analgesic in the management of moderate to severe pain. NI: use with extreme caution in pts. recieving MAO inhibitors asses bowel patter, vitals pain Toxicity and overdose: Narcan is the antidote

importance of parathyroid

controls Ca in blood and bones

hypothyroidism that develops in infancy and is caused by thyroid hormone deficiencies during fetal or early neonatal life

cretinism

A physician has prescribed propylthiouracil (PTU) for a client with hyperthyroidism and the nurse develops a plan of care for the client. A priority nursing assessment to be included in the plan regarding this medication is to assess for: a) relief of pain b) signs of renal toxicity c) signs and symptoms of hyperglycemia d) signs and symptoms of hypothyroidism

d) signs and symptoms of hypothyroidism Excessive dosing with propylthiouracil (PTU) may convert the client from a hyperthyroid state to a hypothyroid state. If this occurs, the dosage should be reduced. Temporary administration of thyroid hormone may be required. Propylthiouracil is not used for pain and does not cause hyperglycemia or renal toxicity.

A patient with Grave's dz asks the nurse what caused the disorder. The best response by the nurse is a. "The cause of Grave's disease is not known, although it is thought to be genetic." b. "It is usually associated with goiter formation from an iodine deficiency over a long period of time." c. "Antibodies develop against thyroid tissue and destroy it, causing a deficiency of thyroid hormones" d. "In genetically susceptible persons antibodies form that attack thyroid tissue and stimulate overproduction of thyroid hormones."

d. "In genetically susceptible persons antibodies form that attack thyroid tissue and stimulate overproduction of thyroid hormones." (rationale- The antibodies present in Graves' disease that attack thyroid tissue cause hyperplasia of the gland and stimulate TSH receptors on the thyroid and activate the production of thyroid hormones, creating hyperthyroidism. The disease is not directly genetic, but individuals appear to have a genetic susceptibility to become sensitized to develop autoimmune antibodies. Goiter formation from insufficient iodine intake is usually associated with hypothyroidism.)

hypothyroidism: high or low T4

low

When an elderly client with cancer experiences "breakthrough pain," the nurse should expect that pharmacological treatment will include: a. Initiation of a placebo after every third dose of narcotic. b. More aggressive chemotherapy. c. Giving narcotics every hour. d. Increasing the dose of the narcotic.

d. Increasing the dose of the narcotic.

The safest narcotic choice for an elderly client with acute pain is: a. Meperidine (Demerol). b. Oxycodone. c. Fentanyl transdermal patch. d. Morphine sulfate.

d. Morphine sulfate. Rationale: Morphine is the "gold standard" of narcotics for acute pain. The other choices are incorrect.

hypothyroidism: high or low basal metabolic rate

low

The benefits of using an insulin pump include all of the following except: a. By continuously providing insulin they eliminate the need for injections of insulin b. They simplify management of blood sugar and often improve A1C c. They enable exercise without compensatory carbohydrate consumption d. They help with weight loss

d. They help with weight loss Using an insulin pump has many advantages, including fewer dramatic swings in blood glucose levels, increased flexibility about diet, and improved accuracy of insulin doses and delivery; however, the use of an insulin pump has been associated with weight gain.

A patient suspected of having acromegaly has an elevated plasma growth hormone level. In acromegaly, the nurse would also expect the patient's diagnostic results to include a. hyperinsulinemia b. a plasma glucose of less than 70 c. decreased growth hormone levels with an oral glucose challenge test d. a serum sometomedin C (insulin-like growth-factor) of more than 300

d. a serum somatomedin C (Insulin-like-growth-factor) of more than 300 (rationale- a normal response to growth hormone secretion is stimulation of the liver to produce somatomedin C which stimulates growth of bones and soft tissue. The increased levels of somatomedin C normally inhibit growth hormone, but in acromegaly the pituitary gland secretes GH despite elevated somatomedin C levels.)

Causes of primary hypothyroidism in adults include a. malignant or benign thyroid nodules b. surgical removal or failure of the pituitary gland c. surgical removal or radiation of thyroid gland d. autoimmune-induced atrophy of the gland

d. autoimmune-induced atrophy of the gland (rationale- both Graves disease and Hasimotos thyroiditis are autoimmune disorders that eventually destroy the thyroid gland, leading to primary hypothyroidism. Thyroid tumors most often result in hyperthyroidism. Secondary hypothyroidism occurs as a result of pituitary failure, and iatrogenic hypothyroidism results from thyroidectomy or radiation of the thyroid gland.)

Physical changes of hypothyroidism that must be monitored when replacement therapy is started include a. achlorhydria and constipation b. slowed mental processes and lethargy c. anemia and increased capillary fragility d. decreased cardiac contractility and coronary atherosclerosis

d. decreased cardiac contractility and coronary atherosclerosis (rationale- hypothyroidism affects the heart in many ways, causing cardiomyopathy, coronary atherosclerosis, bradycardia, pericardial effusions, and weakened cardiac contractility. when thyroid replacement therapy is started, myocardial oxygen consumption is increased and the resultant oxygen demand may cause angina, cardiac arrhythmias, and heart failures. It is important to monitor patients with compromised cardiac status when starting replacement therapy.)

In a patient with central diabetes insipidus, administration of aqueous vasopressin during a water deprivation test will result in a a. decrease in body weight b. increase in urinary output c. decrease in blood pressure d. increase in urine osmolality

d. increase in urine osmolality (rationale- a patient with DI has a deficiency of ADH with excessive loss of water from the kidney, hypovolemia, hypernatreamia, and dilute urine with a low specific gravity. When vasopressin is administered, the symptoms are reversed, with water retention, decreased urinary output that increases urine osmolality, and an increase in blood pressure.)

SIADH: high or low- sodium levels

low (dilutional hyponatremia)

The client informs the nurse that he has experienced pain in the lower extremities for the past eight months. The nurse recognizes that this pain is classified as: a.) Moderate. b.) Severe. c.) Acute. d.) Chronic.

d.) Chronic. Chronic pain persists longer than six months.

The nurse administers morphine sulfate 4 mg IV to a client for treatment of severe pain. Which of the following assessments requires immediate nursing interventions? a.) Blood pressure 110/70 b.) The client is drowsy. c.) Pain is unrelieved in 15 minutes. d.) Respiratory rate 10/minute

d.) Respiratory rate 10/minute Opioids activate mu and kappa receptors that can cause profound respiratory depression. Respiratory rate should remain above 12. The BP is not significantly low. Drowsiness is an expected effect of morphine. Unrelieved pain warrants further assessment, but not as immediately as do decreased respirations.

how to test for addison's disease

decrease plasma cortisol level, ACTH stimulation test, CT, MRI

how to diagnosis hyperthyroidism

decreased TSH levels; elevated T4 levels; RAIU test

hypothyroidism

decreased metabolism, fat, decreased HR, intolerance to cold, lethargic

What does Metformin, Glucaphage do to help with diabetes?

decreases glucose release from the liver decreases insulin resistance

Nursing Diagnoses: adrenal hormone

deficient knowledge of drug discontinuation related to lack of recall excess fluid volume related to retention risk for impaired tissue integrity related to side effect of rash

gastroparesis

delayed gastric emptying

Client Teach: self-admin

demonstrate how to take pulse. Instruct to monitor pulse and report increases/marked decreases

endogenous

derived or originating internally

Nursing interventions: adrenal hormones

determine vitals: glucocorticoids can increase BP and Na and water retention Admin these only as ordered. Routes include: PO, IM (not deltoid), IV, aerosol, and topical Topicals should be applied in thin layers. Rashes, infection, and purpura should be noted and reported Record weight. Report gain of 5 lbs in several days: mostly likely b/c of water retention Monitor labs, especially electrolytes: serum K could decrease to below 3.5, and glycemia will probably increase Watch for signs of hypokalemia:nausea, vomit, muscular weakness, abdom distention, paralytic ileus, and irregular HR Asses for side effects when therapy has lasted more than 10 days and drug is taken in high dose. Cortisone preparation should not be abruptly stopped, b/c adrenal crisis can result Monitor oldies for signs of increased osteoporosis. These can promote Ca loss from bone Report changes in muscle strength. High doses promote loss of muscle tone

What are the examples of first generation sulfas?

diabinese tolinase

Nursing Diagnoses: PTH insufficiency

diarrhea related to adverse effect of calcitriol deficient fluid vol related to fluid loss from vomit/polyuria

Client teaching: hypoparathyroidism

direct client to report symptoms of tetany

retinopathy

disease of the retina

Deficiency of GH

dwarfism

how to diagnosis Cushings

elevated plasma cortisol, 24 hour urine collection for free cortisol, CT, MRI

S&S of acromegaly

enlargement of hands and feet, joint pain, thickening and enlargement of bony and soft tissues on the face and head, enlargement of the tongue, voice deepening, sleep apnea, thick leathery and oily skin, menstrual disturbances

drugs to treat hyperparathyroidism

low Ca levels: Fosamax, sensipar

increase secretion of parathyroid hormone (PTH)

hyperparathyroidism

Contraindications for Prednisone

hypersensitivity, psychosis, fungal infection Caution: diabetes

What side effects should the patient report when taking sulfas?

hypoglycemia, nausea, epigastric fullness, heartburn

S&S of SIADH

hyponatremia (muscle cramping, pain, weakness), thirst, dyspnea on exertion, fatigue, increase body weight without edema severe: vomiting, abdominal cramps, muscle twitching, seizures even more severe: cerebal edema, lethargy, anorexia, confusion, headache, seizure, coma

how to treat Addisonian crisis

hypotension may lead to shock-give high dose hydrocortisone replacement and large volumes of 0.9% NS and D5@ to reverse hypotension and electrolyte imbalances

insufficient circulating thyroid hormone as a result of a variety of abnormalities

hypothyroidism

most common cause of Cushing syndrome

iatrogenic administration of exogenous corticosteroids (prednisone); ACTH

how to diagnosis DI

identify cause and type

When give HGH to affect growth?

in childhood before bones fuse

how to treat nephrogenic DI

low sodium diet, thiazide diuretics; indomethacin (indocin)

how to diagnosis hypothyroidism

increase TSH, decrease T4

DLF: synthroid

increase cardiac insufficiency with epinephrine increases effects of anticoagulants, tricyclic antidepressants, vasopressors, decogestants decreases effects of antidiabetics(oral and insulin) and digitalis products decreases absoprtion of cholestyramine and colestipol

S&S of hyperthyroidism

increase metabolism, goiter, exophthalmos, increase HR, increase RR, increase appetite, thirst, weight loss, increase peristalsis, warm, smooth, moist skin, hair loss, intolerance to heat, increase nervousness

DLF for prednisone

increased effect with estrogents, diltiazem, ketoconazole decreased effect with barbs, phenytoin, rifampin Concurrent use of aspirin and NSAIDs increase GI toxicity Concurrent use of diuretics and amphotericin B increase K depletion Concurrent use with cardiac glycosides increase risk of dysrhythmias and digitalis toxicity

hyperthyroidism

increased metabolism, thin, bulging eyes, increased HR, irritable, heat intolerance

insulin needs

increased need when stressed or sick

Kussmaul's respirations

increased rate and depth with panting and long grunting exhalation

TH: Nursing diagnoses

ineffective health maintenance related to inability to maintain drug regimen ineffective tissue perfusion related to hyposecretion of TH activity intolerance related to generalized weakness

DI: high or low: urine specific gravity

low: diluted

glycopyrrolate/ Robinul

inhibits salivation and excessive respiratory secretions when given preop. Assess vitals, for peds assess hyperexcitability, monitor intake and output ratios, assess abdomen. toxicity and overdose: neostigmine is the antidote

importance of pancreas

insulin and glucagon production

NPH INSULIN

intermediate acting insulin

most common cause of hypothyroidism

iodine deficiency

hyperparathroidism

leads to hypercalcemia; causes moodiness

TSH

leads to secretion of T3 and T4

Ultralente

long acting insulin. Onset: 4 to 6 hrs. Peak: 14 to 24 hrs. Duration: 28 to 36 hrs. Mode of Action: Insulin promotes use of glucose by body cells. It is evently distributed over a 24-hr duration of action; thus it is administered once a day, usually at bedtime. Effects: Confusion, agitation, tremors, headache, flushing, hunger, weakness, lethargy, fatigue, hunger, weakness, lethargy, fatigue, urticaria; redness, irritation, or swelling at insulin injection site. Adverse reactions: tachycardia, palpitations, hypoglycemic reactions, rebound hyperglycemia. Life-threatening: shock, anaphylaxis. Nursing Interventions: Monitor vital signs (tachycardia can occur). Determine blood glucose levels and report changes. Monitor the clients HbA1c. Prepare a teaching plan based on client\'s knowledge of health problem, diet, and drug therapy. Teaching: Instruct client to report symptoms of hypoglycemic (insulin) reaction: headache, nervousness, sweating, tremors, rapid pulse; and symptoms of hyperglycemic reaction (diabetic acidosis): thirst, increased urine output. Advise client that hypoglycemic reactions are more likely to occur during the peak action time. Explain that orange juice, sugar-containing drinks may be used when hypoglycemic reactions begin. Teach family members to administer glycagon by injection of client has a hypoglycemic reaction. Inform client that certain herbs may react with insulin and oral antidiabetic drugs. Instruct client about the necessity for compliance with prescribed insulin therapy and diet. HbA1c provides most accurate picture of optimal diabetic control. Advise client to carry a MedicAlert card, tag, or bracelet. Instruct client how to check blood glucose with a glucometer.

Lantus

long acting insulin. subQ onset: 1 h. Peak: none. Duration: 24 h. Mode of Action: Insulin promotes use of glucose by body cells. It is evently distributed over a 24-hr duration of action; thus it is administered once a day, usually at bedtime. Effects: Confusion, agitation, tremors, headache, flushing, hunger, weakness, lethargy, fatigue, hunger, weakness, lethargy, fatigue, urticaria; redness, irritation, or swelling at insulin injection site. Adverse reactions: tachycardia, palpitations, hypoglycemic reactions, rebound hyperglycemia. Life-threatening: shock, anaphylaxis. Nursing Interventions: Monitor vital signs (tachycardia can occur). Determine blood glucose levels and report changes. Monitor the clients HbA1c. Prepare a teaching plan based on client\'s knowledge of health problem, diet, and drug therapy. Teaching: Instruct client to report symptoms of hypoglycemic (insulin) reaction: headache, nervousness, sweating, tremors, rapid pulse; and symptoms of hyperglycemic reaction (diabetic acidosis): thirst, increased urine output. Advise client that hypoglycemic reactions are more likely to occur during the peak action time. Explain that orange juice, sugar-containing drinks may be used when hypoglycemic reactions begin. Teach family members to administer glycagon by injection of client has a hypoglycemic reaction. Inform client that certain herbs may react with insulin and oral antidiabetic drugs. Instruct client about the necessity for compliance with prescribed insulin therapy and diet. HbA1c provides most accurate picture of optimal diabetic control. Advise client to carry a MedicAlert card, tag, or bracelet. Instruct client how to check blood glucose with a glucometer.

Impaired vision - definition

loss of vision such that additional support is needed

Hyperthyroidism: high or low LDL

low

Hyperthyroidism: high or low TSH

low

celecoxib/ Celebrex

management of acute care. used for endoscopic surveillance, surgery NI: assess pt. for allergy to sulfonamides, asprin, or NSAIDs, monitor for signs of Stevens-Johnson syndrome and toxic epidermal necrolysis. Discontinue celecoxib at first sign of rash.

treatment for addison's disease

management of underlying cause; adrenocoritcal replacement therapy (hydrocortisone), increase salts added to diet

Importance of thyroid

metabolism

acetaminophen/ Tylenol

mild pain, fever NI: toxicity and overdose- if overdose occurs. acetylcysteine (acetadote) is the antidote

Long acting

mimics body with constant slow supply

Oxycodone/ acetaminophen/ Percocet

moderate to severe pain NI: assess pain. assess vitals physical stimulation may be sufficient to prevent significant hypoventilation. drowsiness will deminish; narcan antidote for overdose/toxicity

Hydromorphone/ Dilaudid

moderate to severe pain,extended release product for opioid-tolerant patients requiring around-the-clock management of persistent pain NI:assess vitals, initial drowsiness will diminish with continued use. assess pediatric and geriatric patients more frequently. assess pain, & bowel function narcan antidote for overdose/toxicity

nursing interventions: GH

monitor glycemia and electrolyte levels. Hyperglycemia can occur with high doses

Nursing Interventions: PTH insufficeny

monitor serum CA level: normal = 4.5-5.5 mg/dL(ionized) or 9-10.5 (total) Serum ionized Ca levels are usually used, b/c much of the Ca is protein-bound and is nonionized and nonactive

nursing interventions: ADH

monitor vitals: increased HR and decreased systolic BP can indicate fluid vol loss resulting from decreased ADH production. W/ less ADH secretion, more water is excreted, decreasing vascular fluid (hypovolemia) Record urinary output: increased output can indicate fluid loss by a decrease in ADH

accumulation of hydrophillic mucopolysaccharides in the dermis and other tissues; puffiness, periorbital edema

myxedema

Side effects: Synthroid

nausea, vomit, diarrhea, cramps, tremors, nervousness, insomnia, headache, weight loss

Myopia - definition

nearsightedness; blurry distance vision

Emmetropia - definition

normal vision: no need for correction

Pit Hormones: assessment

obtain baseline vitals determine urinary output and weight assess for infectious process. Coriticotropin can suppress signs of infection Note client's physical growth. Compare child's growth with reported standards

glaucoma - open angle assessment

pallor of optic nerve, cupping of optic disk, elevated IOP

Adverse effects of Prednisone

petechiae, ecchymosis, hypertension, tachycardia, osteoporosis, muscle wasting Life-threatening: GI hemorrhage, pancreatitis, circulatory collapse, thrombophlebitis, embolism

S&S of DI

polydipsia, polyuria, increase HR, decrease BP, hypovolemic shock

3 P's of diabetes

polydipsia, polyuria, polyphagia

What side effects does Starlix have?

possible hypoglycemia, upper respiratory infection, diarrhea

What are goal values of blood glucose for pre-meal and at bedtime for the diabetic?

pre-meal: 80-120 bedtime: 100-140

preprandial

preceding a meal (especially dinner)

less common condition, associated with excessive water intake; structural lesion in the thirst centre or may be caused by psychiatric problems

primary (psychogenic) DI

hypothyroidism has 2 classifications

primary and secondary

hypothyroidism: 1 or 2-destruction of thyroid tissue or defective hormone synthesis

primary hypothyroidism

B-cells of pancreas

produce insulin in response to raising glycemia

S&S of Addison's

progressive weakness, fatigue, weight loss, anorexia, skin hyper pigmentation, orthostatic hypotension, hyponatremia, hyperkalemia, N&V, diarrhea

Growth Hormone

promotes growth of tissues and bones; works on newly forming bones

antithyroid drugs for hyperthyroidism

propylthiouracil (PTU), methimazole (Tapazole); iodine, beta blockers

Which organ is destroyed when administering radioactive I-131? a.) Pituitary gland b.) Adrenals c.) Parathyroid d.) Hypothalamus

radioactive I-131 destroys the thyroid because the thyroid cells are the only cells in the body that can take up iodine

Insulin Types

rapid acting: onset = 5-15 min Short acting: regular; peak = 2-4h; duration 6-8h intermediate: NPH peak: 6-12h; Lente duration: 18-24h Long-acting: evenly over 24h

Which insulins are clear?

rapid and short acting and Lantus

TH: nursing interventions

record vitals: w/ hypothyroidism, the temp, HR, and BP are usually decreased; w/hyper, tachycardia and palpitations usually occur Monitor client's weight. Weight gain commonly occurs with hypothyroidism

glaucoma -OA Tx

reduce IOP 20-50%, w/ topical or oral agents, laser trabeculoplasty

treatment for thyrotoxic crisis

reducing circulating thyroid hormone levels and clinical manifestations

polyuria

renal disorder characterized by the production of large volumes of pale dilute urine

treatment of hypothyroidism

restoration of euthyroid state as safely and rapidly as possible with hormone replacement; low calorie diet to promote weight loss

management of SIADH

restrict total fluids no more than 1000mL/day, HOB no more than 10 degrees

treatment of mild SIADH (serum sodium greater than 125)

restriction of fluids to 800-1000mL/day

therapeutic goal in acromeglay

return GH levels to normal; surgery, irridiation, drug therapy, or combination

hypothyroidism: 1 or2- pituitary disease with decreased TSH secretio of hypothalamic dysfunction with decrease TRH secretion

secondary hypothyroidism

S&S of thyrotoxic crisis

severe tachycardia, heart failure, shock, hyperthermia, restlessness, agitation, seizures, abdominal pain, nausea, vomiting, diarrhea, delirium, coma

Hypoglycemia symptoms

shaky, confusion

Regular

short acting insulin. Trade name: Humulin R. subQ onset: 30 to 60 min. Peak: 2 to 3 h. Duration: 4 to 8 h. Generally given 30 minutes before meals. Mode of Action: Insulin promotes use of glucose by body cells. Effects: Confusion, agitation, tremors, headache, flushing, hunger, weakness, lethargy, fatigue, hunger, weakness, lethargy, fatigue, urticaria; redness, irritation, or swelling at insulin injection site. Adverse reactions: tachycardia, palpitations, hypoglycemic reactions, rebound hyperglycemia. Life-threatening: shock, anaphylaxis. Nursing Interventions: Monitor vital signs (tachycardia can occur). Determine blood glucose levels and report changes. Monitor the clients HbA1c. Prepare a teaching plan based on client\'s knowledge of health problem, diet, and drug therapy. Teaching: Instruct client to report symptoms of hypoglycemic (insulin) reaction: headache, nervousness, sweating, tremors, rapid pulse; and symptoms of hyperglycemic reaction (diabetic acidosis): thirst, increased urine output. Advise client that hypoglycemic reactions are more likely to occur during the peak action time. Explain that orange juice, sugar-containing drinks may be used when hypoglycemic reactions begin. Teach family members to administer glycagon by injection of client has a hypoglycemic reaction. Inform client that certain herbs may react with insulin and oral antidiabetic drugs. Instruct client about the necessity for compliance with prescribed insulin therapy and diet. HbA1c provides most accurate picture of optimal diabetic control. Advise client to carry a MedicAlert card, tag, or bracelet. Instruct client how to check blood glucose with a glucometer.

why should any clear nasal drainage be monitors after transsphenoidal dhypophysectomy (acromegaly surgery)

should be tested for glucose; glucose greater than 30mg/dL indicates CSF leakage

Ways to decide how much insulin to give

sliding scale or formula based

Addisonian crisis can be triggered by

stress (infection, surgery, trauma, hemorrhage, psychological distress), sudden withdrawal of corticosteroid hormone replacement therapy, adrenal surgery, sudden pituitary gland destruction

Treatment in addition to diet and exercise should include...?

sulfonlurea therapy is effective for most patients - these are oral diabetic drugs specific for type 2 diabetics. Insulin is also required for 20-30% of type 2s

treatment for hypothyroidism

synthroid

Adverse Reactions: synthroid

tachycardia, hypertension, palpitations Life-threatening: thyroid crisis, angina pectoris, cardiac dysrhythmias, CV collapse

Recovery from hyperglycemia (over 200)

takes two weeks to recover each time it goes over 200

Client Teaching: side effects

teach side effects of anithyroids: skin rash, hives, nausea, alopecia, loss of hair pigment, petechiae, ecchymoses, and weakness Advise client to contact HCP if a sore throat/fever occur while taking antithyroids. A serious adverse reaction is agranulocytosis. A complete blood count should be monitored for leukopenia.

Pituitary gland aka

the "Master Gland"

Planning: adrenal hormone

the client's side effects of glucocorticoid therapy will be minimal Client's inflammatory response will decrease

What is the preferred test to diagnose for diabetes?

the fasting blood glucose test

Glycosuria results when...

the kidneys are unable to reabsorb all the glucose in the renal tubules

glycosuria

the presence of abnormally high levels of sugar in the urine

nursing implementation after acromegaly surgery

transssphenoidal hypophysectomy: elevated HOB 30 degrees, monitor neurological status, avoid vigorous coughing, sneezing, straining at stools, monitor any clear nasal drainage; perform mouth care every 4 hours to keep surgical area clean, avoid tooth brushing for 10 days to prevent disrupting the suture line and avoid discomfort

how to treat DI

treat underlying cause

calcitriol

treats hypocalcemia. Class: VD analogue. Trade name: Rocatrol. Dosage: 25 mcg/d PO. Action: increase calcium deposit in bones. Side effects: Drowsiness, photophobia, GI upset. Contra: hypersensitivity, hypercalcemia, increased Vitamin D, malabsorption. Nurse: Ca level 4.5-5.5 mg/dL. Teaching: report symptoms of hypercalcemia; bone pain, anorexia, lethargy, bradycardia, polyuria

intermediate acting

used for someone w/ constant hyperglycemia

rapid-acting

used pre-meal

Anti-diarrheals

used to stop diarrhea and decrease GI motility

short-acting

usually for someone that needs insulin all the time

diagnositic test specifically for Central DI

water deprivation test

S&S of cushings

weight gain, truncal obesity, purple striae, slow wound healing, thin skin and subcutaneous tissue, buffalo hump, moon face, thinning of hair, hirsutism in women, menstraual disorders, hypertension, unexplained hypokalemia

Monitoring glucose: When should patients with type 1 diabetes check their urine for ketones?

when they have persistently elevated blood sugar levels, during illness and when pregnant

Meniere's disease is most common in who?

white men ages 20-50

expiration or insulin

with fridge: 3 mo w/o: 30 days


Related study sets

Strategic Management - Chapter 3

View Set

Health and Illness: exam 3 practice questions

View Set

A1.1. Grammar Course. Class 9: Some / a / an / any / no

View Set

Science - Physics - Electricity and Energy Unit 2

View Set